You are on page 1of 240

i

Contents

5
Olympiad Champs
Mathematics
INDIA'S FIRST BOOK FOLLOWING
National Curriculum Framework 2005

v Use of Interactive and Self Indulging language.

v Quick Concept Review with Diagrams & Illustrations.

v Amazing Facts, Real-life Examples, Historical Preview, Concept-


Misconcept, Short-cut to Problem Solving.

v Exercises in MCQ format in 2 difficulty levels.

v Level 1 contains Beginner's level MCQs - Odd One Out, Analogy,


Fillers.

v Level 2 contains Advanced level MCQs - Matching, Chronological


order, Feature based, Integer based, Statement Correct-Incorrect,
Puzzle based, Grid based, Crossword, Venn diagrams, Table/ Chart
based MCQs.

v Comprising questions of various National and International


Olympiads.
EBD_7364
Corporate. Office:- 45, 2nd Floor, Maharishi Dayanand Marg, Corner Market, Malviya
Nagar, New Delhi-110017

Tel.: 011-49842349/49842350

Typeset by Disha DTP Team

DISHA PUBLICATION
ALL RIGHTS RESERVED

© Copyright Publisher
No part of this publication may be reproduced in any form without prior permission of the
publisher. The author and the publisher do not take any legal responsibility for any errors
or misrepresentations that might have crept in. We have tried and made our best efforts to
provide accurate up-to-date information in this book.
or further information about the books from DISHA,
Log on to www.dishapublication.com or www.aiets.co.in or email to info@aiets.co.in

(ii)
iii
Contents

Preface
We are pleased to launch the thoroughly revised Third edition of Olympiad Champs
Mathematics Class 5 which is the first of its kind book on Olympiad in many ways.
The Unique Selling Proposition of this new edition is the inclusion of past year questions of
different Olympiad exams held in schools.
The book is aimed at achieving not only success but deep rooted learning in children. It is
prepared on content based on National Curriculum Framework prescribed by NCERT. All the
text books, syllabi and teaching practices within the education programme in India must follow
NCF. Hence, Olympiad Champs become an ideal book not only for the Olympiad Exams but also
for strengthening the concepts for the relevant grade.
There is an exhaustive range of thought provoking questions in MCQ format to test the
student’s knowledge thoroughly. The questions are designed so as to test the knowledge,
comprehension, evaluation, analytical and application skills. Solutions and explanations are
provided for all questions. The questions are divided into two levels-Level 1 and Level 2. The
first level, Level 1, is the beginner’s level which comprises of questions like fillers, analogy
and odd one out. When the children covers Level 1, it means his basic knowledge about the
subject is clear and now he is ready for Level 2. The second level is the advanced level.
Level 2 comprises of techniques like matching, chronological sequencing, picture, passage and
feature based, statement correct/ incorrect, integer based, puzzle, grid based, crossword,
venn diagram, table/ chart based and much more.
The first concern which each parent faces is how to make their children read a book especially
when it is based on academics. Keeping this in mind interesting facts, real life examples,
historical preview, short cuts to problem solving, charts, diagrams, illustrations and poems are
added.
With the vision to remove all the misconception a child may have pertaining to the subject, to
relate his knowledge to the real world and to develop a deeper understanding of the subject,
this book will cater all the requirements of the students who are going to appear in Olympiads.
While preparing this book, some errors might have crept in. We request our readers to identify
those errors and send it across on feedback_disha@aiets.co.in.
We wish you all the best for your Olympiads and happy reading…….
 Team Disha
 For feedback : feedback_disha@aiets.co.in.

(iii)
EBD_7364
iv Contents

Contents
1. Roman Numerals 1-14

2. Number System 15-30

3. Operations on Numbers 31-50

4. Factors and Multiples 51-68

5. Fractions and Operation on Fractions 69-94

6. Decimals 95-116

7. Bodmas Application 117-132

8. Ratio 133-148

9. Temperature 149-164

10. Area and Perimeter 165-186

11. Geometry 187-216

12. Data Handling 217-236

(iv)
Roman Numerals 1

CHAPTER FOREWORD
You are familiar with Arabic number system, but here you will learn another way of representing
numbers called Roman Numerals.
In school, we usually write our class as I-A, II-C, IX-D etc, where I, II and IX represents
Roman Numerals.
Now fill the blank spaces with suitable Roman numerals.

X
VIII

÷ =
II

=
= +
V
II
Write down your birthday month in Roman numeral __________.
How would you write 'zero' Roman numeral ? __________

After reading this chapter, you will be able to understand more about Roman Numerals.

Note : There is no symbol for zero in Roman numbers .


We write Roman numbers from 1. V is the symbol to represent 5, X represents 10, L is
for 50, C represent 100, D is the symbol for 500 and 100 is represented by M. A symbol
can be repeated at most 3 times.
EBD_7364
2 Olympiad Champs–Mathematics

Chapter
1
Roman Numerals
LEARNING OBJECTIVES
This lesson will help you to:—
v understand Roman numbers.
v learn rule of Roman numerals.

QUICK CONCEPT REVIEW

Historical Preview We use numbers every day. When we get up in the morning
we see the clock and read the time with numbers. We wait
v Roman numerals originally came for our bus number to go to school. We go to our class say
from Etruscan numerals which
class 5 or 6. We learn mathematics in the school. We
were changed. The Roman
numerals came in to use from count money using numbers only.
4th century B.C. Do you know, what are the numbers (0 1 2 3 4 5 6 7 8 9)
which we use everywhere called? These are the Arabic
numbers. These numerals came from Arabia.
But apart from these numbers, we use some symbols too,
which represent mathematical values. These numerals are
the Roman numerals.
Roman numerical system uses combination of letters from
Try It ! Latin alphabets to present mathematical numbers.

Example : Write Roman number In this chapter we shall learn about the Roman numerals.
for 54 and 108 In Roman numeral system, symbols are used to represent
Solution : the numbers.
54 → LIV Rules for Roman numerals:
108 → CVIII
I. The first three numbers (1, 2, 3) are represented
as → I, II and III.
II. The symbol for 5 is V.
III. One symbol cannot be used more than thrice
continuously. That is why we do not write 4 as IIII.
This is wrong.
Roman Numerals 3
IV. If a letter is placed before the letter of greater
value then subtract it from the greater number.
For example, I represents 1 and V represents 5, Real-Life Examples
so when IV is written this means 5 - 1 = 4. So IV
v In some watches or clocks you
represents 4.
can see the Roman numerals on
V. If one or more different letters are there after a the dial.
letter of greater value, then add them all.
For example VI (5 + 1 = 6), VIII (5 + 1 + 1 + 1 = 8).
VI. The symbol for ten is X.
VII. If a letter repeats itself, then add the letter that
many times. For example XXX = 10 + 10 + 10 = 30.
VIII. Only one number can be subtracted from one number.
For example: It will be wrong to write 13 = IIXV
v Roman numerals are also used
(15 - 1 - 1). 13 will be written as XIII.
in the names of monarchs and
IX. A bar or line on a number increases its value by popes. For example: Elizabeth
1000 times. For example : XV = 15 and XV = 10000 II.
+ 5000 = 15000. v You write your standard in the
school in Roman numerals. Like
Here are a few examples of some Roman numerals:
you study in class V.
I. 67=LXVII
II. 545=DXLV
III. 303=CCCIII
IV. 98=XCVIII
Remember: V, L and D can not be
Here is the Table of Roman Numerals for Numbers 1-20 repeated in Roman numbers
but I, C and M can be repeated
Roman Roman 3 times.
No. No.
Representation Representation
1 I 11 XI Some Other Important
Roman Representations
2 II 12 XII
3 III 13 XIII L C D M
4 IV 14 XIV 50 100 500 1000
5 V 15 XV EXAMPLE:
6 VI 16 XVI CM (1000 - 100) = 900

7 VII 17 XVII
8 VIII 18 XVIII
9 IX 19 XIX
10 X 20 XX
EBD_7364
4 Olympiad Champs–Mathematics
Here are some easy ways to remember some of the
Roman representations:
v The roman numeral I (One) resembles with one finger
Try It!
Amazing Facts of our hands.

I
Example : Which number is being
represented by the following
Roman Number?
(a) DCC (b) MCM
(c) LXIII (d) XLIV
(e) CDXV
Solution :
(a) DCC = 500+100+100=700 v The roman numeral V (Five) can be remembered
(b) MCM = 1000+900 = 1900 as → there are five fingers in our hand and the shape
between any two fingers of our hand resembles the
(c) LXIII = 50+10+3=63 shape of V.
(d) XLIV= 40+4=44
(e) CDXV= 400+15=415
(f) MMCC= 2000+200= 2200
Note : V, L, D Can't be repeated in
V
Roman numbers

v If we place our two hands both in opposite direction


Do You Know? but touching at one point we have ten fingers and this
Amazing Facts resembles with shape of roman numeral X (Ten).
v V, L and D are never subtracted.
v I can be subtracted from V and
X only.
v X can be subtracted from L and
C only.
v C can be subtracted from D and
M only.

v The symbol ‘C’ represents value of 100, so half of 100


will be 50.
If we do half of symbol C we get a shape which
resembles the letter representing 50 which is L
So now you are able to recognize and use the roman
numerals.
Roman Numerals 5

Multiple Choice Questions


LEVEL 1
Directions (Qs. 1 to 8) : Give the Arabic representation for the following numerals.
(Mental Mathematics)
1. CXI
(a) 112 (b) 111 (c) 110 (d) 191
2. XCIV
(a) 96 (b) 104 (c) 114 (d) 94
3. XXIII

(a)
23 (b)
28 (c)
19 (d)
14
4. LXIV

(a)
68 (b)
64 (c)
75 (d)
54
5. L

(a)
50 (b)
46 (c)
8 (d)
100
6. XLIX

(a)
69 (b)
59 (c)
49 (d)
99
7. M

(a)
900 (b)
950 (c)
1000 (d)
800
8. CDXC

(a)
490 (b)
440 (c)
400 (d)
500
9. Human beings have _____ teeth. (Tricky)
(a) L (b) XXXII (c) XL (d) C
10. L + XX + X + VI = _____. (2009, Tricky)
(a) LXXXC (b) CX (c) LXXXVI (d) LVI
11. Roman numerals have been adapted from________.
(a) British (b) Mughal (c) Etruscan (d) none of these
12. If a letter is placed before the Roman numeral of greater value, then it
is ________.
(a) subtracted from it (b) added to it
(c) multiplied to it (d) divided by it
13. L = 50, C = 80, D = 100, M = 90

(a)
TFFT (b)
FFTT (c)
TFFF (d)
FFFF
EBD_7364
6 Olympiad Champs–Mathematics
Directions (Qs. 14 to 16) : Complete the series. (Mental Mathematics)
14. CX, CXI, CXII, CXIII, _______.
(a) CXX (b) CXIV (c) CXIX (d) CXVI
15. CXX, CXXV, _______, CXXXV, CXL
(a) CL (b) CXXX (c) CL (d) CD
16. L, LX, LXX, _______, XC
(a) LXXX (b) C (c) D (d) M

17. Put the sign between these: (2017)


XL _______ XXX
(a) > (b) < (c) = (d) ≥
Directions (Qs. 18 to 21) : Choose the Roman representation for: (Mental Mathematics)
18. 180
(a) CVII (b) CIX (c) CXX (d) CLXXX
19. 1000
(a) L (b) C (c) D (d) M
20. 85
(a) LXXXV (b) DXXV (c) LXXV (d) DIXV
21. 600
(a) DLXX (b) DC (c) DXL (d) DCXX
22. Which of the following is the greatest? (2008)
(a) L (b) LX (c) C (d) XXX
23. Pick the odd one out.
(a) L (b) XIV (c) XXXX (d) XVI
24. Pick the odd one out.

(a)
XIX (b)
XXI (c)
XXIV (d)
XXV

25. When there is bar over any roman numeral, the value of the number increase _____
times.
(a) 500 (b) 50 (c) 10 (d) 1000

26. X means _____.


(a) 1100 (b) 10000 (c) 13000 (d) 14000

Directions (Qs. 27 and 28) : Find the missing digit.


27. XV- _____ = X (2008)
(a) V (b) L (c) VI (d) IX
28. X + XXX = _____
(a) IV (b) L (c) XL (d) M
Roman Numerals 7
29. Which of the following is the greatest? (2015)
(a) XLIII + XLIV (b) LXXIX – XXXIX
(c) XCIX – LXVIII (d) LVII + XL
Directions (Qs. 30 to 33) : Choose the correct option. (Mental Mathematics)
30. X + XX = _______.
(a) XVIII (b) LXV (c) XXX (d) LXX
31. XX + L = _______.
(a) XVIII (b) LXV (c) XXX (d) LXX
32. L + X + V = _______.
(a) XVIII (b) LXV (c) XXX (d) LXX
33. XIII + V = _______.
(a) XVIII (b) LXV (c) XXX (d) LXX
34. Which of the following is meaningless? (Critical Thinking)
(a) XIII (b) XIX (c) XVV (d) XL
35. Which is a meaningless Roman numeral? (2012)
(a) XXXIX (b) LXXXIX (c) XXIX (d) XXXXIX
36. Which is a meaningful Roman numeral? [Tricky]
(a) VL (b) IVC (c) XC (d) LIL
37. Number 15 can be written as :
(a) VVV (b) VIV (c) XIV (d) XV
38. How will the number 24 be written in Roman numeral system?
(a) XXIIII (b) IVXX (c) XXVI (d) XXIV
39. I had 8 paper sheets and my brother had 10 paper sheets. So together we both
have _______ sheets. (2008)
(a) XVIII (b) XVI (c) IX (d) XIX
40. I have C candies and went to divide them between L children. How many candies
will each child get?
(a) III (b) II (c) X (d) V
41. There are 50 students in a class and Rohan wants to give 2 pencils to each student.
How many should he buy?
(a) L (b) C (c) D (d) M
42. I went to market with ` C. I bought fruits for ` IX, a pencil of ` V and some
grocery of ` 50. I am left with:
(a) VI (b) L (c) LX (d) XXXVI
EBD_7364
8 Olympiad Champs–Mathematics
43. Reema’s father is 69 years old. 69 can be written as (2010)
(a) LXIX (b) C (c) XC (d) LV
44. Romans did not have a symbol for:
(a) 0 (b) 10 (c) 100 (d) 1000
45. Repetition of a Roman numeral means
(a) multiplication (b) addition
(c) squaring (d) twice the number
46. Roman numerals can be repeated ________.
(a) once (b) twice
(c) thrice (d) n number of times
47. XL represents a number. If place of the symbols is interchanged, the number will
increased by: (2010)
(a) 1 (b) 5 (c) 10 (d) 20
48. Roman numerals which cannot be repeated are
(a) I, X (b) C, M (c) V, L (d) none

LEVEL 2
49. The Indo-Arabic numeral for LXXV is (2013)
(a) 750 (b) 75 (c) 125 (d) 525
50. Roman numerals which are used for both addition and subtraction are
(a) I, V, X (b) V, X, L (c) L, C, D (d) I, X, C
51. Match the following :

List I List II
A. 800 1. 16
B. LX 2. DCCC
C. XVI 3. LXXX
D. 80 4. 60

A B C D
(a) 1 2 3 4
(b) 2 3 4 1
(c) 2 4 1 3
(d) 3 2 4 1
Roman Numerals 9
52. What will be the outcome for the given diagram? (Critical Thinking)

+

IV

+
– =
I II ?

(a) XIX (b) XII (c) XIII (d) XV


53. DXLV means _____.
(a) 500 (b) 540 (c) 545 (d) 600
54. Statement A: C = 40. (Critical Thinking)
Statement B: M > 100.
(a) Statement A is true. (b) Statement B is false.
(c) Both statements are true. (d) Only statement B is true.
55. Roman numerals that can be repeated in Roman system are (2015)
(a) I, X and C (b) I, V and X (c) V, L and D (d) None of these
56. Statement A : In Roman numerals a letter can be used more than three times
continuously.
Statement B : A bar or a line on roman number increases its value by 1000
times. (Critical Thinking)
(a) Both are true. (b) Both are false.
(c) Only A is true. (d) Only B is true.
57. Which of the following is true or false? (Tricky)
(A) XXVIII > 20 + 8 (B) 30 + 4 = XXXVI
(C) XXVI = 20 + 6 (D) 30 + 8 < XXXIX

(a)
TFFT (b)
FFTT (c)
TFFF (d)
TTTT
58. Roman numeral for 498 is _______. (2014)
(a) CDCXVIII (b) CDCXIV (c) CDXCVIII (d) None of these
59. Select the correct match of Roman numerals in Column I with Hindu-Arabic numerals
in Column II. (2012)
Column-I Column-II
A. CCCXVIII (i) 318
B. DCCLXIX (ii) 769
C. MMMCCXCIX (iii) 3299
D. DDCCXLVIII (iv) 5748
(a) A-iii, B-i, C-iv, D-ii (b) A-i, B-ii, C-iii, D-iv
(c) A-iii, B-ii, C-i, D-iv (d) A-iii, B-i, C-ii, D-iv
EBD_7364
10 Olympiad Champs–Mathematics
60. A cricket stadium manager counted the number of matches held in each month.

Matches held
Months Number of
matches
August XCIV
September LXXXVI
October XCIX
November CVI
In which month, the stadium had the lowest number of matches?
(a) August (b) September (c) October (d) November
61. The year 2015 will be written as:
(a) MMXV (b) MMMV (c) MMIX (d) MMXX
62. Select the INCORRECT match. (2013)
(a) CXLIX – 149 (b) DCCX – 710 (c) XVVII – 108 (d) MCLIV – 1154
63. Select the INCORRECT match. (2013)
(a) LIV = 54 (b) XLVI = 46 (c) LXXIX = 89 (d) XL = 40
64. A movie was released in MCMLV. Choose the numeral number for this roman
number.
(a) 1948 (b) 1965 (c) 1955 (d) 1960
65. Statement A : In Roman numerals, the digits do not have any place value.
Statement B : In MCMXVII year India became independent. Which of the statement
is correct? (Tricky)
(a) Only A is false (b) Only B is false (c) Both are false (d) None of these
66. Statement A : In Roman numerals system, the symbol VC represents the number
95. (2009, Tricky)
Statement B : The ascending order of numbers X, V, VIII, IX is X, IX, V, VIII.
(a) Only B is true (b) Only A is true (c) Both are true (d) None of these
67. Statement A : Roman numerals can be added together to represent different
numbers. (Critical Thinking)
Statement B : The successor of XVIII is XIX. Which of the statement is true?
(a) Only A (b) Only B (c) Both (d) None of these
68. The equivalent of 2134 in Roman numerals is (2010, Tricky)
(a) MMLXXIV (b) MMCXXXIV (c) LMMXXXIV (d) DDDLXXIV
69. The Indo Arabic numeral for CMLXXXII is

(a)
482 (b)
532 (c)
982 (d)
1532
Roman Numerals 11
70. The Roman numeral for 3390 is (2011)
(a) MMMCCCXC (b) MMXCCD
(c) MMMXCD (d) MMMCCXCD
71. The Indo-Arabic numeral for MCDXIV is
(a) 1014 (b) 1404 (d) 1400 (d) 1414
72. IX + XV + XX = _______. (2013)

(a)
45 (b)
35 (c)
44 (d)
76
73. M + CCC + _______ = 1344 (Critical Thinking)
(a) XXXXIV (b) XLIV (c) XLIVX (d) XLHII
74. CCM + ______ + IV = 884 (2008)
(a) LXXL (b) LXXX (c) XXC (d) LXXVV
75. The Roman numeral for 1296 is
(a) MCCXCVI (b) MCCLXXXXVI
(c) MCCCDXCVI (d) DDCCXCVI
76. Select the INCORRECT match. (2017)
(a) 318 - CCCXVIII (b) 523 - DXXIII (c) 694 - DCXIV (d) 940 - CMXL
77. Smallest 3 digit number is subtracted from smallest 4 digit number. Write the
result in Roman numeral.
(a) DM (b) CM (c) DCM (d) MC
78. Garima is XIII years old. Her sister is X years old. How old will they be when their
total age is LV years? (2014)
(a) XXIX, XXVI (b) XXV, XXII (c) XX, XXIII (d) XIX, XXII

RESPONSE GRID
1. a b c d 2. a b c d 3. a b c d 4. a b c d 5. a b c d
6. a b c d 7. a b c d 8. a b c d 9. a b c d 10. a b c d
11. a b c d 12. a b c d 13. a b c d 14. a b c d 15. a b c d
16. a b c d 17. a b c d 18. a b c d 19. a b c d 20. a b c d
21. a b c d 22. a b c d 23. a b c d 24. a b c d 25. a b c d
26. a b c d 27. a b c d 28. a b c d 29. a b c d 30. a b c d
31. a b c d 32. a b c d 33. a b c d 34. a b c d 35. a b c d
36. a b c d 37. a b c d 38. a b c d 39. a b c d 40. a b c d
41. a b c d 42. a b c d 43. a b c d 44. a b c d 45. a b c d
46. a b c d 47. a b c d 48. a b c d 49. a b c d 50. a b c d
51. a b c d 52. a b c d 53. a b c d 54. a b c d 55. a b c d
56. a b c d 57. a b c d 58. a b c d 59. a b c d 60. a b c d
61. a b c d 62. a b c d 63. a b c d 64. a b c d 65. a b c d
66. a b c d 67. a b c d 68. a b c d 69. a b c d 70. a b c d
71. a b c d 72. a b c d 73. a b c d 74. a b c d 75. a b c d
76. a b c d 77. a b c d 78. a b c d
EBD_7364
12 Olympiad Champs–Mathematics

Solutions with Explanation


LEVEL 1

1. (b) CXI = 100 + 10 + 1 = 111


2. (d) XCIV = 100 – 10 + 5 – 1 = 90 + 4 = 94
3. (a) XXIII = 10 + 10 + 1 + 1 + 1 = 23
4. (b) 5. (a) 6. (c) 7. (c)
8. (a) CDXC = 400 + 90 = 490
9. (b) We have 32 teeth.
10. (c) [50 + 20 + 10 + 6 = 86]
11. (c) 12. (a)
13. (c) L = 50, C = 100, D = 500, M = 1000
14. (b) There is difference of one.
15. (b) There is difference of five.
16. (a) There is difference of ten.
17. (a) [40 is greater than 30]
18. (d) 19. (d) 20. (a) 21. (b)
22. (c) [L = 50, LX = 60, C = 100, XXX = 30]
23. (c) It cannot be the representation of any number.
24. (c) The series is with the difference of 2, so it will be 19, 21, 23 and 25.

25. (d) 26. (b)


27. (a) [15-5=10]
28. (c) [10 + 30 = 40]
29. (d) 57 + 40 = 97 is greatest
30. (c) 10 + 20 = 30
31. (d) 20 + 50 = 70
32. (b) 50 + 10 + 5 = 65
33. (a) 13 + 5 = 18
34. (c)
35. (d) XXXXIX is meaningless as no number can be repeated more than 3 times.
36. (c) XC is a meaningful Roman numeral
V, L, D are never subtracted thus option a, b, d are meaningless.
37. (d) 38. (d)
39. (a) [10 + 8 = 18]
Roman Numerals 13
40. (b) [C = 100, L = 50, 100/50 = 2]
41. (b) [50 x 2 = 100 = C pencils are required]
42. (d) [100 - (9 + 5 + 50) = 100 - 64 = 36]
43. (a)
44. (a) In Roman system, there is no symbol for zero. Thus, they did not use place value
system.
45. (b) Repetition of Roman numerals means addition but numerals like V, L and D cannot
be repeated.
46. (c) Roman numerals can be repeated thrice.
47. (d)
48. (c) V, L and D cannot be repeated.

LEVEL 2

49. (b) (L = 50) + (XX = 20) + (V = 5)


⇒ 50 + 20 + 5 = 75
50. (d) Roman numerals I, X, C are used for both addition and subtraction.
51. (c)
52. (c) [10 + 4 + 1 = 15 - 2 = 13]
53. (c) [D = 500, XL = 40, V = 5 so 500 + 40 + 5 = 545]
54. (d) C = 100 and M = 1000 > 100
55. (a) As 1, X and C can be repeated
56. (d)
57. (b)
58. (c) CDXCVIII = 498
59. (b) A - i, B - ii, C - iii, D - iv
60. (b) September
61. (a) [M = 1000, X = 10, V = 5 → 1000 + 1000 + 10 + 5 = 2015]
62. (c) XVVII – 108 is the incorrect match
63. (c) LXXIX = 89 is the incorrect match
64. (c) [1000 + 900 + 50 + 5 = 1955]
65. (b) 66. (d) 67. (c) 68. (b)
69. (c) (CM = 900) + (LXXX = 80) + (II = 2)
⇒ 900 + 80 + 2
CMLXXXII = 982
EBD_7364
14 Olympiad Champs–Mathematics
70. (a) 3390
⇒ 3000 + 300 + 90
(3000 = MMM) + (300 = CCC) + (XC = 90)
MMMCCCXC
71. (d) MCDXIV
(M = 1000) + (CD = 400) + (XIV = 14)
⇒ 1000 + 400 + 14
⇒ 1414 = MCDXIV
72. (c) As 9 + 15 + 20 = 44
73. (b) (M = 1000) + (CCC = 300) + 44 = 1344
⇒ 44 = XLIV
74. (b) CCM = 800
80 = LXXX
IV = 4
CCM + LXXX + IV
800 + 80 + 4 = 884
75. (a) 1296
1000 + 200 + 90 + 6
(1000 = M) + (CC = 200) (XC = 90) + (VI = 6)
MCCXCVI
76. (c) 694 – DCXIV is incorrect match
77. (b)
78. (a) XXIX + XXVI = 55
Number System 15

CHAPTER FOREWORD
A number is a mathematical object used to count, measure and label. Numbers are the
foundation of mathematics. No one can understand mathematics without proper knowledge
of numbers. In your daily life, you can see many examples of use of the numbers, but the most
common is a clock.

12 1
11
10 2

9 3

8 4
7 5
6

Everyone is aware about it. In the above figure, the objects which are used to represent the
time are nothing but numbers.
To represent any number, we use ten different numerals. These are :
0, 1, 2, 3, 4, 5, 6, 7, 8, 9
All numbers can be written by using the combination of these numerals. For example, in
the given figure of a clock 10, 11 and 12 are the combination of 1 and 0, 1 and 1, 1 and 2
respectively.
Now answer the following questions
1. What is the difference between 1,00, 000 and 10,00, 000?
2. How can we write numbers greater than 10, 000, 00, 000?

After reading this chapter, you will learn about the role of place value in numbers. You will
also know about factors, multiples and LCM.

Note : Natural number start with 1 and continue till infinity but whole number start with 0
and continue till infinity. So, we can't write the largest natural number and largest
whole number.
EBD_7364
16 Olympiad Champs–Mathematics

Chapter
2 Number System

Amazing
Amazing Facts
Facts
LEARNING OBJECTIVES
v Zero was not even considered a
number by the Ancient Greeks. This lesson will help you to:
However, they also questioned
whether 1 was a number. v learn and study about Indian and International place
v The Mayans discovered/ value system.
developed zero. v learn to find place value of numbers beyond 1000.
v 2 and 5 are the only prime
numbers that end with 2 or 5. v study and learn the role of place value in addition,
v Different names for the subtraction and multiplication algorithms.
number 0 include zero, nought,
naught, nil, zilch and zip. v understand and study about informal and standard
v The name of the popular search division algorithm.
engine ‘Google’ came from a
misspelling of the word ‘googol’, QUICK CONCEPT REVIEW
which is a very large number
(the number one followed by Place Value System
one hundred zeros to be exact).
Place Value System

Indian Place International Place


Value System Value System

Indian Place Value System

Crores Lakhs Thousands Ones     ← Periods


Ten Crores Ten Lakhs Ten Thousands Hundreds Tens Ones
Crores C Lakhs L Thousands Th H T O ← Places
TC TL T Th
10,00,00,000 1,00,00,000 10,00,000 1,00,000 10,000 1,000 100 10 1

While writing large numbers having six or more digits, the numbers are broken up into Periods
with the help of Commas.
Dividing into Periods
Tc C TL L TTh Th H T O

Crores Lakhs Thousands Ones


Period Period Period
Period
Number System 17
For example: Number 2,75,96,415 is read as two crore
seventy five lakh ninety six thousand four hundred
fifteen.

INTERNATIONAL SYSTEM OF NUMERATION Important


International system of numeration follows the following The place value of 0 does not
periods—ones, thousands and millions. This system is depend upon the place which comes
mostly used all over the world. on the number.
International Place Value System

Millions Thousands Ones ← Periods

Hundred Ten Millions Hundred Ten Thousands Hundreds Tens Ones ← Places
Millions Millions Thousands Thousands
(HM) (TM) (M) (HTh) (T Th) (Th) (H) (T) (0)

100,000,000 10,000,000 1,000,000 100,000 10,000 1,000 100 10 1

For easier readability, commas are used to separate each


Do You Know ?
group of three digits, which is called a period. When a
number is written in this form, it is said to be in "standard Indian place value system is used
form. "Example: four hundred sixteen thousand, seven in India as well as in Bangladesh,
hundred thirty-one can be written as 416,731. Nepal, Pakistan and Sri Lanka

COMPARISON BETWEEN INDIAN AND INTERNATIONAL PLACE VALUE SYSTEM


Table chart of both numeration systems (National and International) is given below.

100000000 10000000 1000000 100000 10000 1000 100 10 1

Indian Ten Ten


Ten crores Crores Lakhs Thousands Hundreds Tens Ones
System Lakhs Thousands

International Hundred Ten Hundred Ten


Millions Thousands Hundreds Tens Ones
System Millions Millions Thousands Thousands

FORMING THE GREATEST AND THE SMALLEST


Remember
NUMBERS
Always put '0' in second place from
Greatest Number left, while forming the smallest
To form the greatest number from the given digits, number.
start from extreme left and arrange the digits in their
descending order.
Smallest Number Do You Know ?
To form the smallest number from the given digits, start
from extreme left and arrange the digits in ascending Successor means after and
order. predecessor means before.
EBD_7364
18 Olympiad Champs–Mathematics
The role of place value in addition, subtraction and
multiplication algorithms.
v The place value of a number starts from right to
Historical
Amazing Preview
Facts
left in the following order: ones, tens, hundreds,
thousands, ten thousands, hundred thousands, etc.
v The Egyptians had a base 10
system of hieroglyphs for v Place values are extremely important when doing
numerals. There was no symbol addition, subtraction and multiplication.
for zero. They had seven v When doing addition or subtraction, add or subtract
separate symbols (hieroglyphs) like places, and you may need to group in addition
for one unit, one ten, one and ungroup in subtraction to get enough to subtract
hundred, one thousand, one from.
ten thousand, one hundred Example: Add 65,000 and 1500
thousand, and one million as
65,000
shown below:
+ 1,500
1 = staff
66,500

10 = heel bone Example: Subtract 7,400 from 74,000.


74,000
100 = coil of rope – 7,400
66,600
1000 = lotus flower
Role of place value in multiplication algorithms
v The number to be multiplied is the multiplicand.
10,000 = pointing finger v The number we are multiplying with is the multiplier.
v Multiplication is repeated addition. Adding multiplicand
100,000 = tadpole
by multiplier times gives the product.
1,000,000 = astonished man Lets us understand the concept of place value in
multiplication, with the help of an example.
v Indians were the first to
Example: Multiply the following numbers: 263 and 64
develop a base ten system.
They developed methods of STEP 1: Multiply the multiplicand by ones digit of the
expressing every possible multiplier.
number using a set of ten
1. Multiply the number in the ones place of the
symbols very similar to the
decimal system we use today multiplicand with the number in the ones place of
with symbols close to the ones the multiplier. (263 x 64)
we use today. Zero was used to
3 x 4 = 12 (1 tens, 2 ones).
denote an empty space.
Put the 2 in ones column and carry over 1 to the tens
column.
2. Multiply the number in the tens place of the
multiplicand with the number in the ones place of
Brahmi numerals around 1st the multiplier. (263 x 64)
century A.D.
6 x 4 = 24 + 1 (carry over from step 1)
= 25 (2 hundreds,5 tens).
Put the 5 in tens column and carry over 2 to the
hundreds column.
Number System 19
3. Multiply the number in the hundreds place of the
multiplicand with the number in the ones place of
the multiplier. (263 x 64)
2 x 4 = 8 + 2 (carry over from step 2)
= 10 (1 thousands,0 hundreds).
Put the 0 in hundreds column and 1 (carry over) in
thousands column.
2 6 3
× 6 4
1 2 1
1 0 5 2

STEP 2: Multiply the multiplicand by tens digit of the


multiplier
1. Multiply the number in the ones place of the
multiplicand with the number in the tens place of
the multiplier. (263 x 64)
3 x 6 = 18 (1 hundreds , 8 tens).
Put the 8 in tens column and carry over 1 to the
hundreds column.
2. Multiply the number in the tens place of the
multiplicand with the number in the tens place of
the multiplier. (263 x 64)
6 x 6 = 36 + 1 (carry over from step 1)
= 37 (3 thousands,7 hundreds).
Put the 7 in hundreds column and carry over 3 to the
thousands column.
3. Multiply the number in the hundreds place of the
multiplicand with the number in the tens place of the
multiplier. (263 x 64)
2 x 6 = 12 + 3 (carry over from step 2)
= 15 (1 ten thousands,5 thousands).
Put the 5 in thousands column and 1 (carry over) in
ten thousands column.
2 6 3
× 6 4
1 0 5 2
1 5 7 8
1 6 8 3 2
EBD_7364
20 Olympiad Champs–Mathematics
STEP 3: Add result of Multiplier 1's and Multiplier 10's
results and put the result in the Product columns.
1052 + 15780 = 16832
Product of 263 and 64 is 16832

To study about informal and standard division algorithm


v Division is equal distribution of a given quantity.
v The number to be divided is the dividend.
v The number which divides is called divisor.
v The answer is called the quotient.
v The number left after division is called the
remainder.

Standard Division Algorithms


Unlike addition, subtraction and multiplication, division is
performed from left to right.
(Highest place value to lowest place value).
Number System 21

Multiple Choice Questions


LEVEL 1
1. Which is the correct word form of 9.04? (Mental Mathematics)
(a) Nine and four tenths (b) Ninety and four hundredths
(c) Nine and four hundredths (d) Nine and four thousandths
2. What is the value of the 5 in the following numbers? 132,070,689,050
(Mental Mathematics)
(a) ten (b) ten thousand (c) ten million (d) ten billion
3. Pick the odd one out keeping in mind the divisibility rules of 3. (Mental Mathematics)
(a) 252 (b) 367 (c) 183 (d) 492
4. 12 : Even number :: ___ : Odd number. (Mental Mathematics)
(a) 14 (b) 17 (c) 16 (d) 10
5. 11 : Prime Number :: ____ : Composite number. (Mental Mathematics)
(a) 12 (b) 13 (c) 17 (d) 7
6. This is the value where the digit is in the number, such as units, tens, hundreds,
etc. (Mental Mathematics)
(a) Time value (b) Place value
(c) Difference value (d) Product value
Directions (Qs. 7 to 10) : In the given questions, identify the place value of 5.
(Mental Mathematics)
7. 17,526,010
(a) five ten thousands (b) five thousands
(c) five hundred thousands (d) five hundreds
8. 2,110,735,000
(a) five thousands (b) five hundreds
(c) five millions (d) five ten thousands
9. 780,756
(a) five ones (b) five tens (c) five hundreds (d) 5 tenth
10. 50,697
(a) five hundreds (b) five millions
(c) five hundred thousands (d) five ten thousands
EBD_7364
22 Olympiad Champs–Mathematics
11. Complete the number sentence.
327,421 = 300,000 + 20,000 + ? + 400 + 20 + 1
(a) 70,000 (b) 700,000 (c) 7,000 (d) 700
12. Which shows five hundred six million, seventy-three thousand, eight in standard
form?
(a) 516,073,008 (b) 506,073,008 (c) 506,111,0008 (d) 506,068,908
13. How do you write 10.067 in expanded notation?
(a) 10 + 6/100 + 7/1,000 (b) 1 + 6/10 + 7/100
(c) 100 + 60 + 7 (d) 1 + 6/100 + 7/1,000
14. The expanded form of 6,153,122 is: (2010)
(a) 6,000,000 + 100,000 + 50,000 + 3000 + 100 + 20+2
(b) 60,000,000 + 100,000 + 50,000 + 3000 + 100 + 20 + 2
(c) 600,000,000 + 100,000 + 50,000 + 3000 + 100 + 20 + 2
(d) 6,000,000,000 + 100,000 + 50,000 + 3000 + 100 + 20 + 2
15. 17422.16 in word form is
(a) seventeen thousand, four hundred, two, two and one tenths and six hundredths
(b) seventeen, four hundreds, twenty-two and sixteen hundredths
(c) seventeen thousand, four hundred, twenty-two and sixteen hundredths
(d) seventeen thousand, four hundred, twenty-two and sixteen tenths
16. Sara's user ID is a 5-digit number. The 9 is in the ten thousands place. The 0 is
in the ones place. An 8 is in the thousands place. A 4 is in the tens place. A 2 is
in the hundreds place. What is Sara's user ID number? (Critical Thinking)
(a)
90,842 (b)
89,204 (c)
98,420 (d)
98,240
17. Which of the following expressions does not describe the value of the digit 5 in the
number 21.3572? (Tricky)
(a) 500 ten-thousandths (b) 50 tenths
(c) 50 thousandths (d) 5 hundredths
18. John spends 2.63 hours studying for Math, 6.37 hours studying for English and
0.4 hours studying for reading. How much total time does John spend studying?
 (Critical Thinking)

(a) nine and four thousandth hours (b) nine and four tenths hours
(c) nine and four hundredth hours (d) nine hours
19. How do you write 240,004,395 in expanded notation?
(a) 200,000,000 + 40,000,000 + 4,000,000 + 300 + 90 + 5
(b) 200,000,000 + 40,000,000 + 4,000 + 300 + 90 + 5
(c) 200,000,000 + 40,000,000 + 4,000,000 + 300,000 + 90,000 + 5,000
(d) 200,000,000 + 40,000,000 + 300 + 90 + 5
Number System 23
20. What is the value of 2 in the number: 529,307,604,000 ?
(a) Hundred billion (b) Ten billion (c) Ten million (d) One million
21. Choose an answer that has a digit in the hundreds place that has a greater value
than the digit in the thousands place.
(a) 101,100 (b) 428,304 (c) 580,340 (d) 873,212
22. How many four digit numbers are there between 999 and 3000? (2008, Tricky)
(a) 2001 (b) 2000 (c) 1999 (d) 1998
23. Estimate to the nearest hundred 496 plus 318.
(a) 814 (b) 800 (c) 178 (d) 700
24. Write the following four numbers in descending order :
I. 4 2 0 3 5 6 7 II. 4203657
III. 4 2 0 3 7 5 6 IV. 4 2 0 3 6 7 5
(a) I, II, III, IV (b) III, IV, II, I
(c) I, II, IV, III (d) III, II, IV, I
25. The number 5 crores 9 lakhs 4 thousands 9 hundred eighty-eight in numerals can
be written as
(a) 59004988 (b) 590400988 (c) 509049088 (d) 50904988
26. Sum of a number of two digits and the number obtained by reversing the digits of
the first number is 110. If the difference of the digits is 4, then the number is

(a)
62 (b)
73 (c)
84 (d)
51
27. Which of the following is correct? (2008, Tricky)
(a) Successor of predecessor of 1000 is 1001
(b) Successor of predecessor of 1000 is 1002
(c) Predecessor of successor of 1000 is 1000
(d) Predecessor of predecessor of 1000 is 999
28. Sum of place values of 6 in 63606 is (Critical Thinking)
(a) 6066 (b) 18 (c) 60606 (d) 6606
29. The difference of 5671 and the number obtained on reversing its digits is

(a)
7436 (b)
3906 (c)
4906 (d)
3916
30. In the number 3.4625, the place value of the digit 2 is
(a) 1000 (b) 100 (c) 1/1000 (d) 1/100
31. 407928 is read as (2012)
(a) Forty thousand nine hundred twenty eight
(b) Four lakh seven thousand nine hundred twenty eight
(c) Four lakh seventy nine thousand twenty eight
(d) Forty seven thousand nine hundred twenty eight
EBD_7364
24 Olympiad Champs–Mathematics
LEVEL 2
32. In the given number 890436,if you write 0 in place of 4, by how much the resulting
number be less than this given number? (Tricky)
(a) 40 (b) 400 (c) 436 (d) 36
33. Match the following numbers in list I with the corresponding place value of
number 1.

List I List II
A. 761,364 1. One hundred thousand
B. 13,486.3 2. One thousand
C. 1,234,567 3. One ten thousand
D. 3,143,064 4. One million
A B C D
(a) 2 3 4 1
(b) 3 2 1 4
(c) 4 2 3 1
(d) 1 3 2 4
34. Read the statement and choose the correct option.
Statement A : As per the place value system, multiplication is performed from left
to right i.e. Highest place value to lowest place value).
Statement B : As per the place value system, division is performed from left to right
i.e. Highest place value to lowest place value).
(a) Only A is true. (b) Only B is true.
(c) Both A and B are true. (d) Both A and B are false.
35. Ten thousands + ten ones + ten tens equals : (2010)

(a)
10110 (b)
11010 (c)
10011 (d)
101010
36. In number 97580, when the digits 7 and 5 as interchanged its place, then the
difference between the original and the new number is (Critical Thinking)

(a)
1800 (b)
1080 (c)
1008 (d)
1000
37. What is 650.287 rounded to the nearest tenth? (2010)
(a) 6050 (b) 6100 (c) 6050.29 (d) 6050.3'
38. What number am I?
• I am a two-digit even number.
• I am a common multiple of 6 and 7.
• I have a total of 8 factors.  (2010)

(a)
35 (b)
42 (c)
36 (d)
84
39. What is the numeric form of the number given below? (2011)
“One hundred four thousand, one hundred three”
(a) 1,413 (b) 14,103 (c) 104,103 (d) 104,113
Number System 25
40. Which list shows all the prime numbers between 0 and 22? (2011)
(a) 1, 3, 5, 7, 11, 13, 19 (b) 2, 3, 5, 7, 11, 13, 17, 19
(c) 2, 4, 6, 8, 10, 12, 14, 16, 18, 20, 21 (d) 1, 2, 4, 6, 8, 9, 10, 12, 14, 15, 16, 18, 20, 21
41 . Find the greatest number which divides 36 and 84 leaving no remainder in each
case. (2012)

(a)
6 (b)
9 (c)
12 (d)
18
42. Hundred million = _____________. (2012)
(a) 10 crore (b) 1 crore (c) 100 crore (d) 10 lakh
43. I am an even number. You will find me if you count in 7s. I am less than 100. I am
more than 80. Who am I? (2013)

(a)
72 (b)
84 (c)
96 (d)
82
44. Place value and face value of a number is always equal at ________. (2014)
(a) One’s place (b) Ten’s place
(c) Hundred’s place (d) None of these
45. 1 billion = ______ crores. (2015)
(a) 1 (b) 10 (c) 100 (d) 1000
46. Sum of divisors of 78 is _______. (2012)

(a)
168 (b)
170 (c)
167 (d)
189
47. 500 thousand + 600 hundred + 320 tens = ____________. (2016)
(a) 563200 (b) 5632000 (c) 5063200 (d) 5603200
48. Number of thousandths in 2 tenths is ____________. (2016)
(a) 20 (b) 200 (c) 2 (d) None of these
49. The smallest ten digit number that has two digits the same is ____________.
(2015)
(a) 1000000001 (b) 11000000000 (c) 1010000000 (d) 1000000010
50. 15 hundreds multiplied by fifteen tenths multiplied by three fifteenths divided by
forty five thousandths is equal to the successor of ____________.
 (2014, Critical Thinking)
(a) 999 (b) 9999 (c) 99999 (d) 14999
51. The sum of two numbers is 484. One of the numbers ends in a zero. If this zero
is removed, we get the second number. The numbers are _______ and _______.
 (2017, Tricky)
(a) 40, 444 (b) 4, 480 (c) 44, 440 (d) 84, 400
52. Using all the given digits, form the greatest possible 7-digit even number, if
repetition of digits is allowed.  (2016)
9 3 0 8
(a)
9830000 (b)
9998830 (c)
9988330 (d)
9999830
EBD_7364
26 Olympiad Champs–Mathematics
53. 
What is the difference in the place value of the digits 8 and 6 in the numeral
8962321?  (2016)
(a) 7994000 (b) 7904000 (c) 7960000 (d)
7940000
54. Match the columns.  (2015)
Column I Column II
(i) 1524 (p) MMMDL
(ii) 3678 (q) MDXXIV
(iii) 2540 (r) MMMDCLXXVIII
(iv) 3550 (s) MMDXL
(a) (i)  (q), (ii)  (r), (iii)  (s), (iv)  (p)
(b) (i)  (p), (ii)  (s), (iii)  (r), (iv)  (q)
(c) (i)  (p), (ii)  (r), (iii)  (q), (iv)  (s)
(d) (i)  (q), (ii)  (p), (iii)  (s), (iv)  (r)
55. ________ is equivalent of one lakh in International System of number. (2014)
(a) Hundred thousand (b) Ten million
(c) One million (d) Ten thousand
56. Which of the following statements in INCORRECT? (2014)
(a) The place value of the digit '2' in the number 532689 is 200 times the number 10.
(b) The difference in place values of the digits '7' and '4' in the number 6734581 is
696000.
(c) In the number 8176942, the place value of the digit '1' is 50000 × 20.
(d) The place value of the digit '5' in the number 7538612 is 21869 more than 478131.
57. Which of the following options hold? (2013)
Statement 1 : The value of CM – CD + CVII is 607.
Statement 2 : Symbol C can be added to every Roman Symbol. But it can be subtracted
from D and M only.
(a) Statement 1 is true but statement 2 is false.
(b) Statement 1 is false but statement 2 is true.
(c) Both the statements are true.
(d) Both the statements are false.
58. Taking a number from 32 thousands gives 14 hundreds and 26 tens. The number
is ____. (2013)
(a) 1660 (b) 30340 (c) 32000 (d) 33660
59. Gautam is thinking of a four digit number. (2014, Tricky)
• The fourth digit is thrice the first digit.
• The second digit is the second multiple of 2.
• The third digit is the smallest even number.
Find the number.
(a) 3426 (b) 6421 (c) 9423 (d) 3429
Number System 27
60. Select the correct match. (2014)
(a) 897 - DCCXCVII (b) 1234 - MCCXXIV
(c) 541 - CDXLI (d) 547 - DXLVII
61. Which one of the following statements is true? (Critical Thinking)
(a) All the even numbers are composite numbers.
(b) All the odd numbers are prime numbers.
(c) These are infinitely prime numbers.
(d) A prime number can be written as the product of more than two natural number.

LEVEL- 1
1. a b c d 2. a b c d 3. a b c d 4. a b c d 5. a b c d
6. a b c d 7. a b c d 8. a b c d 9. a b c d 10. a b c d
11. a b c d 12. a b c d 13. a b c d 14. a b c d 15. a b c d
16. a b c d 17. a b c d 18. a b c d 19. a b c d 20. a b c d
21. a b c d 22. a b c d 23. a b c d 24. a b c d 25. a b c d
26. a b c d 27. a b c d 28. a b c d 29. a b c d 30. a b c d
31. a b c d 32. a b c d 33. a b c d 34. a b c d 35. a b c d
36. a b c d 37. a b c d 38. a b c d 39. a b c d 40. a b c d
41. a b c d 42. a b c d 43. a b c d 44. a b c d 45. a b c d
46. a b c d 47. a b c d 48. a b c d 49. a b c d 50. a b c d
51. a b c d 52. a b c d 53. a b c d 54. a b c d 55. a b c d
56. a b c d 57. a b c d 58. a b c d 59. a b c d 60. a b c d
61. a b c d
EBD_7364
28 Olympiad Champs–Mathematics

Solutions with Explanation


LEVEL 1
1. (c) 9.04 = Nine and four hundredths.
2. (a) The place value of 5 in 132,070,689,050 is tens.
3. (b) 367 is the odd one out. According to the divisibility rules of 3, a number is divisible
by 3 if its sum is divisible by 3. Here 3 + 6 + 7 = 16, this is not divisible by 3.
4. (b) 12: Even number::17:Odd number.
5. (a) 11:Prime Number::12:Composite number.
6. (b) Place Value is the value where the digit is in the number, such as units, tens,
hundreds, etc.
7. (c) In 17,526,010 the place value of 5 is five hundred thousands.
8. (a) In 2,110,735,000 the place value of 5 is five thousands.
9. (b) In 780,756 the place value of 5 is five tens.
10. (d) In 50,697 the place value of 5 is five ten thousands.
11. (c) 327,421 = 300,000 + 20,000 + 7000 + 400 + 20 +1
12. (b) 506,073,008 = five hundred six million, seventy-three thousand, eight.
13. (a) 10 + 6 + 7 = 10.067
100 1,000
14. (a) 6153122 = 6,000,000 + 100,000 + 50,000 + 3000 + 100 + 20 + 2
15. (c) 17422.16 in word form is seventeen thousand, four hundred, twenty-two and
sixteen hundredths.
16. (d) Sara's user ID number is 98,240.
17. (b) Since 5 is present on the right side of the decimal, therefore 50 tenths does not
tell the place value of 5 in 21.3572.
18. (b) Time spent by John studying = 2.63 + 6.37 + 0.4 = 9.4 = nine and four tenths hours.
19. (b) 200,000,000 + 40,000,000 + 4,000 + 300 + 90 + 5 = 240,004,395
20. (b) The value of the 2 in 529,307,604,000 is ten billion.
21. (c) A digit in the hundreds place that has a greater value than the digit in the
thousands place can be seen only in the third option wherein number at hundreds
place (3) is greater than number at thousands place (0).
22. (c) Required number = 2999 – 1000 = 1999.
23. (b) 496 + 318 = 814
Nearest hundred = 800.
24. (b) 4203756 > 4203675 > 4203657 > 4203567
25. (d) 50000000 + 900000 + 4000 + 900 + 80 +8
= 50904988
26. (b) Let the two digit number be 10x + y
Number System 29
Reversing the digit, number become 10y + x.
sum = 10x + y + 10y + x
⇒ 11x + 11y = 110
⇒ x + y = 10 ...(1)
x–y=4 (given) ...(2)
From eqs. (1) & (2)
2x = 14 ⇒ x = 7
∴ y = 3
Hence the number is 73.
27. (c) Successor of 1000 = 1000 + 1 = 1001 then predecessor of 1001 = 1001 – 1 = 1000.
28. (c) Place values of all 6 in
63606
60000
600
6
Sum = 60000 + 600 + 6 = 60606.
29. (b) Given number = 5671
After reversing its digits = 1765.
∴ Difference = 5671 – 1765 = 3906.
30. (c) 3.4625
1000th place
1
∴ Place value of 2 in 3.4625 is .
1000
31. (b)

LEVEL 2
32. (b) Original number = 890436
New number = 890036
Then the resulting number is 400 less than by given number.
33. (a)
34. (b) As per the place value system, division is performed from left to right i.e. Highest
place value to lowest place value). Whereas multiplication is performed from right
to left. So, only B is true.
35. (a) Ten thousands + ten ones + ten tens
= 10 × 1000 + 10 × 1 + 10 × 10
= 10000 + 10 + 100 = 10110
36. (a) Original number = 97580
New number = 95780
Required difference = 97580 – 95780 = 1800
37. (d) 6050.3
38. (b) As 42 is two digit even number having 8 factors (1, 42, 2, 21, 3, 14, 6, 7)
EBD_7364
30 Olympiad Champs–Mathematics
39. (c) 104, 103
40. (b) 2, 3, 5, 7, 11, 13, 17, 19 are prime number between 0 and 22
41. (c) 12 is the greatest number that divides 36 and 84 leaving no remainder each case.
42. (a) Hundred million = 10 crore
43. (b) 84 is the required number
44. (a) One's place
45. (c) 1 billion = 100 crores
46. (a) Divisors of 78 are = 1, 2, 3, 6, 13, 26,39, 78
Sum of divisors of 78 is 1 + 78 + 2 + 39+ 3 + 26+ 6 + 13 = 168
47. (a) 500 × 1000 + 600 × 100 + 320 × 10
= 500000 + 60000 + 3200
= 563200
2 2 ×100 200
48. (b) 2 tenths = = =
10 10 ×100 1000
So, these are 200 thousandths in 2 tenths.
49. (a) The smallest ten digit number that has two digits the same is 1000000001.
50. (b) 15 3 45
15 ×100 × × ÷
10 15 1000
15 3 1000
= 15 ×100 × × ×
10 15 45
= 10000
10000 is successor of 9999.
51. (c)
Since, one of the number ends in a zero
Let it be of the form a × 10
And second number is obtained when zero from a × 10 is removed.
Then second number is of the form = a
According to question sum the two numbers is 484
a + 10a = 484
11 a = 484
a = 44
a = 44
Then, a × 10 = 44 × 10 = 440
Therefore the numbers are 44 and 440.
52. (d) 9999830
53. (d) Place value of 8 in the number = 8000000
Place value of 6 in the numbber = 60000
Now, the difference of place values of the digit 8 and 6 = 8000000 – 60000
= 7940000
54. (a) (i)  q, (ii)  r, (iii)  s, (iv)  p
55. (a) Hundred Thousand is equivalent to one lakh.
56. (c)
57. (c) Bath statements are true CM – CD + CVII = 900 – 400 + 107 = 1007 – 400 = 607
58. (d) The required number is 32000 + 1660 = 33660
59. (d) 3429
60. (d) D X L VII = 500 + 40 + 7
= 547
61. (c)
Operations on Numbers 31

CHAPTER FOREWORD
In this chapter, you will learn what are operations and how they are performed on numbers.
You will learn four types of operations namely:

Addition
(+)

Operations on
Division Multiplication
(÷) Numbers
(×)

Subtraction
(–)

All these operations are used in our daily life.


• Suppose, you have 5 pencils, and you bought 7 more pencils. How many pencils do you
have now?
• You have 6 pens. But you lost 2 of them. How many pens do you have now?
• You are buying 8 books. The cost of each book is ` 20. What will be the total cost of
books?
• You bought 5 litres of kerosene. The total amount you paid was ` 100. What is the price
of kerosene per litre?
Alongwith the above mentioned situations, these operations are also used in calculation
of speed of vehicles and aircrafts, calculation of time to reach any point, calculation of
price of bulk materials or goods. etc.
After reading this chapter, you will understand of various mathematical operations that help
us to solve our daily life problems.

Remember that 7 + 7 + 7 is same as 7×3.


So, multiplication is repeated addition.
Also 32 – 8 – 8 – 8 – 8 = 0. So 32 ÷ 8 = 4
So division is same as repeated subtraction
EBD_7364
32 Olympiad Champs–Mathematics

Chapter
3 Operations on
Numbers
LEARNING OBJECTIVES
Real-Life Examples This lesson will help you to:—

v Addition and subtraction are v study and understand about the operation of numbers
used in the calculations of including additions, subtraction, multiplication and
money. division.
v We use division when we have
v learn and understand about the importance of place
to divide something equally.
Example : Pizza can be divided value in performing operations of numbers.
into 8 pieces so that all the four v learn to use the operations in order.
friends can eat two slices of
pizza each.
QUICK CONCEPT REVIEW
What are operations?

An operation is an action or procedure which produces a


new value from one or more input values, called “operands”.

Operations such as addition, subtraction, multiplication


and division are binary operations since they involve two
or more values.
Amazing
Amazing Facts
Facts

v Subtraction can also be viewed The Basic Operations


as addition of signed numbers. Symbol Words Used
Extra minus signs simply denote
additive inversion. Then we + Addition, Add, Sum, Plus, Increase, Total
have Subtraction, Subtract, Minus, Less, Difference,
-
3 – (– 2) = 3 + 2 = 5. Decrease, Take Away, Deduct
v The study of numbers and Multiplication, Multiply, Product, By, Times,
×
its operations is called as Lots of
algorism. ÷ Division, Divide, Quotient, How Many Times
Operations on Numbers 33
ADDITION
v Addition is bringing two or more numbers (or things) Try It!
together to make a new total.
Example :- Fill in the blanks
v Other names for Addition are Sum, Plus, Increase,
Total. (a) 50 + 90 = 90 + ___
v And the numbers to be added together are called (b) 60 + ___ = 60
the “Addends” (c) 70 + (80 + 90)
Addition: = (70 + 80 ___) + 90
(d) 75 + 100 = ______
8 + 3 = 11
Solution :-
Addend Addend Sum (a) 50 + 90 = 90 + 50
(b) 60 + 0 = 60
Properties for Addition
(c) 70 + (80 + 90)
1. Commutative property of addition: It states that
changing the order of the addends will not affect = (70 + 80) + 90
the sum. (d) 75 + 100 = 175
a + b = b + a
2. Associative property of addition: It states that
changing the groupings of the addends will not
affect the sum.
a + (b + c ) = (a + b) + c Historical Preview
3. Identity property of addition: It states that when
you add 0 to any number, the sum is the number Amazing Facts
itself. v The earliest written records indicate
the Egyptians and Babylonians
a + 0 = 0 + a = a used all the elementary arithmetic
operations as early as 2000 BC.
SUBTRACTION
v Modern methods for four
Subtraction is taking one number away from another. fundamental operations (addition,
subtraction, multiplication and
8 – 3=5 division) were first devised by
Minuend Subtrahend Difference Brahmagupta of India.

Minuend: The number that is to be subtracted from.


Subtrahend: The number that is to be subtracted. Try It !
Difference: The result of subtracting one number from
Example :- Fill in the blanks
another.
(a) ___ – 90 = 50
MULTIPLICATION (b) 108 – ___ = 90
(c) 65 – 59 = ______
Multiplication in its simplest form is repeated addition.
Solution :-
Multiplication:
(a) 140 – 90 = 50
6 × 3 = 18 (b) 108 – 18 = 90
(c) 65 – 59 = 6
Factor Factor Product
(or Multiplier) (or Multiplicand)
EBD_7364
34 Olympiad Champs–Mathematics
Here we see that 6 + 6 + 6 (three 6s) make 18.
It could also be said that 3 + 3 + 3 + 3 + 3 + 3 (six 3s)
Misconcept/Concept make 18.
But you can also multiply by fractions or decimals, which
Misconcept: Dividing a number by
goes beyond the simple idea of repeated addition.
0 gives 0.
Example : 3/0 = 0. Example: 3.5 × 5 = 17.5
Concept: We know that 0/3 = 0. which is 3.5 lots of 5, or 5 lots of 3.5
Conveniently, we also assume
the same result for 3/0. This is Properties for Multiplication
not true as 3/0 is undefined. 1. Commutative property of multiplication : It states
Misconcept: Zero divided by zero that changing the order of the factors will not
is equal to 1. i.e. 0/0 = 1. affect the product.
Concept: We all are aware that
a × b= b × a
3/3, 5/5 result in 1. As long as
we see fraction in the form of 2. Associative property of multiplication : It states
a/a, we will expect an answer that changing the groupings of the factor will not
of 1. This however does not affect the product.
hold when a = 0 as 0/0 is a × (b × c) = (a × b) × c
undefined.
3. Identity property of multiplication : It states that
when you multiply any number by 1, the result is the
number itself.

Try It!
a × 1 = a
4. Zero property of multiplication : It states that
Example: Fill in the blanks
when you multiply any number by 0, the result
(a) 75 × ___ = 300
is 0.
(b) ___ × 5 = 90

a × 0 = 0
(c) 16 × 5 = ___
(d) 500 ÷ 125 = ___ DIVISION
(e) 600 ÷ ___ = 10
v Division is splitting into equal parts or groups.
(f) ___ ÷ 5 = 40
v We use the ÷ symbol, or sometimes the / symbol to
Solution:
symbolise divide.
(a) 75 × 4 = 300
12 ÷ 3 = 4
(b) 18 × 5 = 90
12 / 3 = 4
(c) 16 × 5 = 80
(d) 500 ÷ 125 = 4 v Division is the opposite of multiplication.
(e) 600 ÷ 60 = 10 v There are special names for each number in a
(f) 200 ÷ 5 = 40 division:
DIVIDEND ÷ DIVISOR = QUOTIENT
v Example: 12 ÷ 3 = 4, here
12 is the dividend, 3 is the divisor, 4 is the quotient.
v It there is any left over in the division, it is called
remainder.
Operations on Numbers 35
ORDER OF OPERATIONS
Division Algorithm
v Do things in brackets first. If 'a is divided by 'b' and we
v Multiply or divide before you add or subtract. get 'c' as quotient and 'd' as
remainder then
v Otherwise just go left to right and follow the word
BODMAS. a=b×c+d
or Dividend = Divisor ×
v These steps are summarized as BODMAS.
Quotient + Reminder.
B Brackets first For example:
O Orders (i.e. Powers and Square Roots, etc.) If 50 is divided by 3 then
DM Division and Multiplication (left-to-right) quotient obtained is 16 and 2
is the remainder
AS Addition and Subtraction (left-to-right)
So 50 = 3×16 + 2
v You can also divide and multiply rank equally (and go or 50 = 45 + 2
left to right). ⇒ 50 = 50
v And then add and Subtract rank equally (and go left So, we can check if our division
to right). is correct or not by wring this
relation.
PLACE VALUE Example:
1 1
(a) Divided by
v While doing the operation of numbers, place value 2 4
is very important and should be taken into account (b) Divided 0.5 by 0.25
while performing addition, subtraction, multiplication Solution :
and division.
(a) 1 ÷ 1 = 1 × 4 = 2
2 4 2 1

(b) 0.5 500


= = 2
0.25 250
EBD_7364
36 Olympiad Champs–Mathematics

Multiple Choice Questions


LEVEL 1
1. Which of the following property of multiplication is shown in the statement given
below? (Mental Mathematics)
3 × 6 = 6 × 3
(a) Associative (b) Distributive
(c) Identity (d) Commutative
2. The correct order to apply operations is
(a) Addition - Multiplication - Division - Subtraction
(b) Division - Multiplication - Subtraction - Addition
(c) Division - Multiplication - Addition - Subtraction
(d) Subtraction - Addition - Multiplication - Division
3. 96 ÷ 2 = 48 is not equivalent to (Mental Mathematics)
(a) 2 × 24 (b) 4 × 12 (c) 8 × 6 (d) 7 × 8
4. 86 - 23 = ? Which of the following statement is not equivalent to given
statement? (Mental Mathematics)
(a) 7 × 9 (b) 3 × 21 (c) 45 + 18 (d) 187 ÷ 2
5. Which of the following equation shows the identity property of addition?
(a) (6 + 8) + 3 = 6 + (8 + 3) (b) 0+3=3
(c) 9 + 1 = 1 + 9 (d) 1+2=3
6. Which equation shows the commutative property of addition? (Mental Mathematics)
(a) 5 + 4 = 5 + 4 (b) 1 + (4 + 7) = (1 + 4) + 7
(c) 6 + 1 = 7 (d) 8+4=4+8
7. Look at these numbers: (Mental Mathematics)

0.7 0.5 0.8 0.6

Which two numbers in the box have a difference of 0.3?


(a) 0.8 , 0.5 (b) 0.8, 0.7 (c) 0.8, 0.6 (d) 0.7, 0.5
1 1 1
8. Find the number in place of box in 3 +6 + = 10
4 4 10
5 6 7 8
(a) (b) (c) (d)
10 10 10 10
9. If 20 × 2.5 = 50 then 2 × 0.0025 = ?
(a) 0.0005 (b) 0.005 (c) 0.05 (d) 5
Operations on Numbers 37
10. Match the following: (Mental Mathematics)

List I List II
A. 25 + 8 1. 80-11
B. 87 + 6 2. 220-76
C. 78 + 66 3. 42-9
D. 65 + 4 4. 100-7

A B C D
(a) 3 4 2 1
(b) 2 1 3 4
(c) 4 3 1 2
(d) 3 2 1 4
11. Read the statements and choose the correct option.
Statement A : 3 × 15 = 15 × 3.
Statement B : 3 ÷ 2 = 2 ÷ 3.
(a) Only statement A is true. (b) Only statement B is true.
(c) Both A and B are true. (d) Both A and B are false.
12. Match the following : (Mental Mathematics)

List I List II
A. 3×8 1. 6×5
B. 15 × 3 2. 3 × 27
C. 9×9 3. 4×6
D. 3 × 10 4. 9×5

A B C D
(a) 3 4 2 1
(b) 2 3 1 4
(c) 4 3 2 1
(d) 1 3 2 4
13. If there are 5000 mangoes in 100 boxes, how many mangoes will be there in 75
boxes? (2008)
(a) 3570 (b) 3750 (c) 4000 (d) 2750
14. If Fatima has an equal number of notes of denomination 10,5 and 2 and the total
amount she has ` 510, then the number of notes of each she had (Tricky)
(a) 17 (b) 25 (c) 30 (d) 20
EBD_7364
38 Olympiad Champs–Mathematics
15. Estimate. Which sign makes the sentence true?
296 ÷ 5 _____ 78
(a) > (b) <
(c) = (d) None of these
16. There are 1,82,039 roses in a garden. If one rose out of every 13 roses is spoiled,
then the total number of spoiled roses, is  (2017)
(a) 16,343 (b) 14,533
(c) 14,003 (d) Can't be determined

17. 25 ÷ 10
36 9
5 8 5 5
(a) (b) (c) (d)
8 5 2 6

Directions (Qs. 18 to 25) : Use the MULTIPLICATION OPERATION and evaluate the
following questions.
18. 24 × 64

(a) 1356 (b) 1563 (c) 1536 (d) 1653

19. 1042 × 5

(a) 5210 (b) 5102 (c) 5201 (d) 50102

20. 35 × 106

(a) 3710 (b) 3071 (c) 3701 (d) 37011

21. 101.0 × 101

(a) 100011 (b) 10201 (c) 10201.1 (d) 10201.100

7 4
22. × (2009)
9 5
45 28 35 35
(a) (b) (c) 36 (d)
28 45 35
23. 10.14 × 7.6
(a) 77.064 (b) 70.764 (c) 7.7064 (d) 770.64
24. 0.50 × 0.025 (2010)

(a) 125 (b) 0.125 (c) 0.0125 (d) 125.005


1 3
25. × (Mental Mathematics)
7 5
3 21 35 5
(a) (b) (c) (d)
35 5 3 21
Operations on Numbers 39
26. Match the following :

List I List II
A. 3×3 1. 100 ÷ 5
B. 5 × 10 2. 150 ÷ 3
C. 4×5 3. 48 ÷ 6
D. 4×2 4. 72 ÷ 8
A B C D
(a) 4 2 1 3
(b) 3 2 1 4
(c) 1 3 2 4
(d) 2 3 4 1
Directions (Qs. 27 to 32):Use the ADDITION OPERATION and evaluate the following
questions.
27. 881 + 88
(a) 968 (b) 969 (c) 970 (d) 900
28. 2547 + 9998
(a) 12545 (b) 24524 (c) 12505 (d) 12547
29. Look at these numbers:

0.2 2.5 2.6 6.1


Choose two numbers from the box to complete the addition sentence.
______ + _______ = 2.7
(a) 2.6, 0.2 (b) 0.2, 6.1 (c) 2.5 , 0.2 (d) 2.6 + 2.5
30. 428.65 + 500.5 (2011)
(a) 930 (b) 929.70 (c) 929.15 (d) 928.70

3 1 2
31. + + (2012)
7 7 7
5 4 6
(a) (b) (c) 1 (d)
7 7 7
2 2 1
32. + + (2010)
5 5 5
4 6
(a) (b) 0 (c) 1 (d)
5 5
EBD_7364
40 Olympiad Champs–Mathematics

LEVEL 2
Directions (Qs. 33 to 38): Solve the following questions using the correct order of
operations.
33. 1 + 3 × 8 ÷ 2
(a) 15 (b) 5 (c) 13 (d) 17
34. 9 + 1 × 9 ÷ 1
(a) 10 (b) 11 (c) 19 (d) 18
35. 5 × 6 - 9 ÷ 3
(a) 27 (b) 25 (c) 26 (d) 23
36. 5 × 2 - 9 + 5
(a) 0 (b) 5 (c) 4 (d) 6
37. 4 × 2 ÷ 4 + 3
(a) 5 (b) 7 (c) 6 (d) 4
38. 5 + 9 × 8 ÷ 1
(a) 5 (b) 15 (c) 77 (d) 16
Directions (Qs. 39 to 48): Solve the word problems based on the operations of numbers.
39. Tamana bikes 2.2 kilometres each school day. In total, how far will Tamana bike
over 10 school days? (2008)
(a) 2.2 (b) 22 (c) 220 (d) 22.20
40. A chef bought 6.6 kilograms of almonds and 5.8 kilograms of pecans. How many
kilograms of nuts did the chef buy in all?
(a) 12.4 (b) 13.4 (c) 1.24 (d) 13
41. A builder needs 6 nails to finish a project. If the nails come in packages of 6, how
many packages should the builder purchase?
(a) 3 (b) 1.5 (c) 8 (d) 1
42. Maya had 9.2 grams of pepper. Then she used 5 grams of the pepper to make some
scrambled eggs. How much pepper does Maya have ? (Tricky)
(a) 5.2 (b) 6.2 (c) 3.2 (d) 4.2
43. A carpenter bought a piece of wood that was 4.8 centimetres long. Then he sawed
0.9 centimetres off the end. How long is the piece of wood now?
(a) 3.9 (b) 3 (c) 5.7 (d) 6
44. So far, an orchard has sold a total of 77,449 kilograms of fresh fruit and
503 kilograms of frozen fruit this season. In total, how many kilograms of fruit
have been sold? (Critical Thinking)
(a) 77698 (b) 77574 (c) 77952 (d) 77463
Operations on Numbers 41
45. Jaya’s science class places weights on a scale during an experiment. Each weight
weighs 8.4 kilograms. If the class puts 2 weights on the scale at the same time,
what will the scale read? (Critical Thinking)
(a) 16.8 (b) 16.2 (c) 15.8 (d) 10.4
46. Since he was hired, a chef has served a total of 8,805 guests. Of those guests,
8,704 were adults. How many children has the chef served?
(a) 100 (b) 105 (c) 101 (d) 201
47. A birthday card costs `2.37. How much would it cost to buy 4 birthday cards?
(a) `8 (b) `9.48 (c) `9.56 (d) `8.95
48. Each piece of cardboard is 0.4 centimetres thick. If Tanveer stacks 7 pieces of
cardboard on top of one another, how thick will the stack be?
(a) 3 (b) 2.8 (c) 2.6 (d) 47
49. The population of a village is 78692. Out of which 29642 are men and 28167 are
women and the remaining are children. Then the number of children is
(Critical Thinking)
(a) 20883 (b) 21883 (c) 20893 (d) 20783
50. Simplify : 25 × 8358 × 4 (2013)
(a) 837900 (b) 835100 (c) 835800 (d) None of these
51. Which one of the following is divisible by 25? (Tricky)
(a) 4855 (b) 5685 (c) 6790 (d) 7150
52. Divide 50 by half and add 20. From the same, subtract 35. What do you get ?
(Tricky)
(a) 10 (b) 85 (c) 15 (d) None of these
53. Each digit 1, 2, 3, 4, 5, 6, 7, 8 and 9 is represented by a different letter
A, B, C, D, E, F, G, H and I but not necessarily in that order. Further each of
A + B + C, C + D + E, E + F + G and G + H + I is equal to 13. What is the sum
of C, E and G ?  (Critical Thinking)
(a) 7 (b) 9
(c) 11 (d) Cannot be determined
54. A shop has 239 toys. Seventy more toys were brought in. Then 152 of them were
sold. The number of toys left was (2010)
(a) 239 + 70 – 152 (b) 239 – 70 – 152
(c) 239 + 70 + 152 (d) 239 – 70 + 152
55. In the product 3759 × 9573, the sum of tens’ digit and units’ digit is  (2010)
(a) 16 (b) 0 (c) 7 (d) 9
56. In which of the following divisions, will the remainder be more than the remainder
you get when you divide 176 by 3? (2011)
(a) 176 ÷ 2 (b) 173 ÷ 5 (c) 174 ÷ 4 (d) 175 ÷ 3
EBD_7364
42 Olympiad Champs–Mathematics
57. What should be added to the product 103 × 301 to get 31103 ? (2012)
(a) 301 (b) 103 (c) 110 (d) 100
58. What should be subtracted from the product 101 × 101 to get 10101 ?
(a) 102 (b) 101 (c) 100 (d) 99
59. Which operation would be used to solve the problem?
If Jonathan has four candy bars, Rudy has three candy bars, and Duncan has 26
candy bars, how many candy bars do they have altogether?
(a) Addition (b) Subtraction (c) Multiplication (d) Division
60. The result of adding two or more numbers. Or, what you call the answer for an
addition equation.
(a) Product (b) Quotient (c) Sum (d) Difference
61. The aquarium sold tickets to the polar bear exhibit on Saturday and Sunday.
On Saturday, the aquarium sold 584 tickets. On Sunday, the aquarium sold 296
tickets. How many total tickets did the aquarium sell on Saturday and Sunday?
(2011)
(a) 770 (b) 880 (c) 900 (d) 990
62. The Electronics Experts store sold 8,124 computers last year. The store also sold
2,336 printers. How many computers and printers did the store sell last year?
(a) 10,460 (b) 10,640 (c) 10,840 (d) 10,940
63. Ms. Rice drove 12,481 miles and Mr. Whitaker only drove 10,913 miles. How many
more miles did Ms. Rice drive than Mr. Whitaker?
(a) 2568 (b) 1568 (c) 1562 (d) 23394
64. Which number would make this statement true?
674 < _______ < 764
(a) 785 (b) 693 (c) 654 (d) 876
65. 92 people ride the train to work each day. After the train stops at the train
station, only 36 people are still on the train. How would you justify the number of
students who were dropped off at the train station? (Critical Thinking)
(a) 65 people were dropped off at the train station because 92 - 36 = 65
(b) 56 people were dropped off at the train station because 92 + 36 = 56
(c) 56 people were dropped off at the train station because 92 - 36 = 56
(d) 56 people were dropped off at the train station because 92 - 36 = 92 + 36
66. Paco’s store has 45 boxes of plums. In each box there are 345 plums. How many
plums does Paco’s store have in all?
(a) 15,000 (b) 15,525 (c) 14,545 (d) 390
67. Which has the greatest product?  (2010)
(a) 132 × 20 (b) 100 × 40 (c) 142 × 30 (d) 123 × 50
Operations on Numbers 43
68. Which operation would be used to solve the problem?
There are 20 students in Mr. Adam’s class. Each student has 5 dollars. How much
money is there in all?
(a) Addition (b) Subtraction (c) Multiplication (d) Division
69. At the school concert there were 560 people seated in 8 rows. If there were no
empty seats, how many people were in each row? (2012)
(a) 553 people (b) 480 people (c) 70 people (d) 60 people
70. There are 9 rows of seats in a theatre. Each row has the same number of seats.
If there is a total of 162 seats, how many seats are in each row? (2010)
(a) 17 (b) 18 (c) 19 (d) 20
71. There are 26 birdhouses made a factory each hour. What is the total number
ofbirdhouses made at the factory in 8 hours? (2010)
(a) 34 (b) 64 (c) 202 (d) 208
72. There are 18 pupils in a group. There are 10 boys and the rest are girls. 7 pupils
are wearing glasses. If 3 girls are wearing glasses, how many boys are not wearing
glasses? (2011)
(a) 3 (b) 6 (c) 8 (d) 9
73. What is missing number in the box? (2012)
8147 = 8000 + 100 + + 7
(a) 4 (b) 40 (c) 400 (d) 4000
74. Which number should come in place of * to make the number divisible by 3?
3 * 69
(a) 5 (b) 0 (c) 2 (d) 1
75. There are 32 boys in a physical fitness camp. If the total students are 4 times the
number of boys, then how many more girls were there? (2013)
(a) 84 (b) 96 (c) 108 (d) 72
76. A bike travelled 486 km in 6 hours. How far did it travel in 4 hours? (2013)
(a) 224 km (b) 324 km (c) 420 km (d) 234 km
77. The product of 24 tenths and 4 ones is the same as _______. (2011)
(a) 96 tenths (b) 9 tenths 6 hundredths
(c) 96 hundredths (d) 8 ones 6 hundredths
78. Varun purchased a book for 18.45, a pair of shoes for ` 70.25 and a shirt for
` 55.75. How much money did Varun spend in all? (2014)
(a) ` 144.35 (b) ` 154.53 (c) ` 144.45 (d) ` 114.55
EBD_7364
44 Olympiad Champs–Mathematics
79. 100 × 26.25 is same as ________. (2014)
(a) 2.625 × 10 (b) 2.625 ÷ 10 (c) 2625 × 10 (d) 26250 ÷ 10
80. What is the missing value in the given mathematical statement?
0.25 × 12 = 0.25 × 3 + 0.25 × 3 + 0.25 × ___ (2015, Tricky)
(a) 2 (b) 3 (c) 6 (d) 8
81. Divide 80 ones, 2 tenths and 4 hundredths by 4. The result is the same as
________. (2015)
(a) 8.24 ÷ 5 (b) 100.3 ÷ 5 (c) 100.9 ÷ 3 (d) 4280 ÷ 4
82. 28845 is written in words as _______. (2014)
(a) Twenty eight thousand eight hundred and forty five
(b) Twenty thousand eighty eight hundred and forty
(c) Two thousand eight hundred and five.
(d) Two hundred and eighty eight and forty five.

RESPONSE GRID
1. a b c d 2. a b c d 3. a b c d 4. a b c d 5. a b c d
6. a b c d 7. a b c d 8. a b c d 9. a b c d 10. a b c d
11. a b c d 12. a b c d 13. a b c d 14. a b c d 15. a b c d
16. a b c d 17. a b c d 18. a b c d 19. a b c d 20. a b c d
21. a b c d 22. a b c d 23. a b c d 24. a b c d 25. a b c d
26. a b c d 27. a b c d 28. a b c d 29. a b c d 30. a b c d
31. a b c d 32. a b c d 33. a b c d 34. a b c d 35. a b c d
36. a b c d 37. a b c d 38. a b c d 39. a b c d 40. a b c d
41. a b c d 42. a b c d 43. a b c d 44. a b c d 45. a b c d
46. a b c d 47. a b c d 48. a b c d 49. a b c d 50. a b c d
51. a b c d 52. a b c d 53. a b c d 54. a b c d 55. a b c d
56. a b c d 57. a b c d 58. a b c d 59. a b c d 60. a b c d
61. a b c d 62. a b c d 63. a b c d 64. a b c d 65. a b c d
66. a b c d 67. a b c d 68. a b c d 69. a b c d 70. a b c d
71. a b c d 72. a b c d 73. a b c d 74. a b c d 75. a b c d
76. a b c d 77. a b c d 78. a b c d 79. a b c d 80. a b c d
81. a b c d 82. a b c d
Operations on Numbers 45

Solutions with Explanation


LEVEL 1
1. (d) Commutative property. We can multiply in any order and get the same product.
2. (c) According to BODMAS, the correct order of operations is
Division -Multiplication - Addition - Subtraction.
3. (d) 96 ÷ 2 = 48. Now evaluating the options, we get, 2 × 24 = 48 ; 4 × 12 = 48; 8 × 6 = 48
and 7 × 8 = 56. Thus the solution other than 48 is the odd one out. Therefore 7 ×
8 is the odd one out.
4. (d) 86-23 =63. Now evaluating the options, we get 7 × 9 = 63; 3 × 21 = 63, 45 + 18
= 63 and 187 ÷ 3 = 62.333. Thus the solution other than 63 is the odd one out.
Therefore 187 ÷ 3 is the odd one out.
5. (b) Identity property: t = 0 + t
Adding zero does not change a number. Here 0 + 3 = 3 shows the identity property
where adding zero does not change the sum.
6. (d) Commutative property: a + b = b + a
We can add numbers in any order and get the same sum. Here 8 + 4 = 4 + 8 shows
the commutative property.
7. (a) Look for numbers in the box that are greater than 0.3.
Try 0.7. What number can you subtract from 0.7 to get 0.3?
0.7 - 0.4 = 0.3
0.4 is not in the box.
Try 0.5. What number can you subtract from 0.5 to get 0.3?
0.5 - 0.2 = 0.3, 0.2 is not in the box.
Try 0.8. What number can you subtract from 0.8 to get 0.3?
0.8 - 0.5 = 0.3
0.5 is in the box.
The numbers 0.8 and 0.5 have a difference of 0.3.
13 25 101
8. (b)
+ + =
4 4 10
13 + 25 101
⇒ + =
4 10
101 38 101 19

= = – –
10 4 10 2
101 – 95 6
= =
10 10
EBD_7364
46 Olympiad Champs–Mathematics
9. (b) 20 × 2.5 = 50
25
⇒ 20 × = 50 ⇒ 2 × 25= 50
10
2 × 25
⇒ 2 × 0.0025 =
10000
50
= = 0.005
10000
10. (a)

11. (a) 3 × 15 = 45 and also 15 × 3 = 45. Thus A is true. 3 ÷ 2 = 1.5 and 2 ÷ 3 = 0.66 which
are not equal. Therefore only statement A is true.
12. (a)
13. (b) Mangoes in 100 boxes = 5000
5000
mangoes in 1 box = = 50
100
mangoes in 75 boxes = 50 × 75 = 3750.
14. (c) Let the number of notes of each kind be .

510
then 10 +5 +2 = 510  = = 30 .
17
15. (b) 296 ÷ 5 = 59.2. This is less than 78. Thus 296 ÷ 5 < 78.

16. (c) No. of spoiled roses = 182039 ÷ 13 = 14003.


25 10 25 9 5
17. (a) ÷ = × = . [When we divide two fractions, then one number gets
36 9 36 10 8
reciprocated and then the product is simplified.]
18. (c) 24 × 64 = 1536
19. (a) 1042 × 5 = 5210
20. (a) 35 × 106 = 3710
21. (b) 101.0 × 101 = 10201
7 4 28
22. (b) × =
9 5 45
23. (a) 10.14 × 7.6 = 77.064
24. (c) 0.50 × 0.025 = 0.0125
1 3 3
25. (a) × =
7 5 35
26. (a)
27. (b) 881 + 88 = 969
28. (a) 2547 + 9998 = 12545
Operations on Numbers 47
29. (c) Look for numbers in the box that are less than 2.7. Try 0.2. Now we have to find
the number which when added to 0.2 gives 2.7. That number is 2.5.
Thus 2.5 + 0.2 = 2.7.
30. (c) 428.65 + 500.5 = 929.15
3 1 2 6
31. (d) + + =. [Since the denominator is the same, we will only add the numerators.]
7 7 7 7

2 2 1 5
32. (c) + + = = 1 . [Since the denominator is the same, we will only add the
5 5 5 5
numerators. Now, since after addition, both the numerator and denominator are

same which is equal to 1.]

LEVEL 2
33. (c) 1+3×8÷2
=1+3×4
= 1 + 12
= 13
34. (d) 9 + 1 × 9 ÷ 1
=9+1×9
=9+9
= 18
35. (a) Step 1: Multiply and divide from left to right.
5 × 6 - 9 ÷ 3 = 30 - 9 ÷ 3 = 30 - 9 ÷ 3 = 30 - 3
Step 2: Add and subtract from left to right.
30 - 3 = 27
So: 5 × 6 - 9 ÷ 3 = 27
36. (d) Step 1: Multiply and divide from left to right.
5 × 2 - 9 + 5 = 10 - 9 + 5
Step 2: Add and subtract from left to right.
10 - 9 + 5 = 1 + 5 = 1 + 5 = 6
So: 5 × 2 - 9 + 5 = 6
37. (a) Step 1: Multiply and divide from left to right.
4 × 2 ÷ 4 + 3 = 8 ÷ 4 + 3 = 8 ÷ 4 + 3 = 2 + 3
Step 2: Add and subtract from left to right.
2+3=5
So: 4 × 2 ÷ 4 + 3 = 5
EBD_7364
48 Olympiad Champs–Mathematics
38. (c) Step 1: Multiply and divide from left to right.
5 + 9 × 8 ÷ 1 = 5 + 72 ÷ 1 = 5 + 72 ÷ 1 = 5 + 72
Step 2: Add and subtract from left to right.
5 + 72 = 77
So: 5 + 9 × 8 ÷ 1 = 77
39. (b) Multiply the kilometres biked each school day by the number of school days.
Multiply as you would multiply whole numbers.
2.2 × 10 = 220
Count the number of decimal places in the factors. There is 1 decimal place in 2.2.
Move the decimal point 1 place to the left in the answer.
220. = 22.0
Tamana will bike 22 kilometres.
40. (a) Add the numbers of kilograms. Remember to line up the decimal points
6.6 + 5.8 = 12.4
The chef bought 12.4 kilograms of nuts.
41. (d) Divide the number of nails by the number in each package.
6
= 1. The builder should purchase 1 package.
6
42. (d) Subtract the numbers of grams. Remember to line up the decimal points. You can
write extra zeroes to make equivalent decimals.
9.2- 5.0 = 4.2
Maya has 4.2 grams of pepper left.
43. (a) Subtract the numbers of centimetres. Remember to line up the decimal points.
4.8 - 0.9 = 3.9
The piece of wood is 3.9 centimetres long now.
44. (c) Add the kilograms. 77,449 + 503 = 77952 kilograms.
A total of 77,952 kilograms of fruit have been sold.
45. (a) Multiply the weight of each weight by the number of weights. Multiply as you
would multiply whole numbers. 84 × 2 = 168. Count the number of decimal places in
the factors. There is 1 decimal place in 8.4. Move the decimal point 1 place to the
left in the answer.
168 = 16.8
The scale will read 16.8 kilograms.
46. (c) Subtract the number of adults served from the total number of guests.
8805 - 8704 = 101
Thus the chef has served 101 children.
Operations on Numbers 49
47. (b) Cost of 1 birthday card = ` 2.37. Cost of 4 birthday cards = 4 × ` 2.37 = ` 9.48
48. (b) Multiply the thickness of each piece of cardboard by the number of pieces in the
stack.
0.4 × 7 = 2.8. The stack will be 2.8 centimetres thick.
49. (a) Men + Women + Children = 78692
\ Children = 78692 – 29642 – 28167
= 20,883
50. (c) 25 × 8358 × 4= 8358 × 100 = 835800
51. (d)
50
52. (a) + 20 – 35 = 45 – 35 = 10
2
53. (a) A + B + C = 13
C + D + E = 13
E + F + G = 13
G + H + I = 13
Adding these equations,
(A + B + C + D + E + F + G + H + I) + (C + G + E) = 52
(1 + 2 + 3 + 4 + 5 + +6 + 7 + 8 + 9) + (C + G + E) = 52
C+ G + E = 52 – 45 = 7
54. (a) Total number of toys = 239
Since, seventy more toys were brought in
\ Total number of toys = 239 + 70
Now, 152 toys were sold
\ Left toys = 239 + 70 – 152
55. (c) Product of 3759 × 9573 = 35984907
Sum of tens’ and units’ digit = 0 + 7 = 7.
56. (b)

When we divide
176 ÷ 2, Remainder = 0
Similarly, 173 ÷ 5, Remainder = 3
174 ÷ 4, Remainder = 2
175 ÷ 3, Remainder = 1
EBD_7364
50 Olympiad Champs–Mathematics
57. (d) Product of 103 × 301 = 31003
Now, 100 is to be added to get 31103.
58. (c) 101 × 101 = 10201
Required number = 10201 – 10101 = 100.
59. (a) 60. (c)
61. (b) 584 + 296 = 880
62. (a) 8124 + 2336 = 10460
63. (b) 12481 – 10913 = 1568
64. (b) 65. (c) 66. (b) 67. (d) 68. (c)
69. (c) 560 ÷ 8 = 70
70. (b) As 162 ÷ 9 = 18 seats in each now
71. (d) As 26 × 8 = 208 birdhouse
72. (b) No. of boys wearing glasses = 7 – 3 = 4
So, no. of boys not wearing glasses = 10 – 4 = 6
73. (b) 8147 = 8000 + 100 + 40 + 7
74. (b)
75. (b) 128 – 32 = 96 girls
76. (b) In 1 hour, bike travelled = 486 ÷ 6 = 81 km.
In 4 hours, bike travelled = 4 × 81 km = 324 km.
77. (a) 24 tenths × 4 ones = 0.96 = 96 tenths
78. (c) `18.45 + `70.25 + `55.75 = `144.45
79. (d) 100 × 26.25 = 2625 = 26250 ÷ 10
80. (c) 0.25 × 12 = 0.25 × 3 + 0.25 × 3 + 0.25 × 6
81. (d) 80.24 ÷ 4 = 20.06 = 100.3 ÷ 5
82. (a)
Factors and Multiples 51

CHAPTER FOREWORD
Consider a case in which the concept of factors and multiples is used:
Mrs William is very happy, as today is her mother's 50th birthday. She plans to arrange a
party for 20 people and goes to the market to buy 40 eggs, 20 water bottles and 20 hotdogs.
In the market, each item is available under different packets containing different number of
items.

• Egg : 5 eggs in one carton.

• Hotdogs : 4 hotdogs in one packet.

• Water bottles : 2 water bottles in one packet.

Mrs. William bought 8 carton of eggs to get total of 40 eggs.

How many packets of hotdogs and water bottles she should buy?

10 × 1

5×2

2×5

After reading this chapter, you will learn how factors and multiples can be determined and
what are their applications in real life situations.

Factor is a number which divides a given number and multiple is a number is obtained by
multiplying the number by 1, 2, 3 and so on.
EBD_7364
52 Olympiad Champs–Mathematics

Chapter
4 Factors and
Multiples
LEARNING OBJECTIVES
This lesson will help you to:—
Real-Life Example v recognize and learn factors and multiples.
v understand how to find factors and multiples.
v We are surrounded by numbers
in each & every sphere of our v understand the real life applications of factors and
life. Factors & multiples are also multiples.
commonly used in our everyday v understand and draw factor trees.
lives. We use factors when
we want to arrange things in v find common factors and multiples of two numbers.
different ways. For example,
arranging books in rows & Q U I C K C O N C E P T R E V I E W
columns, making groups of
children in different ways etc. FACTORS
It was picture day in Ria’s school. Her teacher made all
the students stand in a single line. But all of them could
not come in the frame.

So she decided to make 2 lines of 10 students each.


Factors and Multiples 53
This way also all the students were not fitting in the
frame.
Then she made 4 lines of 5 each. Now all the students Note : 1 is the only
could fit in the frame. number which has
only one factor. All
other number have at
least two factors.

Amazing
Amazing Facts
Facts

v Every number is the greatest


factor of itself.
v 1 is the factor of every
number.
v The factors of a number
are equal to or less than the
number.
v If we divide one number
by another and there is no
So here we saw three different ways to make 20 students
remainder, then divisor & the
stand in lines. quotient are the factors of the
The first way is 1 × 20 first number.
The second way is 2 × 10 v Every number except 1 has at
least 2 factors. That is 1 & the
& the third way is 5 × 4
number itself.
Therefore, we can say that 1, 20, 2, 5 & 4 are the factors
of 20.
Definition of Factors
The factors of a number are those which divide the number
without leaving any remainder. Thus, factors of a number
divide the number completely.
Note : A number can have many factors.
Prime factors : Factors of a number which are
v
prime are called its prime factors.
Prime factorisation : A factorisation in which
v
every factor is prime is called prime factorisation
of the number.
Co-prime : Two numbers are co-prime if they have
v
only 1 as the common factor.
EBD_7364
54 Olympiad Champs–Mathematics
8 FACTOR TREE
v A Factor Tree is a diagram which is used to break
down a number by dividing it by its factors until all
the numbers left are prime numbers.
4 2 v We can make different factor trees of a same
number.
Example 1: A factor tree of 8 is given in figure A. Here 8
has been broken into 2 factors 4 & 2. But 4 is not a prime
2 2 number. 4 is again broken into 2 factors 2 & 2 as shown in
figure - A. Therefore, the factors of 8 are 2, 2 & 2.
Figure - A
Example 2: We can make factor trees of a number 60 in
different ways as shown in figures - B, C and D:
60
Fig.-B: This is a factor tree of 60. Here 60 has been
broken into two factors 6 & 10. But 6 & 10 are
6 10 not prime numbers. 6 & 10 are again broken into
two factors each. 6 is broken in 2 & 3 and 10 is
broken into 2 & 5.
2 3 2 5
Fig.-C: This is also the factor tree of 60. Here 60 has
Figure - B been broken into 3 & 20. 20 is further broken
into 4 & 5. Again 1 more branch is added to this
factor tree by breaking 4 into 2 & 2.
60
Fig.-D: This is another factor tree for 60. Here 60 has
been broken into 2 & 30. 2 is a prime number
hence we will leave it as it is and further break
3 20 30 into two factors 2 & 15. 15 is again broken into
two factors 3 & 5. Now 3 & 5 are prime numbers,
therefore, the tree is complete now. That is , we
4 5
cannot add any more branch to it.

COMMON FACTORS
2 2
The same factors of two or more than two different
Figure - C
numbers are called common factors.
60 Let us find out the common factors of 15 & 21.
Factors of 15 are 1, 3, 5 & 15.
2 30
Factors of 21 are 1, 3, 7 & 21.
2 15 Therefore, Common factors of 15 & 21 are 1 and 3.

3 5 HIGHEST COMMON FACTOR (H.C.F) OR GREATEST


Figure - D COMMON FACTOR (G.C.F.)
The common factor which is highest among the common
factors of two or more than two numbers is called H.C.F.
of that numbers.
Factors and Multiples 55
Methods to find H.C.F. :
(i) By listing factors : Do You Know ?
Factors of 12 = 1, 2, 3, 4, 6, 12 Amazing Facts
Factors of 32 = 1, 2, 4, 8, 16, 32 H.C.F of two number by Enclid's
Division algorithm is same as
Common factors of 12 and 32 = 1, 2, 4
division algorithm.
H.C.F. of 12 and 32 = 4
Dividend = Divisor × Quotient +
(ii) Division Method : This method is useful to find the Remainder.
H.C.F. of greater numbers. This method involves the For example, to find HCF of 762
following steps: and 1270 we use this method
12700 = 762 × 1 + 508
(a) The greater number is divided by the smaller
number. 762 = 508 × 1 + 254
508 = 254 × 2 + 0
(b) The remainder left after subtraction is taken
As 0 is the remainder in the last
as divisor and divisor as dividend for the next
step.
step of division.
So, HCF = 254.
(c) The process is repeated unless remainder
Note : HCF of any 2 c onsecutive
becomes 0. natural number is 1.
(d) The last divisor is the H.C.F. For example, HCF of 3 and 4 or 4
Example : Find the HCF of 762 and 1270. and 5 or 6 and 7, etc is 1.

Sol. 762 )
1270 ( 1
– 762
508 ) 762 ( 1
– 508
254 ) 508 ( 2
– 508
0
So, HCF of 762 and 1270 is 254.
(iii) By prime factorisation method :
H.C.F. of 12 and 24
2 12 2 24

2 6 2 12

3 3 2 6

1 3 3

1
12 = 2 × 2 × 3
24 = 2 × 2 × 2 × 3
Common prime factors = 2 × 2 × 3 = 12.
So, H.C.F of 12 and 24 = 12.
EBD_7364
56 Olympiad Champs–Mathematics
MULTIPLES
It is Diwali night, Pinki & her family are celebrating &
Try It ! enjoying together. Pinki saw the lightnings all around.
Amazing Facts
Example: Write 3 common multiples
of 7 and 9.
Solution: As 7 and 9 do not have
any common factor,
smallest common multiple
of 7 and 9 = 7 × 9 = 63
Other common multiples of
7 and 9 are 63 × 2 = 126 and
63 × 3 = 189
Example: Find the smallest number
which when divided by She saw many colorful lights. She started counting the
each of the numbers 3, 4, number of bulbs.
5 and 6 leaves remainder
2 in each case. R Y R Y R Y

Solution: First we will find LCM of G B G B G B


3, 4, 5 and 6.
1 2 3 4 5 6 7 8 9 10 11 12
LCM of 3, 4, 5, 6 = 2 × 2 ×
3 × 5 = 60. She saw that blue bulbs are placed at 4th place, 8th place,
Now, the required number 12th place and so on…..
= 60 + 2 = 62 Let us observe these numbers. 4, 8, 12,…..
This is similar to the table of 4 or we can say that these
numbers are the multiples of 4. Can you tell the next
multiple of 4?
Definition of Multiples
A multiple is a number that is the product of a given number
and some other number.
Amazing
Amazing Facts
Facts For example : If A × B = C, C is multiple of both A and B.
v A number is a multiple of itself. v A multiple is basically a time table.
v Every number is a multiple of 1. v We can find the multiple of a number by multiplying
v Every multiple of a number is it by 1,2,3,4, and so on.
greater than or equal to the
number itself. COMMON MULTIPLES
v There is no end to the multiples For Example, let us find out the multiples of 3 & 5.
of a particular number.
3 5
6 10
15 20
9
12 25
Multiples of 3 Multiples of 5
Factors and Multiples 57
Multiples of 3: 3, 6, 9, 12, 15, 18, 21, 24….
Multiples of 5: 5, 10, 15, 20, 25…….
Therefore, the common multiple of 3 & 5 is 15. Try It !
Least Common Multiple (L.C.M.) Example: State
Amazing true or false.
Facts
The least common multiple among the common multiples of (a) A number is divisible by
two or more than two numbers is called their L.C.M. 12 if it is divisible by
Methods to find L.C.M. both 4 and 3.
(i) Listing multiples and then finding out the least (b) A number is divisible by
common multiple. 15 if it is divisible by
both 5 and 2.
Example : Multiples of 5 = 5, 10, 15, 20, 25, 30, 35,
40, --------- (c) A number is divisible by
Multiples of 7 = 7, 14, 21, 28, 35, 42, ---------- 33 if it is divisible by 2
and 11.
Common multiples of 5 and 7 = 35, 70, ---------
Least common multiple of 5 and 7 = 35. (d) A number is divisible by
45 if it is divisible by
(ii) Prime factorization method (Division method)
both 9 and 5.
2 36, 72 Solution:
2 18, 36 (a) Since 12 = 4 × 3
3 9, 18 It is a true statement
(b) Since 15 = 5 × 3
3 3, 6
It is a false statement
2 1, 2 (c) As 33 = 3 × 11
1, 1 So, it is a false statement
Now, 2 × 2 × 3 × 3 × 2 = 72 (d) As 45 = 9 × 5.
Hence, L.C.M. of 36 and 72 = 72. It is a true statement
Relationship between L.C.M. and H.C.F.
L.C.M. × H.C.F. = Product of the two numbers.

Rules of Divisibility
1. A number is divisible by 2 if it has 0 or an even
number in its ones place.
2. A number is divisible by 3 if the sum of its digits is
divisible by 3.
3. A number is divisible by 4 if the number formed by
the tens and ones digits is divisible by 4.
4. A number is divisible by 5 if it has 0 or 5 in its ones
place.
5. A number is divisible by 10 if it has 0 in its ones
place.
6. A number is divisible by 9 if the sum of its digits is
divisible by 9.
7. A number is divisible by 15 if it is divisible by both
3 and 5.
Note : If a number is divisible by another number, it is
also divisible by each factor of that number.
EBD_7364
58 Olympiad Champs–Mathematics

Multiple Choice Questions


LEVEL 1
1. ________ are the numbers which are multiplied together to get another number.

(a) Factors (b) Multiples (c) Product (d) Difference


2. A ________ is a diagram which is used to break down a number by dividing it by
its factors until all the numbers left are prime numbers.
(a) factor tree (b) product tree
(c) maths tree (d) multiplication tree
3. ________ is the factor of every number.
(a) 0 (b) 1 (c) 2 (d) 3
4. Every number is the ________ factor of itself.
(a) greatest (b) smallest
(c) equal to (d) less than equal to
5. The factors of any number are ________ the number.
(a) equal to or less than (b) equal to or greater than
(c) equal to (d) less than
6. We can find factors by ________.
(a) multiplication (b) division
(c) multiplication & division (d) addition
7. When we divide one number by another and there is no remainder, the divisor & the
quotient are the ________ of the first number.
(a) factors (b) multiples (c) product (d) answer
8. Pick the odd one out. (Mental Mathematics)

(a)
2 (b)
4 (c)
6 (d)
7
9. Every number except 1 has at least ________ factors. (Mental Mathematics)

(a)
2 (b)
3 (c)
4 (d)
5
10. A ________ is a number that is the product of a given number and some other
number.(2016)
(a) multiple (b) factor (c) product (d) factor tree
Factors and Multiples 59
11. A multiple is basically a ________.
(a) time table (b) factor tree
(c) product tree (d) division table
12. We can find the ________ of a number by multiplying it by 1,2,3,4, and so on.
(a) factors (b) multiples (c) products (d) sum
13. Pick odd one out. (Mental Mathematics)

(a)
3 (b)
6 (c)
9 (d)
10
14. Every multiple of a number is ________ the number itself. (2010)
(a) greater than or equal to (b) less than or equal to
(c) greater than (d) less than
15. There is no end to the ________ you can get of a particular number.
(a) multiples (b) factors (c) product (d) sum
16. 56 is a ________ of 8. (Mental Mathematics)
(a) factor (b) multiple (c) product (d) difference
17. The common factors of 56 & 44 are ________. (2008, Mental Mathematics)
(a) 1, 2, 4. (b) 1, 2, 11. (c) 1, 8. (d) 2.
18. Is 6 a factor of 78 ? (Mental Mathematics)
(a) Yes (b) No (c) May be (d) May not be

19. How many factors do prime numbers have? (Mental Mathematics)


(a) 0 (b) 1 (c) 2 (d) 3
20. If 6 × 5 = 30, then 6 and 5 are
(a) addends (b) divisor (c) factors (d) none of these
21. Which pair of numbers has a greatest common factor 1?
(a) 7 and 20 (b) 3 and 45 (c) 15 and 18 (d) 4 and 16
22. What is the seventh multiple of 9? (Mental Mathematics)
(a) 45 (b) 54 (c) 63 (d) 72
23. Which list shows the first five multiples of 7? (Mental Mathematics)
(a) 7, 8, 9, 10, 11 (b) 7, 9, 11, 13, 15
(c) 7, 14, 21, 28, 35 (d) 7, 49, 343, 2,401, 6,807
24. Which list contains all prime numbers? (2017)
(a) 1, 7, 13, 29 (b) 2, 13, 27, 41 (c) 5, 11, 29, 31 (d) 6, 18, 21, 50
25. Which number is neither a prime number nor a composite number?
(a) 3 (b) 5 (c) 1 (d) 2
EBD_7364
60 Olympiad Champs–Mathematics
LEVEL 2
26. Which of the following pairs of numbers are co-primes?
(a) (11, 111) (b) (22, 222) (c) (12, 36) (d) (14, 50)
27. Match the following : (Mental Mathematics)

List I List II
A. Factors of 6 1. 1, 3, 9.
B. Factors of 9 2. 5, 10, 15, 20, …..
C. Multiples of 5 3. 3, 6, 9, 12, ….
D. Multiples of 3 4. 1, 2, 3, 6.

A B C D
(a) 4 1 2 3
(b) 4 1 3 2
(c) 1 4 2 3
(d) 4 3 2 1
28. Which number completes the puzzle? (2010)

5 7 11 13 17 ?

(a) 23 (b) 19 (c) 21 (d) 20


29. Fill in the blanks by choosing the correct option for the given factor tree of 120.
(2008, Mental Mathematics)
120

10

5 2 6 2

(a) 12, 2, 2 (b) 12, 2, 3 (c) 2, 3, 1 (d) 1, 2, 3


30. Directions (Qs. 30 to 34) : Consider the following statements and choose the
correct option.
Statement A : A number which has more than two factors is called a prime number.
Statement B : A number which has only two distinct factors, is called a composite
number. (Critical Thinking)
(a) Statement A is correct. (b) Statement B is correct.
(c) Both are correct. (d) Both are incorrect.
Factors and Multiples 61
31. Statement A : If one of the two given numbers is a multiple of the other, the greater
number is the L.C.M. of the given numbers.
Statement B : The Highest Common Factor of two or more given numbers is the greatest
among all their common factors. (Tricky)
(a) Statement A is correct. (b) Statement B is correct.
(c) Both are correct. (d) Both are incorrect.

32. Statement A : The multiples of 8 are 16, 24, 32, 40, 48…. (Critical Thinking)
Statement B : The common multiples of 8 & 6 are 24 & 48.
(a) Statement A is correct. (b) Statement B is correct.
(c) Both are correct. (d) Both are incorrect.

33. Statement A : The H.C.F. is the greatest number which divides two or more numbers
without a remainder. (Critical Thinking)
Statement B : The L.C.M. of two or more numbers is the smallest number which is
divisible by each one of the given numbers.

(a) Statement A is correct. (b) Statement B is correct.


(c) Both are correct. (d) Both are incorrect.
34. Statement A : 30 is the smallest number which when divided by 10 and 15 leaves no
remainder.
Statement B : 4 is the largest number that divides 12 and 20 without a remainder.
(a) A is correct. (b) B is correct.
(c) Both are correct. (d) Both are incorrect.
35. Find the odd one out. (2011)

(a)
15 (b)
21 (c)
27 (d)
31
36. Which of the following represents the complete factor tree for 84?
 (2008, Mental Mathematics)

(a) 84 (b) 84

2 42 2 42

6 7 6 7

3 2

EBD_7364
62 Olympiad Champs–Mathematics

(c) 84 (d) 84

7 12 21 4

6 2 2 2

37. The number of factors of 126 are _______. (2013)



(a)
12 (b)
14 (c)
16 (d)
18
38. In how many different ways we can arrange 24 glasses on a kitchen shelf ? Choose
the best answer.
(a) 4 ways
1. All 24 glasses in 1 row. 2. 2 rows of 12 glasses each.
3. 3 rows of 8 glasses each. 4. 4 rows of 6 glasses each.
(b) 3 ways
1. All glasses in 1 row.
2. 2 rows of 12 glasses each.
3. 3 rows of 8 glasses each.
(c) 2 ways
1. All glasses in 1 row.
2. 3 rows of 8 glasses each.
3. 4 rows of 6 glasses each.
(d) 1 way
1. All glasses in 1 row.
39. Sum of factors of 71 is _______. (2014)
(a) 72 (b) 194 (c) 71 (d) 78
40. Match the following :

List I List II
A. 93 is not a multiple of 1. 1
B. Every number is a multiple of 2. 3
C. 12 is a multiple of 3. 31
D. 62 is a multiple of 4. 7

A B C D
(a) 4 1 2 3
(b) 4 1 3 2
(c) 1 4 2 3
(d) 4 3 2 1
Factors and Multiples 63
41. The greatest number that will divide 37, 50, 123 leaving remainder 1, 2 and 3
respectively is : (2010)
(a) 9 (b) 10 (c) 15 (d) 12
42. Which of the following statements are true or false. (Tricky)
(A) If a number is divisible by 6, it must be divisible by 12.
(B) If a number is divisible by 6, it must be divisible by 3.
(C) If a number is divisible by 100, it must be divisible by 25.
(D) A number divisible by 256 is even.
(a) FTFT (b) FTTT (c) TFTT (d) TTFT
43. The greatest number that will divide 137, 182 and 422 leaving a remainder of 2 in
each case is : (2017)
(a) 15 (b) 12 (c) 21 (d) none of these

Directions (Qs. 44 to 48) : Read the passage and complete it by choosing the correct
option.

A number is divisible by ___(30)___ if it has 0 or an even number in its ones place. A number
is divisible by 3 if the ___(31)___ of its digits is divisible by 3. A number is divisible by 5 if it
has ___(32)___ or 5 in its ones place. A number is divisible by ___(33)___ if it is divisible by
both 3 and 5. A number is divisible by another number if it is also divisible by its ___(34)___
factors.
(a)
44. 2 (b)
3 (c)
4 (d)
6
45. (a) sum (b) product (c) difference (d) division
46. (a) 0 (b) 1 (c) 10 (d) 2
47. (a) 15 (b) 3 (c) 5 (d) 10
48. (a) co-prime (b) prime (c) each (d) others
49. The smallest 4-digit number exactly divisible by 12, 15, 20 and 35 is _______.
 (2011, Tricky)
(a) 1,000 (b) 1,160 (c) 1,260 (d) none of these
50. List the prime factorization of 24: (Mental Mathematics)
(a) 2×2×2×3 (b) 3×2×4 (c) 3 × 8 (d) 12 × 2
51. Add the 4th multiple of 52 and the 8th multiple of 37. The value obtained is 700
less than . Find . (2016)

(a)
1400 (b)
1004 (c)
968 (d)
1204
52. Find the GCF of 15 and 6: (Mental Mathematics)
(a) 5 (b) 6 (c) 3 (d) 2
EBD_7364
64 Olympiad Champs–Mathematics
53. The difference between the (sum of third multiple of 18 and the sixth multiple of
12) and sum of factors of 24 is ______. (2014)
(a) 66 (b) 186 (c) 176 (d) 126
54. What is the greatest common factor (GCF) of 42 and 72?
(a) 2 + 2/3 (b) 3 (c) 6 (d) 9
55. Look at the calendar. What best describes the dates that are on Saturdays?
 (2011)

(a) Multiples of 2 (b) Multiples of 3


(c) Multiples of 4 (d) Multiples of 7
56. Which of the following pairs of numbers have a least common multiple (LCM) of 56?

(a) 2 and 28 (b) 7 and 8 (c) 4 and 14 (d) 3 and 17

57. Girish is making a math puzzle. He writes that ‘w’ is an even number which has 12
as a factor. Which of the following could represent the variable ‘w’? (2010)

(a)
2156 (b)
1728 (c)
1429 (d)
1256
58. Which two numbers do not have 11 as their greatest common factor?

(a) 22 and 44 (b) 33 and 55 (c) 11 and 101 (d) 33 and 99

59. Which one of the following digits should be placed in the middle of the digits of
the number 258970 so that 3 becomes factor of it?

(a) 0 (b) 1 (c) 2 (d) 3

60. A number that has more factors than one and itself. Or, a number that has more
than two factors. For example: 8 has factors 1 and 8, but it also has factors 2
and 4.

(a) Prime number (b) Composite number

(c) Factor number (d) Multiple number


Factors and Multiples 65
61. The smallest multiple (other than zero) that two or more numbers have in common.
For example: the multiples of 3 are 3, 6, 9, 12, 15.... and the multiples of 5 are
5, 10, 15, 20..... The smallest multiple that both 3 and 5 have in common is 15.
(a) Least Common Multiple (b) Least Common Factor
(c) Largest Common Multiple (d) Largest Common Factor
62. A positive number that is divisible only by itself and the number one. For example:
3 has only two factors - one and three - one and itself.
(a) Prime Number (b) Composite Number
(c) Factor Number (d) Addend Number
63. What is the largest factor that two or more numbers have in common. For example:
The factors of 8 are 1, 2, 4, and 8. The factors of 12 are 1, 2, 3, 4, 6, and 12
The largest factor that both 8 and 12 have is the number - 4. Four is the largest
number both 8 and 12 have in common.
(a) GCF - Greatest Common Factor (b) LCF - Least Common Factor
(c) GCM - Greatest Common Multiple (d) LCM - Least Common Multiple

64. Which of the following best defines the properties of a composite number?
(a) It has more than 2 factors.
(b) It has only 2 factors, one and itself.
(c) It has more than 2 multiples.
(d) It has only 2 multiples.
65. Which list shows all the prime numbers between 1 to 50 (2012)
(a) 1, 2, 3, 5, 7, 11, 13, 17, 19, 23, 29, 31, 37, 41, 43, 47
(b) 2, 3, 5, 7, 11, 13, 17, 19, 23, 29, 31, 37, 41, 43, 47
(c) 3, 5, 7, 11, 13, 17, 19, 23, 29, 31, 37, 41, 43, 47
(d) 2, 5, 7, 11, 21, 17, 19, 23, 29, 37, 43, 47
66. Beginning at 8:30 am, the Calgary Zoo starts tours of the tiger and elephant
enclosures. Tours for the tiger enclosure leave every 15 minutes. Tours of the
elephant enclosure leave every 20 minutes. How often do the tours leave at the
same time?
(a) Every 15 minutes (b) Every 45 minutes
(c) Every 60 minutes (d) Every 50 minutes
67. Bhushan counted to 60 using multiples of 6. Which statement is true about multiples
of 6?
(a) They are all odd numbers
(b) They all have 6 in the ones place
(c) They can all be divided evenly by 3
(d) They can all be divided evenly by 12
EBD_7364
66 Olympiad Champs–Mathematics
68. Read the statements and choose the correct option. (2014, Critical Thinking)
Statement A: If a number can only be divided by one and itself, it is a prime number.

Statement B: Prime number has only 2 factors, one and itself.
(a) Only statement A is correct. (b) Only statement B is correct.
(c) Both the statements are correct (d) Both the statement are wrong.
69. A number which is a factor of every number is :
(a) 0 (b) 1 (c) 2 (d) 5
70. Rohan asked a question to Shruti.
“Multiply the second multiple of 12 by the third multiple of 8”.
Help her in solving the question and finding the correct result. (2015)

(a)
48 (b)
96 (c)
192 (d)
576

RESPONSE GRID
1. a b c d 2. a b c d 3. a b c d 4. a b c d 5. a b c d
6. a b c d 7. a b c d 8. a b c d 9. a b c d 10. a b c d
11. a b c d 12. a b c d 13. a b c d 14. a b c d 15. a b c d
16. a b c d 17. a b c d 18. a b c d 19. a b c d 20. a b c d
21. a b c d 22. a b c d 23. a b c d 24. a b c d 25. a b c d
26. a b c d 27. a b c d 28. a b c d 29. a b c d 30. a b c d
31. a b c d 32. a b c d 33. a b c d 34. a b c d 35. a b c d
36. a b c d 37. a b c d 38. a b c d 39. a b c d 40. a b c d
41. a b c d 42. a b c d 43. a b c d 44. a b c d 45. a b c d
46. a b c d 47. a b c d 48. a b c d 49. a b c d 50. a b c d
51. a b c d 52. a b c d 53. a b c d 54. a b c d 55. a b c d
56. a b c d 57. a b c d 58. a b c d 59. a b c d 60. a b c d
61. a b c d 62. a b c d 63. a b c d 64. a b c d 65. a b c d
66. a b c d 67. a b c d 68. a b c d 69. a b c d 70. a b c d
Factors and Multiples 67

Solutions with Explanation


LEVEL 1

1. (a) 2. (a) 3. (b) 4. (a) 5. (a)


6. (c) 7. (a)
8. (d) 2, 4, 6 are all the multiples of 2 but 7 is not the multiple of 2.
9. (a) 10. (a) 11. (a) 12. (b)
13. (d) 3, 6 and 9 are multiples of 3 but 10 is not multiple of 3.
14. (a) 15. (a)
16 . (b) 8 × 7 = 56
17. (a) 56 = 2 × 2 × 2 × 7 × 1
44 = 2 × 2 × 11 × 1
So, common factors are 1, 2, 4
18. (a) Since, 78 = 2 × 3 × 13 × 1
Hence 6 is a factor of 78.
19. (c) 20. (c) 21. (a) 22. (c) 23. (c)
24. (c) 25. (c)

LEVEL 2
26. (a) 27. (a) 28. (b) 19 is the next prime number
29. (b) 30. (d) 31. (c) 32. (c)
33. (c) Both statements are definitions.
34. (c) 35. (d) 31 is not a multiple of 3 and rest are multiple of 3
2 84
36. (a)
2 42
3 21
7 7
1

37. (a) 126 has 12 factors (1, 126, 2, 63, 3, 42, 6, 21, 7, 18, 9, 14)
38. (a)
39. (a) Sum of factor of 71 = 1 + 71 + = 72
40. (a)
EBD_7364
68 Olympiad Champs–Mathematics
41. (d) 37 – 1 = 36, 50 – 2 = 48, 123 – 3 = 120
HCF of 36, 48 and 120 = 12
\ required number = 12.
42. (b) 43. (a) 44. (a) 45. (a)
46. (a) 47. (a) 48. (a)
49. (c) The smallest 4-digit = 1000
Now L.C.M of 12, 15, 20 and 35
2 12, 15, 20, 35
2 6, 15, 20, 35
3 3, 15, 5, 35
5 1, 5, 5, 35
7 1, 1, 1, 1,

1, 1, 1, 1
L.C.M = 2 × 2 × 3 × 5 × 7 = 420
We divide 1000 by 420
420 ) 1000 ( 2
– 840
160 – Remainder
\ The least 4-digit number that is exactly divisible by 12, 15, 20 and 35
= 1000 + (420 – 160) = 1000 + 260 = 1,260
50. (a)
51. (d) Symbol = 52 × 4 + 37 × 8 + 700 = 208 + 296 + 700 = 1204
52. (c) 3
53. (a) Sum of third multiple of 18 and sixth multiple of 12 = 54 + 72 = 126
Sum of factors of 24 = 1 + 24 + 2 + 12 + 3 + 8 + 4 + 6
So difference = 126 – 60 = 66
54. (c) 6
55. (d) 7, 14 , 21 and 28 are all multiple of 7.
56. (b) 7 and 8
57. (b) 1728 is even and has 12 as its factor
58. (c) 11 and 101
59. (c) Sum of digits of the number 258970 = 2 + 5 + 8 + 9 + 7 + 0 = 31. Therefore, 2
should be placed in the middle of the digits such that sum of the digits becomes
divisible by 3
60. (b) 61. (a) 62. (a) 63. (a) 64. (a)
65. (b) Represents all prime number between 1 and 50
66. (c) 67. (c) 68. (c) 69. (b)
70. (d) As 24 × 24 = 576
Fractions and Operation on Fractions 69

CHAPTER FOREWORD
Consider the following case :
Mrs. Venkat had 6 bananas and wanted to divide it amongst her 4 children. She decided to give
2 bananas each to her 2 younger children and 1 banana each to the other two children.
2 1
Hence, fraction of whole bananas given to each of the younger children = =
6 3
1
Fraction of whole bananas given to each of the elder children =
6
Here Mrs. Venkat applied her knowledge of fraction for the distribution of bananas among
her children.

"A fraction is a part of a whole."

1 1 3
2 4 4

1
1 1
2 2
1 1 1 1
4 4 4 4
1 1 1 1 1 1 1 1
8 8 8 8 8 8 8 8

The word 'Fraction' is taken from Latin word 'fractus' which means "broken". In general, a
fraction refers to how many parts are there of a certain size.

After reading this chapter, you will learn about fractions, their comparison, conversion into
decimal notification and vice-versa.

Note : Every Natural number can be written as a fraction as natural number upon 1 is a
3 5 6
fraction. For example 3 = 1 , 5 = 1 , 6 = and so on
1
In a fraction, both numerator and denominator are positive.
EBD_7364
70 Olympiad Champs–Mathematics

Chapter
5 Fractions and
Operation on Fractions
LEARNING OBJECTIVES
Real-Life Example This lesson will help you to:—

v Time is a very good example v learn and study about finding fractional part of a
of fractions. Time is divided collection.
into various denominations of v learn to compare one and more fractions.
hours, minutes and seconds. For v learn to study about identifying equivalent
example, 1 hour has 60 minutes fractions.
or it can also be said that 1 hour
v study and learn to estimate the degree of closeness
can be divided into 60 equal
fractions each being equal to 1
minute.
(
of a fraction to known fractions 1 , 1 and 3 .
2 4 4)
v understand and study about expressing a given
fraction into decimal notation.
v comparison of fractions.
v identifying equivalent fractions.
v add, subtract, multiply or divide the fractions.

Try It ! QUICK CONCEPT REVIEW


v Fractional Part: The fractional part of a number

Example : Find the fraction is the part of the number that appears after the
whose numerator is smallest decimal point.
prime number and denominator
is 3 more than numerator
FRACTIONS
Solution : Number = 2 v A fraction is a way of representing division of a

Denominator = 2 + 3 ‘whole’ into parts.
Fraction = 2 v A fraction is a way of representing division of a
5
‘whole’ into parts. It has the form
Numerator
Denominator
Where the
Numerator = Number of parts chosen
Fractions and Operation on Fractions 71
Denominator = Total number of the parts
Example: Fraction 1 is shown by the pie chart
3
below. The pie is divided into 3 equal parts. The Historical
Amazing Facts
Preview
shaded part is equal to one third of the pie, thus 1 . v In India, fractions were written
3
with one number (the
numerator) above (the
denominator), but without a
line for example: 7
15
v It was the Arabs who added
the line (sometimes drawn
horizontally, sometimes on a
slant) which we now use to
Part of a whole separate the numerator and
7
denominator for example:
15

3
_
4
 the top number (the numerator) says how many parts
the whole is divided into.
 the bottom number (the denominator) says how many
you have.

LIKE AND UNLIKE FRACTIONS


Like Fractions: Fractions having the same denominator
are called like fractions. Historical
Amazing Facts
Preview
1 3
Example: and are like fractions having the same v In ancient Rome, fractions were
2 2 written using words instead of
denominator as 2. numbers.
Unlike Fractions: Fractions having different denominator v Fractions were firstly used in
are called unlike fractions. the Indus Valley civilization,
9 4 followed by the Egyptians and
For example: and .
7 9 the Greeks.
v By about 500AD, the Indians
COMPARING FRACTIONS had developed a system from
Fractions are compared to see if one fraction is equal to a way of writing called brahmi,
(=), greater than (>) or smaller than (<) the other fraction. which had nine symbols and a
• While comparing the fractions, if the fractions are zero.
like fractions, the fraction with bigger numerator is
greater.
• If the fractions are not like fractions, convert
fraction into like fraction using the LCM of the
denominator and then compare.
EBD_7364
72 Olympiad Champs–Mathematics
Steps to compare two fractions :
Step 1: Find a common denominator by taking L.C.M of all
Shortcut
different denominators.
Step 2: Make equivalent fractions with LCM as the new
Cross multiplication method to
denominator.
compare given two fraction. If
Step 3: Compare the numerators.
we have to compare a and
c
Example: Compare the fractions 3 and 7 .
b d 5 9
where a, b, c, d are all different,
Step 1: L.C.M of 5 and 9 = 45
then we can compare then by 3 3 9 27 7 7 × 5 35
Step 2: = × = ; = =
a c 5 5 9 45 9 9 × 5 45
using this short cut
Step 3: Since 27 < 35 so 3 < 7 .
b d
a × d and b × c are compared.
5 9
If ad > bc, then a , then >
c
Note : If the numerator is same, the fraction with
b d greater denominator is smaller.
If bc > ad, then a <
c
b d
This method saves the time as
Shortcut Method
we don't have to find LCM of To compare the two fractions, simplify cross multiply
denominator to make them lite them.
fraction Example : Compare the two fraction and put '>' or '<' or
'=' using short cut method.
4 6 9 3
(a) (b)
5 7 11 7
3 9
(c)
13 39
Solution :
(a) 4 × 7 < 5 × 6
Try It!
28 < 30
Example : Express 4 6
(a) 20 paise as a fraction of So <
5 7
a rupee
(b) 35 cm as a fraction of a (b) 9 × 7 > 3 × 11
meter
Solution: 63 > 33
20
(a) 20 paise = ` 9 3
100 So >
1 11 7
= `
5
(c) 3 × 39 = 13 × 9
(b) 35 cm = 35 m
100
7
= m 117 = 117
20
3 9
So =
13 39
Fractions and Operation on Fractions 73

Misconcept/concept
EQUIVALENT FRACTION
To find equivalent fractions, multiply the numerator and Misconcept : The common
denominator by the same number (except zero). misconception is that decimals
and fractions are different
Equivalent fractions may look different, but they have
types of numbers. Hence there
the same value.
is no equivalent fraction for any
decimal.
Concept : Taking an example of
= = a decimal 4.422 which is to
be converted to its equivalent
fraction. Now this decimal can be
expressed as 4 and the fraction
3 = 6 = 9 422/1000 or 4422/1000. These
4 8 12 are fraction equivalent. These
The fractions three-fourths, six-eighths, and nine- can be simplified by dividing both
twelfths are equivalent. numerator and denominator by
2 to give 4.422= 2211/500 or
Estimating the degree of closeness of a fraction 4(211/500).
1
v A fraction is closest to when the denominator is Misconcept : The value of A ÷ 1/B
2 is equivalent to A÷B and hence
about twice the numerator. has value A/B. Taking an
2 5 8 example, The value of 3÷1/4 is
Example: , , etc. equivalent to 3÷4 and hence has
5 11 15
3/4 or 0.75.
1
v A fraction is closest to when the denominator is Concept : The division of 3 ÷ 1/4
4 means how many 1/ 4 are there
about four times the numerator. in the number 3. Clearly there
3 4 7 are 3 quarters in 1 and hence
Example: , , etc. 3 × 4 ( = 12) in 3. So 3 ÷ 1/4 = 12.
14 17 26
Or A ÷ 1/B = AB.
Facts: For a fraction, a denominator cannot be 0.
ADDITION AND SUBTRACTION OF FRACTIONS 2
For Like Fractions Example : Express as a decimal
5
Fractions can only be added and subtracted only if they
2 2×2 4
have common denominators. Solution : = =
5 5×2 10
For example: 2
5 3 5 +3 8 \ = 0.4
+ = = 5
4 4 4 4 Example : Express the following
decimals into a fraction.
When adding fraction with same denominators, add the
numerators. (a) 0.3 (b) 0.25
5 4 5–4 1 Solution :
– = = 3 25 1
3 3 3 3 (a) 0.3 = (b) 0.25 = =
10 100 4
When subtracting fractions with same denominators,
subtract the numerators.
EBD_7364
74 Olympiad Champs–Mathematics
For Unlike Fractions
First convert unlike fractions to equivalent like fractions
and then add or subtract.
Amazing
Shortcut
Facts
3 1
For example: +
There is a simple rule that works 4 5
for any two fractions.
a c a×d+b×c 3×5 15 1×4 4
+ = = and =
b d b×d 4×5 20 5×4 20

Or 3 1 15 4 15 + 4 19
\ + = + = =
a c a×d–c×b 4 5 20 20 20 20
- =
b d b×d
For example:
MULTIPLICATION OF FRACTIONS
3 5 3×8+5×4 To Multiply Fractions:
+ =
4 8 4×8 1. Multiply the numerators of the fractions to get the
new numerator.
24 + 20 44
= = 2. Multiply the denominators of the fractions to get
32 32
the new denominator.
3 5 3×8–5×4 1 2 1×2 2
– = For example: × = =
4 8 4×8 5 3 5 × 3 15

24 – 20 4 DIVISION OF FRACTIONS
= =
32 32 To divide fractions, reverse the numerator and
denominator of the second fraction and change the
division sign to a multiplication sign.
Do you know? For example:
1 18 1 20 20
÷ = × =
Reciprocal of 0 (zero) is not 2 20 2 18 36
defined. REDUCING OR SIMPLIFYING FRACTIONS
To reduce a fraction to lowest terms, divide the numerator
and denominator by their Highest Common Factor (HCF)
20
Remember For example:
60

Fractions can also be simplified HCF of 20 and 60 = 20


by dividing the numerator and 20 ÷ 20 1
\ =
denominator by any of the 60 ÷ 20 3
common factors. Continue with
reducing the fraction until we
get 1 as common fraction.

For example:
12 12 ÷ 2 6 ÷ 2 3
= = =
16 16 ÷ 2 8 ÷ 2 4
Fractions and Operation on Fractions 75

Multiple Choice Questions


LEVEL 1
1. Pick the odd one out. (Mental Mathematics)

(a)

(b)

(c)

(d)
1 6 1
2. of 6 oranges: orange:: of 11 apples: ____ apple (Mental Mathematics)
5 5 6
11 6 5 6
(a) (b) (c) (d)
6 11 6 5
Directions (Qs. 3 and 4) : Find the value of ‘*’ in the following fractions. (Mental Mathematics)
5
3. = *
7 21
(a) 16 (b)
18 (c)
15 (d)
17
3 18
4. =
* 24

(a)
4 (b)
5 (c)
6 (d)
8
32 42
5. By how much is greater than ?
70 100
45 57 13 13
(a) (b) (c) (d)
94 100 350 700
5
6. Which of the following figure represents of a whole. (Mental Mathematics)
12

(a) (b) (c) (d)

7. Find the missing number. (Tricky)


3 6 9 12
, , ,
4 8 12 ?
(a) 15 (b) 14 (c) 16 (d) 12
8. Which of the following is the smallest fraction? (2009)
4 6 7 2
(a) (b) (c) (d)
11 11 11 11
EBD_7364
76 Olympiad Champs–Mathematics

2 3
9. What is the value of + (2015)
7 7
15 5 6 35
(a) (b) (c) (d)
7 7 7 14
Directions (Qs. 10 to 12) Convert the following fractions into decimals. (Mental Mathematics)
786
10.
1000

(a) 0.786 (b) 78.6 (c) 786.0 (d) 7.86


1478
11.
100000

(a) 147.8 (b) 0.1478 (c) 0.01478 (d) 14.78


133
12.
10

(a) 0.133 (b) 1.33 (c) 13.3 (d) 0.0133


Directions (Qs. 13 to 18) : Fill in the blanks by putting the correct operator ‘<’, ‘>’
or ‘=’. (Mental Mathematics)
4 8
13. _____
10 20

(a) > (b) < (c) = (d) None of these


5 1
14. ____
6 6

(a) > (b) < (c) = (d) None of these


7 3
15. ____
8 8

(a) > (b) < (c) = (d) None of these


8 4
16. ____ (2011)
15 5

(a) > (b) < (c) = (d) None of these


3 6
17. ____
7 14

(a) > (b) < (c) = (d) None of these


5 7
18. ____ (2012)
12 12

(a) > (b) < (c) = (d) None of these


Fractions and Operation on Fractions 77
19. What part of the given 10 cm ribbon is shaded when each fragment equals to 1 cm?
(Express your answer in decimal fraction.) (Mental Mathematics)

(a) 0.8 (b) 0.6 (c) 0.9 (d) 0.4

20. What is the next fraction in this sequence?

2 4 8 16
, , , ,….
77 77 77 77

22 30 32 42
(a) (b) (c) (d)
77 77 77 77

LEVEL 2
1
21. Which of the following is equal to
? (2012)
3
2 4 1 2 2
(a) ÷ 1 (b)
1 ÷ (c) × 4 (d) ÷
3 6 12 6 6

22. Match the corresponding equivalent fraction given in list II with the fractions given
in list I. (Mental Mathematics)

List I List II

A. 3 1. 35
8 42
B. 4 2. 6
5 16
C. 7 3. 16
10 20
D. 5 4. 35
6 50

A B C D

(a) 2 3 4 1

(b) 1 3 2 4

(c) 3 2 1 4

(d) 1 3 2 4
EBD_7364
78 Olympiad Champs–Mathematics
23. Match the following: (Critical Thinking)
List I List II

A. 1 1. 2 rupee 50 paisa
of ` 6
5
B. 1 2. 50 paisa
of ` 18
9
C. 1 3. 1 rupee + 20 paisa
of ` 10
4
D. 1 4. 200 paisa
of ` 3
6
A B C D
(a) 3 4 1 2
(b) 2 3 4 1
(c) 1 3 2 4
(d) 3 1 2 4
1
24. Which of the following has the same value as 5 × ? (2010)
2
1 1 1
(a) + 5 (b) +
2 2 5
1 1 1 1 1 1 1 1 1 1
(c) × × × × (d) + + + +
2 2 2 2 2 2 2 2 2 2

17
25. Which point on the number line represents ?
10
S R W K

0 1 2 3
(a) S (b) R (c) W (d) K
26. Mother Labrador gave birth to 8 puppies. Off these puppies, 1 is black in colour
whereas other 7 are golden coloured. What fraction of the whole group of puppies
is black Labrador. (2011)

1 7 1 8
(a) (b) (c) (d)
7 8 8 7
Fractions and Operation on Fractions 79
27. Below is an aquarium with different types of fishes swimming in it. What fraction
of fishes is NOT star FISHES? (2010)

7 3 4 1
(a) (b) (c) (d)
11 7 11 7
28. If there are 7 apples and 5 oranges in the basket then what fraction of oranges
are there in the fruit basket? (2011)

5 7 7 5
(a) (b) (c) (d)
7 5 12 12
29. P and Q are two points on the following number line. Each of them represents a
fraction. Find their product.

0 P Q 1
14 14 14 13
(a) (b) (c) (d)
81 9 18 9
3
30. If A + 1 = 2, A must be equal to (Tricky, 2008)
4
3 1 1
(a) 1 (b) (c) (d)
4 2 4

31. P and Q are two fractions. When twice of P is subtracted from Q we get 12 . If
3 25
Q is equal to then find the value of P + Q. (Critical Thinking)
4
277 1 200 177
(a) (b) (c) (d)
200 30 277 200
1
32. Which of the following is closest to ?
4
5 6 7 3
(a) (b) (c) (d)
21 19 20 16
EBD_7364
80 Olympiad Champs–Mathematics

1
33. Which of the following is closest to ?
2
5 7 4 3
(a) (b) (c) (d)
11 18 19 11

8 4 6 2 1
34. Simplify ÷ × + – . Choose the correct answer. (Tricky/Critical Thinking)
9 3 8 4 2
1
(a) (b) 1 (c) 2 (d) 3
2
1 1
35. A man sold of his land. He gave of the remaining portion to his son. What
2 2
fraction of the total land is left with him?
1 1 1 1
(a) (b) (c) (d)
2 3 4 5
36. Match the decimal numbers with their corresponding fraction representation.
(Tricky)
List I List II

A. 43.6964 1. 436964
100000
B. 4369.64 2. 436964
1000
C. 4.36964 3. 436964
10000
D. 436.964 4. 436964
100

A B C D
(a) 3 4 1 2
(b) 3 1 2 4
(c) 1 2 3 4
(d) 1 3 2 4
37. Look at the given figure. Which fraction represents the shaded part of the
figure? (2011)

3 3 5 5
(a) (b) (c) (d)
8 5 8 3
Fractions and Operation on Fractions 81
3 1
38. Which of the following drawing shows 3 × 2
= (Tricky)
4 4

(a) =

(b) =

(c) =

(d) None of these

1
39. Which of the following drawing shows 2 × (2015)
5

(a) (b)

(c) (d)

40. Consider the following statements. Which of the following statement(s) is/are true
or false? (Critical Thinking)

2 5 8 7 7
(A) Among , , , fraction is the smallest fraction.
3 6 9 12 12

2 4 5 6 4 1
(B) Among , , , fraction is closest to .
17 17 17 17 17 4
(C) A proper fraction can not be changed into mixed fraction.

(D) Value of equivalent fractions are same.


(a)
TTFF (b)
TFTF (c)
TTTT (d)
FFFF
EBD_7364
82 Olympiad Champs–Mathematics
Directions (Q. 41) Answer the following question by studying the following grid.

W (Critical Thinking)
B
O
W W Y

R R W Y
G
R R G Y

41. What fraction of the large square is Red (R), Blue (B), Orange (O), Green (G),
White (W) and Yellow (Y)? Choose the correct answer.
1 1 1 3 3 3
(a) Red: , Blue: , Orange: , Green: , White: , Yellow:
4 16 16 16 16 16

4 1 1 3 3 3
(b) Red: , Blue: , Orange: , Green: , White: , Yellow:
4 16 16 32 16 16

1 1 1 3 3 3
(c) Red: , Blue: , Orange: , Green: , White: , Yellow:
4 16 16 16 16 16

1 1 1 3 3 3
(d) Red: , Blue: , Orange: , Green: , White: , Yellow:
4 16 32 32 16 16

2
42. The reciprocal of 1 is (2017)
3 3 3 5 3
(a) (b) 1 (c) (d)
2 2 3 5
43. What is the missing sign in the box? (2014)
1 1 1 1 1 1
4 + 9 −3 6 −8 +5
5 5 5 4 4 4

(a) < (b) =


(c) > (d) Can’t be determinend
44. Divide ` 35 into 100 equal parts. Give a decimal fraction that represents each part.
(a) ` 3.5 (b)
` 0.35 (c) ` 0.035 (d)
` 0.0035
2
45. How many minutes are there in of an hour? (Tricky)
3

(a) 40 minutes (b) 60 minutes (c) 20 minutes (d) 100 minutes


46. Anita works for 12 hours and sleeps for 6 hours. What fraction of the day does
Anita sleep? (2016)
6 12 6 18
(a) (b) (c) (d)
12 24 24 24
Fractions and Operation on Fractions 83
47. Fill the table with the fraction of the shaded portion of the figures by choosing the
correct option. One has been done for you. (Critical Thinking)

Input Output Input Output

4

5

— —

1 3 3 1 3 4
(a) , , (b) , ,
2 8 5 2 7 5
1 3 4 3 4
(c) , , (d) 1, ,
2 9 5 9 5

48. A thread of 10 m is cut into 25 equal parts. What decimal fraction represents the
lengths of each part of the thread? (Tricky)
(a) 0.2 m (b) 0.3 m (c) 0.4 m (d) 0.5 m
3
49. A cookie factory uses of a bag of flour in each bag of cookies. The factory used
8
3
of a bag of flour yesterday. How many bags of cookies did the factory made
4
yesterday? (2011, Critical Thinking)
1 3
(a) 1 (b) 2 (c) (d)
2 2

50. Consider the following statements. (Critical Thinking)


6 7
Statement A: < .
13 15
4 16
Statement B: = .
25 100
Choose the correct option.
(a) Only statement A is true. (b) Only statement B is true.
(c) Both A and B are true. (d) Both A and B are false.
51. Consider the following statements. [Tricky]
1
Statement A: of ` 10 is 200 paisa.
5
1
Statement B: There are 50 cm in th part of 3 meters.
6
Choose the correct option.
(a) Only statement A is true. (b) Only statement B is true.
(c) Both A and B are true. (d) Both A and B are false.
EBD_7364
84 Olympiad Champs–Mathematics
52. Which of the following statement(s) is true or false. (2015, Tricky)
1
Statement A: Tuesday is day of a week.
7

1
Statement B: March is month of a quarter year.
12

2
Statement C: April and July are month of a year.
12

2
Statement D: 14th and 15th are days of February.
28

(a) TFFT (b) TFTT (c) TTFF (d) TFTF

7
53. The price of a metre of cloth is ` 12 1 . What would be the price of 5 metre of
2 15
cloth? (2013)
1 1 1 1
(a) ` 59 (b) ` 70 (c) ` 68 (d) ` 45
15 5 3 3
54. Given below a quantities of ingredients to make a Greek salad. Read it and find how
many more cups of olives than cucumber are needed? (Critical Thinking)

Greek salad Greek salad

2
2 cups lettuce of a cup tomatoes
3

2
1 cup red onions of a cup feta cheese
3

15 5
cups olives of a teaspoon salt
8 8

1 1
of a cup cucumber 1 teaspoons pepper
2 2

11 11 12 1
(a) (b) (c) (d)
8 5 3 8

55. A coat costs ` 40. Which is a bigger discount offer for the coat?
1 3
(a) off the normal price (b) off the normal price
4 10

2 1
(c) off the normal price (d) off the normal price
10 2
Fractions and Operation on Fractions 85
56. Which of the following statement(s) is true or false? (Tricky)
1 5 1 3
(i) is equivalent to (ii) is greater than
2 10 4 4
1
(iii) 1 is less than (iv) Decimal fraction is 3 is 0.75
4 4
(a) TFFT (b) TTFT (c) FTTT (d) FTFT
Directions (Qs. 57 and 58) : Read the figures given below and answer the following questions.

57. i. ii. iii. iv.


v. vi . vii. viii.

ix.

1
Which of the above figure(s) represents fraction (Critical Thinking)
2

(a) i, vi, viii (b) i, iii, viii, iv, ix (c) (d) i, v, ix


1
58. Which of the above figure(s) represents fraction
5

(a) iii, v, vii (b) iv, v, vii (c) ii, v, vii (d) i, v, ix
2 1 7
59. What is the value of + + (2017)
8 3 3
10 35 30 10
(a) (b) (c) (d)
3 12 3 27
60. 2 > 3
5 8
(a) True (b) False (c) Partially true (d) None of these
5
61. How many minutes in of an hour? (2014)
3
(a) 40 minutes (b) 60 minutes (c) 20 minutes (d) 100 minutes
62. One half is same as two quarters?
(a) True (b) False (c) Partially true (d) None of these
2
63. The fraction that is equivalent to on the 12th parts number line is
3

7 8 12 6
(a) (b) (c) (d)
12 12 12 12
EBD_7364
86 Olympiad Champs–Mathematics
Directions (64 and 65) : The three number lines divided into 12th parts, 3rd parts, and 24th
parts respectively.

0 1

0 1

0 1
2
64. The fraction that is equivalent to on the 24th parts number line is
3
7 16 8 6
(a) (b) (c) (d)
24 24 24 24
1 5
65. A vessel had 5 L of milk. A cat drink L from vessel. How much milk was left
4 8
in the vessel? (2015)
1 5 5 5
(a) 1 L (b) 3 L (c) 1 L (d) 4 L
8 8 8 8
66. Replace the symbol ‘?” in the table below by choosing correct option to become
equivalent fraction. The rule is to have the denominator as 100. (Tricky)

Input Output

1 ?
2 100
3 ?
10 100
2 ?
10 100
3 ?
4 100

(a) 50, 30, 20, 75 (b) 5, 1, 2, 75 (c) 4, 2, 1, 5 (d) 50, 20, 10, 75

67. Which statement is true or false? (2014, Critical Thinking)


1 3 10 9
Statement A: The like fractions for and are and .
3 10 30 30
2 2
Statement B: is smaller than .
11 5
3 9
Statement C: and are equivalent fractions.
10 10
2 1
Statement D: and are equivalent fractions.
4 2
(a) TTFT (b) FTTF (c) FTFT (d) TFTT
Fractions and Operation on Fractions 87
3 7
68. The health club advises every child to drink 2 L of water in the morning, 1 L
4 8
1
in the afternoon and L before going to bed. How much water should a child drink
2
in whole day? (2013)
1 1 1 1
(a) 5 L (b) 5 L (c) 4 L (d) 6 L
8 7 8 3
69. Compare the fractions given in both the column. Fill the boxes by choosing the
correct operator. (>, <, =)
Fraction Operator Fraction

1 40
2 100

6 42
10 100

7 75
10 100

3 75
4 100

(a) >, >, <, = (b) <, >, <, = (c) >, <, <, = (d) >, >, =, <
1 4
70. The order from the smallest to biggest fraction for the fractions , ,
2 5
3
is (2014, Tricky)
8

3 1 4 1 4 3
(a) , , (b) , ,
8 2 5 2 5 8

4 1 3 3 4 1
(c) , , (d) , ,
5 2 8 8 5 2

71. Choose the correct option for the given expression to be hold true.
3 6 ? ?
= = =
5 ? 20 25

(a) 10, 12, 15 (b) 2, 4, 5 (c) 10, 20, 20 (d) 20, 10, 20

72. In a multi-storey building, 1 of the 546 residents watch the Sports channel. Find
6
the number of people who do not watch this channel. (2014)
(a) 436 (b) 455 (c) 544 (d) 380
EBD_7364
88 Olympiad Champs–Mathematics
73. Fill the output column in the table with the simplified fractions by choosing the
correct option. One has been done for you. (Tricky)

Input Output Input Output Input Output

9 1 18 48
— —
18 2 45 80
12 8
— —
18 32
2 2 1 3 2 3 1 2
(a) , , , (b) , , ,
3 5 4 5 4 5 4 5

1 3 1 2 1 3 4 2
(c)
, , , (d) , , ,
2 5 4 5 2 5 5 5
2
74. Mrs. Sen makes of her garden into a vegetable garden. She plants tomatoes
1 3
in of the vegetable garden. What fraction of the garden does she use to grow
4 (2015)
tomatoes?
1 1 1 2
(a) (b) (c) (d)
6 3 2 3
5 3
75. Billy ate pizza and John ate pizzas. How much more pizza did Billy eat than John?
2 4 1 4 1 3
(a) (b) (c) (d)
3 2 4 4
5
76. Mrs. Priya bought 3 litres of milk in the morning. There was litres left in the
8
evening. How much milk was used during the day? (2016)
17 17 23 19
(a) (b) (c) (d)
8 8 8 8

RESPONSE GRID
1. a b c d 2. a b c d 3. a b c d 4. a b c d 5. a b c d
6. a b c d 7. a b c d 8. a b c d 9. a b c d 10. a b c d
11. a b c d 12. a b c d 13. a b c d 14. a b c d 15. a b c d
16. a b c d 17. a b c d 18. a b c d 19. a b c d 20. a b c d
21. a b c d 22. a b c d 23. a b c d 24. a b c d 25. a b c d
26. a b c d 27. a b c d 28. a b c d 29. a b c d 30. a b c d
31. a b c d 32. a b c d 33. a b c d 34. a b c d 35. a b c d
36. a b c d 37. a b c d 38. a b c d 39. a b c d 40. a b c d
41. a b c d 42. a b c d 43. a b c d 44. a b c d 45. a b c d
46. a b c d 47. a b c d 48. a b c d 49. a b c d 50. a b c d
51. a b c d 52. a b c d 53. a b c d 54. a b c d 55. a b c d
56. a b c d 57. a b c d 58. a b c d 59. a b c d 60. a b c d
61. a b c d 62. a b c d 63. a b c d 64. a b c d 65. a b c d
66. a b c d 67. a b c d 68. a b c d 69. a b c d 70. a b c d
71. a b c d 72. a b c d 73. a b c d 74. a b c d 75. a b c d
76. a b c d
Fractions and Operation on Fractions 89

Solutions with Explanation


LEVEL 1
1 2
1. (d) The first three options show equivalent fractions with shaded portions , and
2 4
4 3
. Whereas is not a equivalent fraction among them.
8 5

1 6 1 11
2. (a) of 6 oranges: :: of 11 apples: apple.
5 5 6 6

5 15
3. (c) = * . Multiplying the numerator and denominator by 3, we get . Thus
7 21 21
* is 15.

3 18 3
4. (a) = , Dividing the numerator and the denominator by 6, we get . Thus
* 24 4
* = 4.

32 42 16 21 160 – 147 13
5. (c) =– = – =
70 100 35 50 350 350

5
6. (d) of a whole means 5 portions are shaded of the total 12 portions. This is
12
represented by the following figure.

3 6 9 12
7. (c) , , , Is following a pattern in which the equivalent fractions are being
4 8 12 ?
multiplied in both numerator and denominator by 1,2,3 and 4 respectively. Thus
following the pattern we get? should be equal to 16.

4 6 7 2
8. (d) , , and all have the same denominator. Thus the fraction with the smallest
11 11 11 11
2
numerator will be the smallest fraction. The smallest fraction is .
11
EBD_7364
90 Olympiad Champs–Mathematics
9. (b)

786
10. (a) = 0.786
1000

1478
11. (c) = 0.01478
100000

133
12. (c) = 13.3
10

4 8 8 4 2
13. (c) = . Upon simplifying, and equals to .
10 20 20 10 5

5 1
14. (a) > . Since the denominator is same, the fraction with greater numerator is
6 6
larger.

7 3
15. (a) > , Since the denominator is same, the fraction with greater numerator is
8 8
larger.

8 4
16. (b) < . The denominators are different here, so making the denominators equal,
15 5
8 12
we get and . Now since the denominator is same, the fraction with greater
15 15
numerator is larger.

3 6 6 3
17. (c) = . on simplifying, equals to .
7 14 14 7

5 7
18. (b) < , Since the denominator is same, the fraction with greater numerator is
12 12
larger.

19. (b) There are 0.6 fragments of 1 cm that are shaded.

20. (c) As can be seen the given fractions are following a pattern in which the proceeding
2 4 8 16
fraction is multiplied by 2, thus the next part of this sequence , , ,
77 77 77 77
32
should be .
77
Fractions and Operation on Fractions 91
LEVEL 2
1 4 1
21. (c) As × 4= =
12 12 3
22. (a)
23. (a)
List I List II
1 1
A. of ` 6 × 6 × 100 = 120 paisa = 1 rupee + 20 paisa
5 5
1 1
B. of ` 18 × 18 × 100 = 200 paisa
9 9
1 1
C. of ` 10 × 10 × 100 = 250 paisa = 2 rupee 50 paisa
4 4
1 1
D. of ` 3 × 3 × 100 = 50 paisa
6 6

24. (d) As
1 1 1 1 1 1
5× = + + + +
2 2 2 2 2 2
17 17
25. (c) W represents portion of the number line. means 1 whole part + 0.7 parts,
10 10
which can be represented by the following figure:
S R W K

0 1 2 3

1 7
10
26. (c) Number of puppies born to mother labrador = 8. Number of golden labradors = 7,
Number of black labradors = 1. Total fraction of black labradors = number of black
1
labrador divided by total labradors = .
8

7
27. (a) Since 7 fishes out of 11 fishes are not star fish therefore required fraction = .
11
28. (d) Number of fruits in the fruit basket = 12 (7 apples and 5 oranges).
5
Fraction of oranges = number of oranges/ number of fruits= .
12

2 7 2 7 14
29. (a) P = , Q = , then P.Q = × =
9 9 9 9 81
7 8–7 1
30. (d) =A 2=
– =
4 4 4
EBD_7364
92 Olympiad Champs–Mathematics

12
31. (d) Q – 2P =
25
3 12
⇒ – = 2P
4 25
27
⇒ 2P =
100
27
⇒ P=
200 27 3 177
∴ P + Q= + =
200 4 200
1
32. (a) A fraction is closest to when the denominator is about fourth the numerator.
4
5 1
Here is the option where the fraction is closest to .
21 4
1
33. (a) A fraction is closest to when the denominator is about twice the numerator.
2
5 1
is the option where the fraction is closest to .
Here
11 2
8 3 2 6 2 2 1 1
34. (a) × = × = + = 1 – =
9 4 3 8 4 4 2 2
(By using BODMAS)
1
35. (c) Remaining land = after selling.
2
1
Half of the land he gave to his son =
4
1 1 1
\ Remaining land = – =
2 4 4

36. (a) 37. (a)  3 38. (b) 39. (d)


8
40. (c)
1 1 1 3 3 3
41. (d) Red: , Blue: , Orange: , Green: , White : , Yellow: .
4 16 32 32 16 16

42. (d)

1 1 1 21 + 46–16 51
43. (c) Since 4 + 9 –3= =
5 5 5 5 5
1 1 1 25 –33 + 21 13
and 6 –8 = +5 =
4 4 4 4 4
51 × 4 = 204 and 13 × 5 = 65
51 12
So >
5 4
Fractions and Operation on Fractions 93
35
44. (b) ` 35 is to be divided into 100 equal parts which is equal to = ` 0.35.
100
45. (a) Number of minutes in 1 hour= 60.
2 2
of 1 hour = X 60 = 40 minutes.
3 3

46. (c) Number of hours in a day = 24 , Number of hours for which Anita sleeps = 6,
6
Fraction of hours for which Anita sleeps = .
24
47. (a)
48. (c) A thread of 10 m is divided into 25 parts. In decimal fraction it can be written as
10
= 0.4 m.
25
49. (b) Divide the total amount of flour used by the amount used in each bag.
3 3 3 8
÷ = × = 2. The factory made 2 bags of cookies yesterday.
4 8 4 3

6 7
50. (c) Both statements are true. < . This can be verified by cross multiplication.
13 15
4 16
And and are equivalent fractions.
25 100

51. (c)
Both statements are true. ` 1 = 100 paisa ; ` 10 = 10 × 100 = 1000 paisa.
1 1
th of ` 10 = × 1000 paisa = 200 paisa.
5 5
And 1 meter = 100cm, 3 meter = 300 cm.
1 1
th of 3 meter = X 300 = 50cm.
6 6
1 1
52. (b) March is month of a quarter year is false. March is month of a year.
12 12

5 41
25 82 205 1
53. (c) As × = 68
2 15 3 3 3
15 1
54. (a) There are cups of olives and cup of cucumber in Greek Salad. The recipe has
8 2
15 1 11
- = cups more olives than cucumber.
8 2 8
55. (d) 56. (a) 57. (a) 58. (a)
59. (b) 60. (a) 61. (d) 62. (a)
EBD_7364
94 Olympiad Champs–Mathematics
63. (b) 64. (b)
1
65. (d) The quantity of milk left in the vessel = 5 – 5 = 21 – 5 = 37 = 4 5 L
4 8 4 8 8 8

66. (a) 1 ×=
50 50
;
3 10 30
×
= ;
2 10 20 3 25 75
×
= ; ×=
2 50 100 10 10 100 10 10 100 4 25 100
67. (a)

11 15 1 22 + 15 + 4 41 1
68. (a) Since + + + = = 5
4 8 2 8 8 8
69. (a)
1 4 3
70. (a) Given fractions are , ,
2 5 8
L.C.M. of 2, 5, 8 = 40
1 20 20 4 8 32 3 5 15
∴ ×= , =
× , ×
=
2 20 40 5 8 40 8 5 40
15 20 32
So, order from the smallest to biggest is , ,
40 40 40
3 1 4
i.e., , , .
8 2 5
3 6 3×2 6
71. (a)
=⇒ =
5 ? 5 × 2 10
Now, 6 = ?

6×2
=
12
10 20 10 × 2 20
Now, 3 3 × 5 15
= =
5 5×5 25
1
72. (b) Number of people who watch sports channel = × 546 =91
6
So, number of people who do not watch sport channel = 546 – 91 = 455
73. (a)
74. (a) The required
fraction of the garden that Mrs. Sen uses to grow,
2 1 1
tomatoes =  × =
3 4 6
75. (b)
5 24 –5 19
3–
76. (d) As= =
8 8 8
95
Decimals

CHAPTER FOREWORD
Consider the case mentioned below:
Boys were very happy and were discussing about the victory of Indian cricket team against
Australia last night.

They listed certain important figures of the match:.


Bowling Overs Runs Economy
(1) Ashish Nehra 4 29 7.25

(2) H.H. Pandya 3 29 9.66

(3) R.A. Jadeja 4 22 5.50

It is clear from the above table that R.A. Jadeja bowled best. Representing his economy is
5.50 which involves the use of decimal.
Decimal system allows us to represent numbers of all types. For this purpose, a symbol (•) is
used which is called the decimal point.
473.26 45.9 2 1 7
Decimal Point.
1 1 1 1
10 100 1000 10, 000
After reading this chapter, you will learn about the different uses of decimal and how a
decimal number can be converted into fraction and vice-versa.

Note : If a fraction has 10, 100, 1000, etc in denominator then decimal can be easily put in
the numerator of the fraction a according to number of user in the denominator.
237 237 237
For Example
= 23.7 ,
= 2.37 and = 0.237 and so on
10 100 1000
EBD_7364
96 Olympiad Champs–Mathematics

Chapter
6 Decimals
Real-Life Examples

v Decimals are used in expressing


money, distance and length,
weight and capacity.
v Decimals are frequently used in LEARNING OBJECTIVES
sciencefrom laboratory experimental
This lesson will help you to:—
data.
v Decimals are used when adding v understand the expanded form of a decimal
and counting money. Whenever number.
we have some number of paise v learn the conversion of decimal into fraction & vice-
that do not add upto a complete versa.
rupee, we express the amount v identify the different types of decimals.
as a decimal.
v compare decimals.
v Decimals are used in all types of
v learn about the different operations on the decimal
measurements.
numbers.
For example: When you fall
sick, doctor prescribes you
medicine as 2.5 ml twice a day
QUICK CONCEPT REVIEW
or so on. (i) Decimal number is another way of representing fractions.
Units Tenths
Tens

Try It! 45 .6
25
Example: Express Decimal Point
1000
and 325 as decimals  Decimal is a fraction having the denominator
100 power of 10.
Example: 325 = 0.025 and  Decimal part read as separately one by one like
100 25.921 is read as twenty five point nine, two,
one.
325 = 3.225
 Decimal numbers have a whole part and a decimal
100
part separated by a decimal point.
Decimals 97
(ii) The decimal point goes between units and tenths
place. Try It!
1
6 Example: Express as a
45 .6 = 40 + 5 +
10
decimal 8

Decimal Number Solution : 1


8

=
1×125 125
= = 0.125
8 ×125 1000
(iii) Place of a decimal: In a decimal number, position or
“place” of each digit is important.
In the number 237,
 the “7” is in the Units position, meaning just 7 Amazing
Amazing Facts
Facts
 the “3” is in the Tens position meaning 3 tens
v One decimal place to the right
 and the “2” is in the Hundreds position, meaning of the decimal point is the
2 hundreds. “tenths” place, but one decimal
Units (1s) place to the left of the decimal
Tens (10s) point is the “ones” place. The
Hundreds (100s) “tens” place is two places to the
left.
v Decimal notation is the writing

237 of numbers in a base-10 numeral


system.
v The word decimal is derived
100s 10s 1s from the Latin root decem
(ten).
10x Bigger
“Two Hundred Thirty Seven”
(iv) As we move left, each position is 10 times bigger.
Hundreds are 10 times bigger than Tens.
 As we move right, each position is 10 times
smaller. From Hundreds, to Tens, to Units Try It!
Example: Convert as a decimal
237 328 49
(a) 425 (b) (c)
100s 10s 1s ? 10 100 1000

10x Smaller Solution: (a) 425 = 42.5


(v) Expanded form of decimals 10
1 6 2 328
315.162= 300 + 10 + 5 + + + (b) = 3.28
10 100 1000 100
49
(vi) Like and unlike decimals : Like decimals have same (c) = 0.049
decimal places unlike decimals have different decimal 1000
places.
EBD_7364
98 Olympiad Champs–Mathematics
Example: 2.56, 5.48, 0.25 etc., are like decimals and
0.2, 1.23, 2.236 etc., are unlike decimals.
(vii) Equivalent decimals: The decimals which have same
Try It!
value are called equivalent decimals.
Example: Add 2.3 and (viii) Decimals on a number line: Each decimal number
4.625 can be plotted on the number line using units, tenths,
hundredths etc.
Solution: 2.300

+ 4.625 –0.4 –0.2 0 0.2 0.4 0.6 0.8 1 1.2
–0.5 –0.3 –0.1 0.1 0.3 0.5 0.7 0.9 1.1

6.925
(ix) Decimal fractions: A decimal number can also be
represented as fraction.
A decimal fraction is a fraction where the denominator
is a number such as (10, 100, 1000 etc). i.e. a power
of tens.
Historical
Amazing Facts
Preview Following decimal number have following fraction
v Decimal fractions were first = 23 1376 .
2.3 = , 13.76
developed and used by the 10 100
Chinese in the 1st century BC, Conversion of a Decimal into a Fraction
and then spread to the Middle Step 1: Remove the point from the decimal and write the
East and from there to Europe. obtained number as the numerator.
v The Jewish mathematician Step 2: Write 1 as denominator and put zeroes right to
Immanuel Bonfils invented it so that the number of zeroes is equal to the number of
decimal fractions around 1350, digits right to the point in the given decimal.
anticipating Simon Stevin, but
2356
did not develop any notation to Example : 23.56 = .
represent them. 100
Comparing Two Decimal Numbers
v We can compare two like decimals just as we compare
two whole numbers ignoring the decimal point.
v For comparing two unlike decimals, first convert
Try It!
them into like decimals then compare.
Example: Subtract 4.23
from 16.984 Addition of Decimals
Solution : 16.984 Step 1: Convert the given decimals into like decimals.
– 4.230 Step 2: Write the addends one under the other so that

12.754 the decimal points of all the addends are in the same
column.
Example: Multiply 12.82 Step 3: Add as in the case of whole numbers.
by 10
Step 4: In the sum, put decimal point directly under the
Solution: 12.82 × 10 = 128.2
decimal points in the addends.
In multiplication with 10,
100, 1000, etc, the decimal Subtraction of Decimals
shift to the right depending Step 1: Convert the given decimals into like decimals.
on no. users .
Step 2: Write the smaller number under the larger one
so that their decimal points are in the same column.
99
Decimals
Step 3: Subtract as in the case of whole numbers.
Step 4: In the difference, put the decimal point directly
under the decimal points of the given numbers.
Try It!
Multiplication of a Decimal by a Whole Number
Step 1: Multiply the decimal without the decimal point Example : Multiply 12.5 by 6.2
by the whole number. Solution : 125
Step 2: Place the decimal point so as to obtain as many × 62
decimal places in the product as there are in the decimal. 250
7500
Multiplication of Two Decimals 7750
Step 1: Multiply the two decimals without the decimal
points, just like whole numbers. Now there are 2 places, so
Step 2: In the product, place the decimal point so that decimal comes before 5
the number of decimal places in the product is equal to So 12.5 × 6.2 = 77.50
the sum of the decimal places in the given decimals.

Division of a Decimal by a Whole Number


Step 1: Perform the division by considering the dividend
Misconcept/Concept
a whole number.
Step 2: When the whole number part of the dividend Misconcept: Longer is larger
is complete, put the decimal point in the quotient and misconcept: A longer decimal
proceed with the division as in case of whole numbers. may not be a larger number
than a shorter decimal.
Division of a Decimal by a Decimal
Concept: Shorter is larger misconcept
Step 1: Convert the divisor into a whole number by
: A shorter decimal is a larger
multiplying the dividend and the divisor by 10, 100 or
number than a longer decimal.
1000 etc. depending upon the number of decimal places
in the divisor.
Step 2: Now, divide the new dividend by the whole
number as discussed above.

Rounding Decimal
Try It!
Rounding means reducing the digits in a number while
trying to keep its value similar. Example: Divide 256.2 by
Example: 73 rounded to the nearest ten is 70, because 100
73 is closer to 70 than to 80. Solution: 256.2
= 2.562
Rules to round numbers 100
v Decide which is the last digit to keep. In divisor decimal shifts
to left depending on zeroes
v Leave it the same if the next digit is less than 5
in denominator.
(this is called rounding down).
v But increase it by 1 if the next digit is 5 or more
(this is called rounding up).
EBD_7364
100 Olympiad Champs–Mathematics
Rounding Decimals
Examples Because ...
3.1416 rounded to ... the next digit (1) is less
hundredths is 3.14 than 5

Try It! 1.2635 rounded to ... the next digit (6) is 5 or


tenths is 1.3 more
Example: Convert as decimal 1.2635 rounded to 3 ... the next digit (5) is 5 or
(a)  750 paise into rupees decimal places is 1.264 more
(b)  6520 ml into liter Rounding whole numbers
(c)  525 grams into Kg .
Examples Because ...
(d)  425 cm into m.
134.9 rounded to tens ... the next digit (4) is less
is 130 than 5
Solution :
12,690 rounded to ... the next digit (6) is 5 or
(a)  750 = `7.50 thousands is 13,000 more
100 1.239 rounded to units ... the next digit (2) is less
6250 is 1 than 5
(b)  = 6.250 L
1000 Rounding to significant digits
525
(c)  = 0.525 Kg Examples Because ..
1000
1.239 rounded to 3 ... the next digit (9) is 5 or
425 significant digits is 1.24 more
(d)  = 4.25 m
100 134.9 rounded to 1 ... the next digit (3) is less
significant digit is 100 than 5
0.0165 rounded to 2 ... the next digit (5) is 5 or
significant digits is more
0.017

PLAY TIME
Divide the students in two teams. Assign the numbers
(0-9) to both the teams. Ask two teams to make the
largest and smallest decimal numbers with one decimal
place and two decimal places with 1, 2, 3 digits as
whole number part. The team who makes the maximum
correct number wins the game..

USES OF DECIMAL SYSTEM


We have seen that 100 cm = 1 m
1000 m = 1 km, 1000 ml = 1l, 1000g = 1 kg etc.
In money also, we use the conversion 100 paise = 1 rupee.
We can use the decimal system in all these places to
convert a smaller unit to a bigger unit.
101
Decimals

Multiple Choice Questions


LEVEL 1
1. Find the decimal number for “one and four tenths”.
(a) 14.0 (b) 1.4 (c) 0.14 (d) 140.0
2. Find the decimal number for “Sixty Seven and Twenty Three One-Hundredths”.
(a) 67.23 (b) 6.723 (c) 0.6723 (d) 6723.0
3. Find the decimal number for “Twenty Three Fiftieths”.
(a) 0.23 (b) 0.46 (c) 2. 3 (d) 4.6
4. Find the decimal number for “Seven Twenty Fifths”.
(a) 0.7 (b) 7.0 (c) 0.28 (d) 0.7
1
5. Find the decimal number for “ 2 “. (Mental Mathematics)
2
(a) 21 (b) 2 (c) 2.5 (d) 0.25
6. Find the decimal number for “12.78” rounded to the nearest tenth.
(Mental Mathematics)
(a) 12.8 (b) 12.7 (c) 12.79 (d) 13.0
7. Find the decimal number for “3.8439” rounded to the nearest hundredth.
(a) 3.84 (b) 3.844 (c) 3.843 (d) 3.85
8. Round 12.6257 to three decimal places. (Mental Mathematics)
(a) 12.6 (b) 12.626 (c) 12.625 (d) 12.63
9. Round 106.5 to the nearest whole number. (Mental Mathematics)
(a) 100 (b) 106 (c) 107 (d) 110
10. Find the round 239.456 to the nearest hundred. (Mental Mathematics)
(a) 200 (b) 300 (c) 239.45 (d) 239.46
11. Diana sells 12 garlands for ` 12.12. What is the cost of each garland?
(a) 1.01 (b) 1.1 (c) 1.0 (d) 0.11
12. If 58 out of 100 students in a school are boys, then write a decimal for the part
of school that consists of boys.
(a) 0.50 (b) 0.58 (c) 58 (d) 59
12
13. 502 if expressed as decimals can be written as: (Mental Mathematics)
100
(a) 502.012 (b) 502.12 (c) 50.212 (d) 5021.2
EBD_7364
102 Olympiad Champs–Mathematics
14. 9.471 if expressed as fraction can be written as: (Mental Mathematics)
471 471 471 471
(a) (b) 9 9
(c) (d) 900
1000 1000 100 10
15. 929.5 ÷ 100 is (Mental Mathematics)
(a)
92.95 (b)
929.5 (c)
929.05 (d)
9.295
16. 2.1143 × 10 is (Mental Mathematics)
(a) 0.21143 (b) 21.143 (c) 2114.3 (d) 2.11430
2 2 2 2
17. 2000 + 200 + 20 + 2 + + + + =
10 100 1000 10000
(a)
222222.22 (b)
2222.2222 (c)
2222.2000 (d)
22222.222
18. 500.5 × __________ = 5005 (Mental Mathematics)
(a) 10 (b) 100 (c) 1000 (d) 10000
19. 727.111 ÷ __________ = 7.27111 (Mental Mathematics)
(a) 10 (b) 100 (c) 1000 (d) 10000
395
20. 3.95 __________ (Mental Mathematics)
100
(a) > (b) < (c) = (d) None
21. Rounding off 9.19 to one decimal place will give

(a)
9.20 (b)
9.00 (c)
9.10 (d)
10.00
22. Rewrite the following in the decimal form of the given unit 8335 g into kg. (2013)
(a) 8.335 kg (b) 83.35 kg (c) 833.5 kg (d) 0.8335 kg
23. Rewrite the following in the decimal form of the given unit 790 m into km. (Tricky)
(a) 79 km (b) 7.90 km (c) 7900 km (d) 0.79 km
24. Rewrite the following in the decimal form of the given unit 92 mm into cm.
(a) 0.92 cm (b) 9.2 cm (c) 920 cm (d) 0.092 cm
25. Rewrite the following in the decimal form of the given unit 7 ml into litre.
(a) 0.007 litre (b) 0.07 litre (c) 0.7 litre (d) 70 litre
26. Rewrite the following in the decimal form of the given unit 29 cm into mm.
(a) 2.90 mm (b) 0.29 mm (c) 290 mm (d) 29000 mm
27. The difference of 325.312 and 253.175 is:
(a) 72.003 (b) 72.137 (c) 72.130 (d) 72.107
28. The product of 982.311 and 2.43 is:
(a) 2387.0133 (b) 2387.01000 (c) 2387.0573 (d) 2387.01573
29. Dividing 145.208 by 0.25.
(a)
580.832 (b)
58.0832 (c)
5808.32 (d)
588.32
103
Decimals
30. Which of the following expressions is true? (2012)
(a) 2.3749 > 2.0349 (b) 2.03749 < 2.003749
(c) 2.3749 > 2.4749 (d) None of these
31. Find the greatest decimal from the following decimals (2008, Tricky)

0.206, 2.06, 0.0206, 0.26.

(a) 2.06 (b) 0.206 (c) 0.0206 (d) 0.26

32. Which of the following decimal numbers is the least? (2010)


1.305, 1.35, 1.053, 1.53

(a) 1.305 (b) 1.35 (c) 1.053 (d) 1.53


33. The value of 9.85 + 0.6 – 0.28 is _______. (2013)
(a) 10.73 (b) 10.37 (c) 10.1 7 (d) 9.17
34. Find (2015)
× Add 36.21 to the
product of 71.2 and 6

×
(a) 390.99 (b) 436.41 (c) 427.20 (d) 463.41
35. Smallest number in the row 31.21, 31.12, 311.2, 3.112 is

(a)
31.21 (b)
31.12 (c)
311.2 (d)
3.112
36. Highest number in the row 9.03, 9.33, 9.39, 9.09 is

(a)
9.03 (b)
9.33 (c)
9.39 (d)
9.09
37. Which of the following holds true or false? (Critical Thinking)
5 1
(A) 0.5 = 0.5000.
. (B)
(C) 0.5 = 5 × 0.5 =
.(D) 0.5 = 5.0.
100 1000

(a)
TFFF (b)
FTFF (c) FFTF (d) FFFT
38. Which of the following holds true or false?
(A) 0.3 + 0.7 = 10.00. (B) 0.30 + 0.70 = 1.00.
(C) 0.30 + 0.70 = 0.10. (D) 0.3 + 0.7 = 1.10.
(a) TFTT (b) FTFF (c) TFFF (d) FFTT
39. If a drum hold 18.75 L of diesel, how many litres of diesel can 7 such drums
hold? (2013)
(a) 130.89 L (b) 131.25 L (c) 128.25 L (d) 130.75 L
EBD_7364
104 Olympiad Champs–Mathematics

1
40. A ticket costs ` 12.60. How much shall I pay for 3 tickets? (2013)
2
(a) ` 40.80 (b)
` 44 (c)
` 44.10 (d)
` 34.80

LEVEL 2

41. The area of a swimming pool is 225.28 sq. metres. Its breadth is 8.8 metres. Find
the length (2013)
(a) 25.8 m (b) 25.6 m (c) 35.2 m (d) 40.1 m
42. A baker buys 5 kg of flour and 3.5 kg of sugar for the cake and uses 2.25 kg of
flour and 1.75 kg of sugar in the cake. What is the amount of flour and sugar left
with him.
(a) 2.75 kg, 0.75 kg (b) 2.25 kg, 1.75 kg
(c) 2.25 kg, 0.75 kg (d) 2.75 kg, 1.75 kg

43. The rainfall in a city an first 5 days of a month was 1.27 cm, 3.25 cm, 2.79 cm,
2.57 cm and 1.37 cm. How much did it rain altogether? (2009)

(a) 11.25 cm (b) 9.98 cm (c) 11.00 cm (d) 11.98 cm

44. A tailor takes 2.5 m of cloth for making a curtain. He received an order of making
25 curtains from Mrs. Radha puri. How much cloth will he require to fulfill the
order?

(a) 62.50 m (b) 65 m (c) 51 m (d) 625.50 m

45. Divya spends 1.23 hours for English reading, 1.40 hours for Math and 0.39 hours
for Science. How many hours does she spend on studying? (2011)

(a) 3.02 (b) 3.0 (c) 302.0 (d) 0.32

46. Anshi bought cosmetic items which cost ` 78.60 in total. She gave ` 100 to the
shop keeper. How much does she receive as change?

(a) 21 (b) 21.40 (c) 20.40 (d) 214.0

47. Riya scored 56.73 points and Asha scored 74.92 points in a University exam. How
many points less did Riya score than Asha? (2012)

(a) 1.8 (b) 18.19 (c) 18.10 (d) 18.00

48. A broken scale is used to measure the height of the plant. The length of the broken
scale is 12 cm. The height of the plant is 10.50 times greater than the broken
scale. What is the height of the plant?


(a)
126.0 (b)
125.5 (c)
126.55 (d)
1.2650
105
Decimals
49. Fill the output column in the table below with the equivalent fraction by choosing
the correct option. (Tricky)

Input Output
0.28 ?
14.50 ?
07.80 ?
75.75 ?
1 4 3 3
(a) 7/25, 14 , 7 , 75 (b) 28/100, 14, 7.8, 75
2 5 4 4
1
(c) 7/25, 14 , 7, 75 (d) 7, 14, 7, 75
2
50. Fill the output column in the table below by rounding the given decimal number till
3 digits. (2009)

Input Output
3.14159 ?
134.9 ?
1.239 ?
0.0165 ?
(a) 3.14, 134, 1.239,0 (b) 3.14, 135, 1.24, 0.017
(c) 3.14, 134, 1.2, 0.016 (d) 3.14, 135, 1.239, 0.016
51. The sum 0.25 + 0.375 = 0.625 can be expressed using fractions as: (Tricky)
(a) 1/4 + 3/8 = 5/8 (b) 1/4 + 3/8 = 4/12
(c) 1/4 + 3/16 = 5/16 (d) 1/25 + 3/75 = 6/25
52. Ekta has ` 90. She wants to buy the following items:
A DVD player for ` 49.95, a DVD holder for ` 19.95 and a personal stereo for
` 21.95. How many items Ekta can buy with the money she has?
(a) All three (b) Any two (c) Any One (d) None
53. If A = 2.5, B = 4.5, C = 1.25, then value of 7A + 5B is (2010)
(a) 22.5 (b) 17.5 (c) 40 (d) 39.8
54. Study the table of distances.
From
A B C
To
P 40.18 km 24.5 km 6.8 km
Q 52.32 km 19.7 km 11.8 km
R 7.9 km 10.12 km 5.6 km

Which of the following distance is the greatest?


(a) A to R (b) A to P (c) C to R (d) B to Q
EBD_7364
106 Olympiad Champs–Mathematics
55. The width of two skateboards are shown in the table. Which number sentence
correctly compares the width of these two skateboards? (2012)

Skateboards widths
Skateboard Width in inches
X 8.125
1
Y 8
8

1 1 1
(a) 8.125 < 8 (b) 8.125 = 8 (c) 8.125 > 8 (d) None of these
8 8 8
56. Consider the following statements and choose the correct option. (Tricky)

Statement A : When a decimal number is multiplied by 10, the decimal point moves
to the right by one place.

Statement B : When a decimal number is multiplied by 1000, the decimal point


moves to the right by three places.

(a) Statement A is false and B is true.

(b) Both statements are true.

(c) Statement A is true and B is false.

(d) Both statements are false.

57. Which of the following is correct? (Critical Thinking)

(a) to divide a decimal number by 10, shift the decimal point to the left by one place.

(b) to divide by 100, shift the decimal point to the left by two places.

(c) to divide by 1000, shift the decimal point to the left by three places.

(d) All above are true.

58. Which one of the following digit will come in the extreme right to the sum

4.235 + 56.230 + 45 + 2.02 + 0.02316


(a)
6 (b)
8 (c)
3 (d)
0

59. Let P = 256.32, Q = 456.321 and R = 95.326. When Rahul adds two of the given
decimals and subtracts third one from the sum he finds 295.327. Which one of the
following is the correct expression for Rahul’s work?

(a) P + Q – R (b) P + R – Q (c) Q + R – P (d) P – R + Q


107
Decimals
60. Given a step by step procedure of subtraction of two decimal numbers through a
chart. Complete it by choosing the correct option. (Critical Thinking)

8 hundredth – 4 hundredth = 4 hundredth


83 . 58
– 71 . 34 5 tenths – 3 tenths = 2 tenths
12 . 24
The Decimal point

8 tens – 7 tens = 1 ten



(a) 3 hundredth – 1 hundredth = 2 hundredth
(b) 3 tenths – 1 tenths = 2 tenths
(c) 3 ones – 1 one = 2 ones
(d) cannot find
61. Varun purchased a book for ` 18.45, a pair of shoes for ` 70.25 and a shirt for `
55.75. How much money did Varun spend in all? (2014)
(a) ` 144.35 (b) ` 154.53 (c) ` 144.45 (d) ` 114.55
62. My father got 3.7 litres of petrol filled in his car. The price of petrol per litre is
` 21. How much money did my father pay at the petrol pump? (2015)
(a) ` 77.70 (b) ` 67.60 (c) ` 65.50 (d) ` 79.40
63. 6 friends go out to eat pizza they order a 2 medium pizza’s of `329 and `429
respectively. How much amount each must contribute to pay the bill if they share
it equally. Round it off to one decimal place. (2015)
(a) `126.30 (b) `54.80 (c) `71.50 (d) `126.33
64. Krish and Kavya are playing a game. Each win gives 0.5 points and each loss takes
away 0.25 points. If Krish stands at a score of 3 and Kavya at 3.5, who will win
if Krish wins 3 out of the next 5 games? (2014)
(a) Krish (b) Kavya
(c) Draw (d) Can’t be calculated
Directions (Qs. 65 to 69): In a multiple choice test of 50 questions each correct answer
gives you 1 mark, each incorrect answer reduces the score by one-fourth of a mark. Any
unattempted question do not get you any mark.
65. What will be the score of Kapil who attempts 38 and has 28 correct answers?

(a)
30.5 (b)
25.5 (c)
3 (d)
31
66. Which of the following is/are correct? (Critical Thinking)

Attempted Incorrect Score


A. Shruti 40 10 27.5
B. Tripti 48 24 18.0

C. Vidhi 36 4 31.5

(a) Only A (b) Only B (c) A and B (d) B and C


EBD_7364
108 Olympiad Champs–Mathematics
67. What should be the most advisable strategy in such a test? (Tricky)
(a) Attempt maximum number of questions.
(b) Attempt all the questions irrespective of you know them.
(c) Avoid questions whose answers you are not sure off.
(d) None of these.
68. Which of the following is correct to obtain a score of 43.75?
(a) Attempt 50 correct 45 (b) Attempt 45 correct 40
(c) Attempt 45 correct 41 (d) Attempt 46 correct 42
69. A shopkeeper mixed 3.6 kg of hazelnuts with 0.756 kg of raisins. He packed the
mixture equally into 5 boxes. What is the weight of each box? (2010)
(a) 14.25 kg (b) 4.35 kg (c) 0.87 kg (d) 0.72 kg
70. Neha weighs 35.4 kg and Manju weighs 42.9 kg. If Ashish, Neha and Manju weigh
117 kg altogether, then what is Ashish’s weight? (2016)
(a) 38 kg (b) 38.9 kg (c) 38.7 kg (d) 38.6 kg
71. Reema went for shopping. She hired an auto to cover the distance of 5.5 km. If
auto fair is `20 for the first 1 km distance and `6.50 for further each 1 km, then
how much amount she gave to the autodriver?
(a) 26.50 (b) 40.25 (c) 49.25 (d) 48
72. Which of the following decimal numbers can represent shaded portion in the given
figure? (Tricky)

(a) 0.50 (b) 4.51 (c) 0.5625 (d) 0.5605


73. Which of the following decimal numbers can represent shaded portion in the given
figure? (2011)


(a)
0.50 (b)
0.75 (c)
1.0 (d)
0.625
74. The cost of 1 metre of wire is `5.50. Find the cost of 42.4 m of wire.
(a) ` 210.00 (b)
` 2332.00 (c)
` 23.32 (d) ` 233.20
109
Decimals
75. A tailor used 62.78 metres of red ribbon, 93.22 metres of blue ribbon, 59.10
metres of yellow ribbon in a dress. What is the total length of ribbon used by the
tailor in the dress? If the cost of ribbon is 2.50/metre, what is the cost of ribbon
used in the dress?

(a) 215.10 m, ` 537.75 (b) 215.22 m, ` 537.00

(c) 215.22 m, ` 537.50 (d) 215.10 m, ` 537.25

76. A cake weighing 3.5 kg was divided among 40 guests in a birthday party. What will
be the share of each person?

(a) 0.875 kg (b) 0.0875 kg (c) 8.75 kg (d) 0.00875 kg

77. A shopkeeper buys 30 toy cars of `112.12 each and gets a discount of `0.50 on
each car. How much total money he is required to pay?

(a) ` 3363.60 (b)


` 3360.00 (c)
` 3348.60 (d) ` 3378.60

78. Electricity bill of Mr. Mohan Das shows the following details:–

Monthly Rent = 500/– (Critical Thinking)

Unit Consumed Rate (`)

1 – 100 3.50

101-150 4.25

151-200 4.50

greater then 201 4.75

(i) Calculate amount payable by Mr. Mohan Das to BSES Rajdhani Power Limited if
he consumes 120 units.

(a) ` 935 (b)


` 875 (c)
` 395 (d)
` 500

(ii) Calculate amount payable by Mr. Mohan Das to BSES Rajdhani Power Limited if
he consumes 150 units.

(a) ` 1063 (b)


` 1062.5 (c)
` 1061 (d)
` 1060

79. Which of the following holds true or false? (Critical Thinking)

1
(A) `5 and 25 paise = `5.025. (B) of a rupee = `100.
100
1
(C) of a rupee = 0.25 paise. (D) 20 paise = `0.20.
50

(a)
TFFF (b)
FFTF (c)
TTFF (d)
FFFT
EBD_7364
110 Olympiad Champs–Mathematics
Directions (Qs. 80 to 85) : Complete the paragraph for a number 94.58.

94 is ______(80) the decimal point and ______(81) is after the decimal point. 94 is a
whole number, it is called the ______(82) part. The digits after the ______(83) are not
taken together. We do not say point fifty-eight, we say point ______(84). This is the
decimal part or ______(85).
80. (a) equal (b) after (c) before (d) none
81.
(a)
58 (b)
94 (c)
45 (d)
5
82. (a) fractional part (b) whole part
(c) natural part (d) greater part
83. (a) fractional part (b) decimal point
(c) whole part (d) None
84. (a) five (b) eight (c) five and eight (d) None
85. (a) fractional part (b) fractional point
(c) decimal point (d) integral part

RESPONSE GRID
1. a b c d 2. a b c d 3. a b c d 4. a b c d 5. a b c d
6. a b c d 7. a b c d 8. a b c d 9. a b c d 10. a b c d
11. a b c d 12. a b c d 13. a b c d 14. a b c d 15. a b c d
16. a b c d 17. a b c d 18. a b c d 19. a b c d 20. a b c d
21. a b c d 22. a b c d 23. a b c d 24. a b c d 25. a b c d
26. a b c d 27. a b c d 28. a b c d 29. a b c d 30. a b c d
31. a b c d 32. a b c d 33. a b c d 34. a b c d 35. a b c d
36. a b c d 37. a b c d 38. a b c d 39. a b c d 40. a b c d
41. a b c d 42. a b c d 43. a b c d 44. a b c d 45. a b c d
46. a b c d 47. a b c d 48. a b c d 49. a b c d 50. a b c d
51. a b c d 52. a b c d 53. a b c d 54. a b c d 55. a b c d
56. a b c d 57. a b c d 58. a b c d 59. a b c d 60. a b c d
61. a b c d 62. a b c d 63. a b c d 64. a b c d 65. a b c d
66. a b c d 67. a b c d 68. a b c d 69. a b c d 70. a b c d
71. a b c d 72. a b c d 73. a b c d 74. a b c d 75. a b c d
76. a b c d 77. a b c d 78. a b c d 79. a b c d 80. a b c d
81. a b c d 82. a b c d 83. a b c d 84. a b c d 85. a b c d
111
Decimals

Solutions with Explanation


LEVEL 1
1. (b) 1.4
one unit and four tenths = 1 + 4/10 = 1 + 0.4 = 1.4.
2. (a) Sixty Seven and Twenty Three One-Hundredths
= 67 + 23/100 = 67 + 0.23 = 67.23.
23 23 × 2 46
3. (b) Twenty Three Fiftieths
= = = = 0.46
50 50 × 2 100
7 7×4 28
4. (c) Seven Twenty Fifths
= = = = 0.28 .
25 25 × 4 100
1 1×5 5
5. (c) 2 =2 + =2 + =2 + 0.5 =2.5 .
2 2×5 10
6. (a) 12.78 rounded to nearest tenth = 12.8.
Since the digit on hundredth place 8 > 5, the tenth place is rounded by adding 1 to 7.
7. (a) 3.8439 rounded to nearest hundredth = 3.84.
Since the digit on thousandth place 3 < 5, the hundredth place remains unchanged.
8. (b) 12.6257 to three decimal places = 12.626.
Since the digit after 5 is 7 and 7 > 5, the thousandth place is increased by 1.
9. (c) 106.5 rounded to nearest whole number = 107.
Since the digit after decimal is 5, the unit place is rounded by adding 1 to 6.
10. (a) 200
“2” needs to be retained in the hundreds position. The next digit is “3” which is
less than 5, so no change is needed to “2”.
∴ Required answer = 2 hundreds.
11. (a) 12.12 ÷ 12 = 1.01
cost of each garland = ` 1.01.
12. (b) A fraction and a decimal for the part of the school that consists of boys are

fraction decimal
5005 0.58
10

12 12
13. (b) 502 = 502 + = 502 + .12 = 502.12
100 100
EBD_7364
112 Olympiad Champs–Mathematics

471 471
14. (b) 9.471 = 9 + .471 = 9 + = 9
1000 1000
929.5 9295 9295
15. (d) = = = 9.295
100 100 × 10 1000
21143 21143
16. (b) 2.1143 × 10 = × 10 = = 21.143
10000 1000
17. (b) 2222 + .2 + .02 + .002 + .0002 = 2222.2222
5005
18. (a) Required number = × 10 = 5005
10
727.111 727111 100000
19. (b) = × =100
7.27111 1000 727111

395
20. (c) 3.95 =
100

395
So, 3.95 =
100
21. (a)
22. (a) 1 kg = 1000 g
8335
\  kg = 8.335 kg.
1000
23. (d) 1 km = 1000 m
790
\  km = .79 km.
1000
24. (b) 1 cm = 10 mm
92
\ 92 mm = cm = 9.2 cm
10
25. (a) 1l = 1000 ml
7
\  7 ml = = 0.007 l.
1000
26. (c) 1 cm = 10 mm
\ 29 cm = 29 × 10 = 290 mm
27. (b) 325.312 – 253.175 = 72.137
982311 243
28. (d) 982.311 × 2.43 = × = 2387.01573
1000 100

29. (a) 145.208 145208 100


= ×
0.25 1000 25
145208
= = 580.832
250
113
Decimals
30. (a) 2.3749 > 2.0349 is correct.
31. (a) Write each of the decimals in a place-value table, and fill in the empty spaces with
zeros:

Units Decimal point Tenths Hundredths Thousandths


0 . 2 0 6
2 . 0 6 0
0 . 0 2 0
0 . 2 6 0
By comparing the four digit numbers, 2.060 = 2.06 is the greatest decimal.
32. (c) Write each of the decimals in a place-value table, and fill in the empty spaces with
zeros:

Units Decimal point Tenths Hundredths Thousandths


1 . 3 0 5
1 . 3 5 0
1 . 0 5 3
1 . 5 3 0
By comparing the four digit numbers, 1.053 is the least.
33. (c) 9.85 + 0.6 – 0.28 = 10.45 – 0.28 = 10.17
34. (d) 36.21 + (71.2 × 6) = 36.21 + 427.2 = 463.41
35. (d) 36. (c)
37. (a) Only (A) is true.
38. (b)
39. (b) 18.75 × 7 = 131. 25 l
40. (c) 12.60 × 3.5 = `44.10

LEVEL 2
41. (b) 225.28 ÷ 8.8 = 25.6 m
42. (d) Amount of flour left = (5.00 – 2.25) kg = 2.75 kg.
Amount of sugar left = (3.50 – 1.75) kg = 1.75 kg.
43. (a) (1.27 + 3.25 + 2.79 + 2.57 + 1.37) cm = 11.25 cm rain.
44. (a) Required cloth = (2.5 × 25) m = 62.50 m.
45. (a) 1.23
1.40
+ 0.39
3.02
She spends 3 hours 2 minutes on her studying.
EBD_7364
114 Olympiad Champs–Mathematics
46. (b) 100.00
— 78.60
21.40
She receives ` 21.40.
47. (b) 74.92
— 56.73
18.19
Riya scored 18.19 points less than Asha.

48. (a) 10.50 × 12 = 126.00 = 126 cm

7 1 4 3
49. (a) , 14 , 7 , 75
25 2 5 4
50. (b) 3.14, 135, 1.24, 0.017

51. (a) Express each of the decimals as fractions:

25 5 1
0.25
= = =
100 20 4

375 3
0.375
= =
1000 8

625 5
0.625
= =
1000 8

52. (b) Sum of amount of all the items = 49.95 + 19.95 + 21.95 = 91.85 > 90.
Only two items can be bought with ` 90 as sum of all three items cost more
than ` 90.
53. (c) 7A = 17.5
5B = 22.5
7A + 5B = 40
54. (b) A to R = 7.9 km
A to P = 40.18 km
C to R = 5.6 km
B to Q = 19.7 km
1
55. (b) 8.125 = 8 is correct.
8
56. (b) 57. (d)
58. (a) 4.235 + 56.230 + 45 + 2.02 + 0.02316 = 107.50816
59. (c) Q + R – P = 456.321 + 95.326 – 256.32 = 295.327
115
Decimals
60. (c)
61. (c) `18.45 + `70.25 + `55.75 = `144.45
62. (a) 3.7 × `21 = `77.70
63. (a) Total amount = `329 + `429 = `758.
Amount contribute by each = `758 ÷ 6 = `126.33 = `126.30.
64. (a) Krish = 3 + 0.5 + 0.5 + 0.5 – 0.25 – 0.25 = 4
Kavya = 3.5 + 0.5 + 0.5 – 0.25 – 0.25 – 0.25 = 3.75
\  Krish will win the game.
1
65. (b) Kapil’s score = 28 × 1 – (38 – 28) × = 28 - 2.5 = 25.5
4
1
66. (c) Shruti’s score = (40 – 10) – 10 × = 30 – 2.5 = 27.5
4
1
Tripti’s score = (48 – 24) – 24 × = 24 – 6 = 18
4
1
Vidhi’s score = (36 – 4) – 4 × = 32 – 1 = 31
4
67. (c)

68. (a) 50 Attempt

45 Correct × 1 ⇒ 45.00
–5 incorrect × .25 ⇒ 1.25

  43.75
3.6 + 0.75 4.35
69. (c) Weight of each box = kg = = 0.87 kg.
5 5
70. (c) Ashish's weight = [117 – (35.4 + 42.9)] kg = 38.7 kg.
71. (c) Total distance = 5.5 km
Amount for first 1 km = `20
Remaining distance = 5.5 – 1 = 4.5 km
Now, amount for 4.5 km = 4.5 × 6.50 = `29.25
Hence, total amount = ` 20 + ` 29.25 = `49.25
72. (c) Total number of squares = 8.
1
Number of shaded squares = 4 = 4.5
4.5 2
\  Required answer = = 0.5625
8
73. (d) Total parts = 16, number of shaded parts = 10.
10 5
\  Required answer = = = 0.625.
16 8
EBD_7364
116 Olympiad Champs–Mathematics
74. (d) Cost of 1 m wire = ` 5.50.
Cost of 42.4 m wire = `(5.50 × 42.4) = ` 233.20.
75. (a) Total length = (62.78 + 93.22 + 59.10) m = 215.10 m
Cost of 1 m ribbon = ` 2.50.
\ Cost of 215.10 m ribbon = 215.10 × ` 2.50 = ` 537.75.
76. (b) Share of each person = 3.5 ÷ 40 = 0.0875 kg.
77. (c) Actual cost of 1 car after discount = ` 112.12 – ` 0.50 = ` 111.62

\  Actual cost of 30 cars = ` 111.62 × 30 = ` 3348.60

78. (i) (a) 500 + 100 × 3.50 + 20 × 4.25

= 500 + 350 + 85 = 935

(ii) (b) 500 + 100 × 3.50 + 50 × 4.25 = 1062.5

79. (d)

80. (c) before

81. (a) 58

82. (b) whole part

83. (b) decimal point

84. (c)

85. (a) Decimal part is also known as fractional part.


Bodmas Application 117

CHAPTER FOREWORD
Earlier we have learnt about mathematical operations,

i.e., addition (+), subtraction (–), division (÷) and multiplication (×).

You know how to solve problems involving single operation and two digits like 128 ÷ 2, 4893 +
208 etc. But if there is a case like,
7 + 9 ÷ 4 + 3 (92 × 2)
then what do we do first? Addition or Multiplication?

These types of problems arise when we solve questions involving two or more than two
mathematical operations.
BODMAS rule is the solution of such problems.
BODMAS stands for :
B → Brackets
O → Of
D → Division
M → Multiplication
A → Addition
S → Subtraction
Try answering the following question.
Do you know what is 'PEMDAS'?
After reading this chapter, you will learn how BODMAS rule can be applied in mathematical
problems.

Note : We will learn it all +, – , ×, ÷ , come together in an expression, then which operation is
to be solved first according & BOD MAS. So first division, then multiplication, then addition
and substraction in the end
For example: 8 × 9 ÷ 3 + 5 – 4
=8×3+5–4
= 24 + 5 – 4
= 29 – 4 = 25
EBD_7364
118 Olympiad Champs–Mathematics

Chapter
7 Bodmas
Application
LEARNING OBJECTIVES
Real-Life Example This lesson will help you to:—

v The rules of BODMAS are very v learn about the concept of BODMAS.
important in daily accounting v study about the application of BODMAS in
and calculations and are mathematics.
used frequently by bankers,
v study about the importance of BODMAS rule while
accountants, students and even
solving mathematical problems.
housewives.
.
QUICK CONCEPT REVIEW
Lesson in a Nutshell

BODMAS: BODMAS is the sequence for working out


and constructing mathematical equations and formulas
containing more than one calculation. This methodology
is commonly referred to as the order of mathematical
operations.
Operations: "Operations" in mathematics refer to
Amazing Fact
Facts addition, subtraction, multiplication, division, etc. If it
isn't a number it is probably an operation.
v As in India we often hear about
BODMAS, in USA the acronym B RACKETS ( ) [ ] { }

PEMDAS is used. The full form


of PEMDAS is 'Parentheses,
O RDER POWER OF ( )2

Exponents, Multiplication, Division, D IVIDE /


Addition and Subtraction'.
M ULTIPLY * x

A DDITION +

S UBTRACTION

v Divide and Multiply rank equally (and go left to


right).
v Add and Subtract rank equally (and go left to
right).
Bodmas Application 119
1. 2. 3. 4.

D A
BO or or
Try It!
MS Example : Simplify
v After you have done "B" and "O", just go from left 8 ×7 + 5 ×6 – 3 ×2
to right doing any "D" or "M" as you find them.
Solution : 8×7+5×6–3×2
v Then go from left to right doing any "A" or "S" as
you find them. = 56 + 30 – 6
= 86 – 6
B rackets Do first
= 80
O rders of
Example : Simplify 8 × 9 ÷ 3 + 3
D ivision
M ultiplication Solution : 8×9÷3+3
A ddition =8×3+3
S ubtraction Do last = 24 + 3 = 27

Steps to simplify the order of operations using


BODMAS rule:
v First part of an equation is start solving inside the
'Brackets'.
For Example; (6 + 4) × 5
First solve inside 'brackets' 6 + 4 = 10, then
10 × 5 = 50. Historical
Amazing Preview
Facts
v Next solve the mathematical 'Of'.
v Since the introduction of modern
For Example; 3 of 4 + 9
algebraic notation, multiplication
First solve 'of' 3 × 4 = 12, then 12 + 9 = 21. has taken precedence over
v Next, the part of the equation is to calculate addition. Thus 3 + 4 × 5 = 4 × 5
'Division' and 'Multiplication'. + 3 = 23. When exponents were
We know that, when division and multiplication follow first introduced in the 16th and
one another, then their order in that part of the 17th centuries, exponents took
equation is solved from left side to right side. precedence over both addition and
multiplication and could be placed
For Example; 15 ÷ 3 × 1 ÷ 5
only as a superscript to the right
'Multiplication' and 'Division' perform equally, of their base. Thus 3 + 25 = 28
so calculate from left to right side. First solve and 3 × 25 = 75. To change the
15 ÷ 3 = 5, then 5 × 1 = 5, then 5 ÷ 5 = 1. order of operations, originally a
v The last part of the equation is to calculate vinculum (an overline or underline)
'Addition' and 'Subtraction'. was used. Today, parentheses or
We know that, when addition and subtraction follow brackets are used to explicitly
one another, then their order in that part of the denote precedence by grouping
equation is solved from left side to right side. parts of an expression that should
be evaluated first.
For Example; 7 + 19 - 11 + 13
'Addition' and 'Subtraction' perform equally, so
calculate from left to right side. First solve 7 + 19 = 26,
then 26 - 11 = 15 and then 15 + 13 = 28.
EBD_7364
120 Olympiad Champs–Mathematics
These are simple rules need to be followed for
Misconcept/Concept simplifying or calculating using BODMAS rule.
Rules of BODMAS are same with integers, decimals
Misconcept: Some questions, for and fractions.
example are written as 6 ÷ 2(1 + 2).
There is no sign between 2 and (. RULES FOR INTEGERS (SIGNED NUMBERS)
This is often misinterpreted as
division. ADDITION SUBTRACTION
Concept: The absence of sign and and
indicates a multiplication. So, this and and
expression is actually - 6 ÷ 2*
(1 + 2). The BODMAS rule says -
First Brackets MULTIPLICATION AND DIVISION
Then Of and and
Then Division and Multiplication in and and
the order they appear from left
to right. • If the number from which the subtraction will be
Then Addition and Subtraction in done is greater than the number to be subtracted,
the order they appear from left then the result is positive.
to right.
• If the number which will be subtracted is more than
Thus note that BODMAS rule the number from which the subtraction will be done,
does not say that division comes the result is negative.
before multiplication. It also
Example 1 : Simplify
does not say that Addition comes
before subtraction. {2 + 3 (4 + 5) ÷ 3 – 1}
In the above example, first we will Solution : = {2 + 3(9) ÷ 3 – 1}
solve the brackets : 1 + 2 = 3. So = {2 + 3 × 3 – 1}
now the equation gets converted
= 2 + 9 – 1 = 11 – 1 = 10
to 6 ÷ 2*3. Now the division and
multiplication will be evaluated Example 2 : Simplify
from left to right. So 6 ÷ 2 = 3 9 ÷ 3 – 12 ÷ 2 + 6 × 3
and 3*3 = 9. The correct answer Solution : 9 ÷ 3 – 12 ÷ 2 + 6 × 3
is 9.
= 3 – 6 + 18
= 21 – 6 = 15
Shortcut to Problem Solving:

B ored
O f
D oing
M aths
A t
S chool
Bodmas Application 121

Multiple Choice Questions


LEVEL 1
1. In BODMAS, M stands for
(a) Mathematics (b) Multiples
(c) Multiplication (d) Model
2. Evaluate 1 + 2 x 3 + 4 -5 (Mental Mathematics)

(a)
5 (b)
9 (c)
6 (d)
8
3. Pick the odd one out. (Mental Mathematics)
(a) 8 ÷ 4 x 2 (b) 5 x 1 – 1 (c) 3 x 3 – 5 (d) 3 x 2 + 1
4. Pick the odd one out. (Mental Mathematics)
(a) 6 x 4 + 2 (b) 3 x 10 - 3 (c) 5 x 5 + 2 (d) 9 x 4 - 9
5. Evaluate (3 + 6) × 2 (Mental Mathematics)
(a) 9 (b) 15 (c) 18 (d) 12

6. Work out 9 × (12 - 8) (Mental Mathematics)


(a) 35 (b) 36 (c) 37 (d) 32

Directions (Qs. 7 to 13) : Fill in the blanks, such as to make the statements below true.
7. 36 ÷________ + 4 = 10 (Mental Mathematics)

(a)
5 (b)
4 (c)
9 (d)
6
8. 24 ÷ 4 - _____ = 5

(a)
0 (b)
8 (c)
4 (d)
1
9. 23 + 28 ÷ ____ = 30

(a)
4 (b)
5 (c)
7 (d)
6
10. 23 × 3 + _____= 90
(a) 26 (b) 21 (c) 25 (d) 24
11. 45 - 38 +____= 21

(a)
15 (b)
14 (c)
17 (d)
16
12. 56 ÷ 4 × ____= 56

(a)
14 (b)
16 (c)
4 (d)
8
13. ____ - 2 X 14 = 4

(a)
30 (b)
35 (c)
38 (d)
32
EBD_7364
122 Olympiad Champs–Mathematics
10
14. Evaluate –3 (Mental Mathematics)
2
(a) 2 (b) 10 (c) 9 (d) 8
15. Evaluate 2 x 4 + 4 x 2 + 1 (2008)
(a) 17 (b) 33 (c) 32 (d) 37
16. Calculate: 50 - 5 x 5 ?
(a) 20 (b) 125 (c) 75 (d) 25
17. What is the value of 3 + 6 ÷ 3 × 2 ?
(a) 9 (b) 5 (c) 7 (d) 3
18. 22 × 12 ÷ 12 – 10 + 12 = ________.
(a) 34 (b) 14 (c) 24 (d) 16
19. 7386 + 3333 – ________ = 10010
(a) 619 (b) 609 (c) 719 (d) 709
20. The value of 100 – 7 × 8 + 4 ÷ 2 is (2014)
(a)
78 (b)
46 (c)
92 (d)
96
21. 100 – 7 × 1 + 5 = ________.
(a) (100 – 7) × (1 + 5) (b) (100 – 7) × 1 + 5
(c) 100 – 7 × (1 + 5) (d) 100 – (7 × 1) + 5
22. The value of 244 + 8 × 4 – 318 ÷ 6 = ________. (2015)
(a)
203 (b)
223 (c)
329 (d)
429
23. What is the solution to the given expression? (2011)
3 × 10 + (9 × 2) =
(a) 48 (b) 78 (c)
84 (d) 114
(6 +15×2)
24. Evaluate: (2009)
6
(a) 8 (b) 9 (c) 6 (d) 7
25. 16 ÷ (4 ÷ 4) is equal to
1
(a) 4 (b) 1 (c) (d) 16
4
26. 16 ÷ 8 + 4 × 2 = ________

(a)
2 (b)
8 (c)
10 (d)
12
27. 30 × 8 ÷ 2 + 62 – 24 = ________ (Critical Thinking)
(a) 168 (b) 158 (c) 185 (d) 142
28. 60 × 12 ÷ 6 + 12 = ________
(a) 142 (b) 122 (c) 132 (d) 102
29. Evaluate 5 + 2 x 7 (2011)
(a) 15 (b) 19 (c) 17 (d) 16
Bodmas Application 123
30. Evaluate 1 + 2 x 3 + 3 x 4 (2008)
(a) 16 (b) 19 (c)
108 (d)
48
Questions (Qs. 31 to 40) : Evaluate the following questions using the rules of BODMAS.
31. (10 + 5) ÷ 3 + 2 × 8 (2009)
(a) 35 (b) 25 (c) 22 (d) 21
32. (3 × 8 + 20 - 7) ÷ 2
(a) 18.5 (b) 33 (c) 35.5 (d) 36.2
33. 15 - (6 + 6 × 6) ÷ (9 + 5) (2008)
(a) 18 (b) 15 (c) 12 (d) 21
34. (300 ÷ 15 - 10) × 1

(a)
9 (b)
2 (c)
8 (d)
16
35. 9 + 5 × 3 - 9 × 3 + 4 × (2) (2011)
(a) 5 (b) 6 (c) 8 (d) 10
36. (3 + 3 - 5) × (15 - 5) × 10 - 99

(a)
2 (b)
6 (c)
8 (d)
1
37. 3 × 2 + 8 ÷ 4 (2010)

(a)
7 (b)
8 (c)
6 (d)
9
38. 16 ÷ 4 + 24 ÷ 6

(a)
7 (b)
9 (c)
6 (d)
8
39. 4(5 x 3 - 10)
(a) 15 (b) 20 (c) 25 (d) 23
40. 30 ÷ 2 - 18 ÷ 3 (2012)

(a)
9 (b)
6 (c)
8 (d)
12

LEVEL 2
41. Match the following:

List I List II
A. ( ), [ ], { } 1. Solved second
B. Square roots and under roots 2. Solved third
C. +, – 3. Solved first
D. ÷, × 4. Solved last
A B C D
(a) 2 1 4 3
(b) 1 2 3 4
(c) 3 1 4 2
(d) 4 1 2 3
EBD_7364
124 Olympiad Champs–Mathematics
42. Read the statements and choose the correct option.
Statement A : 10 ÷ 2 = 2 ÷ 10.
Statement B : 12 + 8 ÷ 2 = 10.
(a) Both statements are true. (b) Both statements are false.
(c) Only A is true. (d) Only B is true.
43. Match the following: (Tricky)
List I List II
A. 6x3-6 1. 3 x 7- 5
B. 3x3-9 2. 4 x 9- 1
C. 2x9-2 3. 5 x 6 - 30
D. 8x5-5 4. 5x4-8
A B C D
(a) 4 3 1 2
(b) 3 1 2 4
(c) 1 2 3 4
(d) 4 3 2 1
5 5
44. Find 5 + - (2008)
5 5
(a) 5 (b) 8 (c) 7 (d) Math error
(9929-9917)
45. What is the value of the following numerical:
(12-8)
(a) 5 (b) 8 (c) 4 (d) 3
46. Hubert thinks that 5 + 2 × 3 = 21. What can you add to the calculation to make
the left hand side equal 21?
(a) 9 (b) 10 (c) 11 (d) 12
47. What is the value of 5 × 3 - 12 ÷ 4 + 8. (2011)
(a) 3 (b) 4 (c) 14 (d) 20
Directions (Qs. 48 to 51) : Solve the following questions by filing up the grid.
(Critical Thinking)
1. 2. 3.
3 A × 6 = 33

+ × C

12 5 3

÷ D +

3 10 5

= × =

B 2 23

5
Bodmas Application 125
48. (Across 1). 3 + A x 6 = 33
(a) 7 (b) 9 (c) 5 (d) 2
49. (Down 1) 3 + 12/3 = B
(a) 6 (b) 7 (c) 8 (d) 5
50. (Down 2) A x 5 D 5 x 4 = 5
(a) + (b) - (c) x (d) ÷
51. (Down 3) 6 C 3 + 5 = 23
(a) + (b) - (c) x (d) ÷
52. Read the statements and choose the correct option. (Tricky)
Statement A : 3 + 12 ÷ 4 = 6.
Statement B : 6 ÷ 2 + 3 = 6.
(a) Only A is true. (b) Only B is true.
(c) Both A and B are true. (d) Both A and B are false.
53. 44 – 6 + 43 × 2 – 87 = ?
(a) 37 (b) 38 (c) 73 (d) 83
5 2 2 6
54. The simplest form of × + ÷ is (2012, Tricky)
6 9 9 7

(a)
0.44 (b)
0.32 (c)
0.49 (d)
0.36
55. The value of 50 + [100 – {35 – (30 – 20)}] × 2 is
(a) 0 (b) 50 (c) 100 (d) 200
3 3 3
56. ÷ × 2 + is equal to
2 2 2
7 2
(a) 2 (b) 7 (c) (d)
2 7
57. Evaluate (4 – 3) x (4)

(a)
7 (b)
4 (c)
6 (d)
8
58. Evaluate (7 + 3) x (3 + 4)

(a)
28 (b)
70 (c)
49 (d)
26
59. Evaluate 3 x 4 - 2(5 - 2)
(a) 5 (b) 12 (c) 9 (d) 6
60. Find 15 - (99 - 93)
(a) 5 (b) 7 (c) 9 (d) 6
61. What is the value of 600 + 1200 ÷ 6 × 2 + 400 × 2? (2012)
(a) 1400 (b) 1500 (c) 1800 (d) 2000
(11 + A)
62. = 5. What number is A. (Tricky)
(97 – 94)
(a) 4 (b) 5 (c) 9 (d) 8
EBD_7364
126 Olympiad Champs–Mathematics
63. Find two whole numbers that make this BODMAS sum work: ? × (3+ ?) = 14


(a)
2,4 (b)
4,2 (c)
2,3 (d)
3,4

64. Calculate following: 145 + (120 ÷ 20 + 6) ? (2017)

(a) 175 (b) 150 (c) 157 (d) 169

65. What is the value of 5 × 4 - 2 × 3 + 16 ÷ 4.

(a) 18 (b) 34 (c) 20 (d) 11(1/2)

66. Carry out the operation as written, (9 + 7) 4 × 5 = ?

187 16 4
(a) (b) (c) 20 (d)
7 20 5
1 7  7 1
67. Simplify : + ÷  ×1  (Critical Thinking)
3 9  10 4

7 2 17 4
1 (c)
(a) (b) 1
(d)
9 9 9 9
68. 11.2 – 0.12 × 1.2 + 1.2 ÷ 0.2 = ________ (Tricky)

(a) 17.56 (b) 17.056 (c) 17.560 (d) 17.65

69. Find the missing value in the empty box. (2014)

1 1
2.25 ÷ 9 +1 - (6.5 × 0.2) =×
8 4
(a) 0.075 (b) 0.3 (c) 1.3 (d) 1.375
1 5 9 1
70. 3
+2 × ×2 ÷2 =________. (Critical Thinking)
2 7 26 2
491 4 491
(a) (b) (c) 4 (d) none of these
728 728 728

71. 0.33 ÷ 3 × 33 + 0.33 – 3.3 = ________.

(a) 0.003 (b) 3.03 (c) 0.66 (d) 1.2

72. 100 ÷ 200 × 300 + 400 – 500 = ________. (Tricky)


(a) 150 (b) 100 (c) 50 (d) none of these
1 5 7 1
73. 3 +2 × – ÷2 =________. (Tricky)
2 7 19 2
1 1 3
(a) 4 (b)
4 (c)
4 (d) none of these
4 2 4
Bodmas Application 127
74. Which of the following step is wrong for the given below mathematical sentence?
37091 – 72 ÷ 3 × 687 + 5973
Step I : 37091 – 24 × 687 + 5973 (Divide)
Step II : 37091 – 16388 + 5973 (Multiply)
Step III : 43064 – 16488 (Add)
Step IV : 26386 (2017)
(a) Step I only (b) Step III and IV
(c) Step II only (d) Step II and IV
75. I thought of a decimal number. After I had multiplied it by 2, then added 1.5 and
then divided the result by 3, I got 3.5. What was the decimal number I thought?
(2015)
(a) 7.1 (b) 5.6 (c) 5.8 (d) 4.5

RESPONSE GRID
1. a b c d 2. a b c d 3. a b c d 4. a b c d 5. a b c d
6. a b c d 7. a b c d 8. a b c d 9. a b c d 10. a b c d
11. a b c d 12. a b c d 13. a b c d 14. a b c d 15. a b c d
16. a b c d 17. a b c d 18. a b c d 19. a b c d 20. a b c d
21. a b c d 22. a b c d 23. a b c d 24. a b c d 25. a b c d
26. a b c d 27. a b c d 28. a b c d 29. a b c d 30. a b c d
31. a b c d 32. a b c d 33. a b c d 34. a b c d 35. a b c d
36. a b c d 37. a b c d 38. a b c d 39. a b c d 40. a b c d
41. a b c d 42. a b c d 43. a b c d 44. a b c d 45. a b c d
46. a b c d 47. a b c d 48. a b c d 49. a b c d 50. a b c d
51. a b c d 52. a b c d 53. a b c d 54. a b c d 55. a b c d
56. a b c d 57. a b c d 58. a b c d 59. a b c d 60. a b c d
61. a b c d 62. a b c d 63. a b c d 64. a b c d 65. a b c d
66. a b c d 67. a b c d 68. a b c d 69. a b c d 70. a b c d
71. a b c d 72. a b c d 73. a b c d 74. a b c d 75. a b c d
EBD_7364
128 Olympiad Champs–Mathematics

Solutions with Explanation


LEVEL 1
1. (c) In BODMAS, M stands for Multiplication.
2. (c) 1 + 2 x 3 + 4 - 5 = 1 + 6 + 4 - 5 = 11 - 5 = 6
3. (d) 3 x 2 + 1 is the odd one out as it equals to 7. Whereas all other options i.e.
8/4 x 2 ; 5 x 1- 1; 3 x 3 - 5 equals to 4 when solved using the BODMAS rule.
4. (a) 6 x 4 + 2 is the odd one out as it equals to 24+2=26. Whereas all other options i.e.
3 x 10 - 3; 9 x 4 - 9; 5 x 5 + 2 equals to 27 when solved using the BODMAS rule.
5. (c) Brackets first:
First (3 + 6) = 9, then 9 × 2 = 18
6. (b) 9 × (12 - 8) = 9 × 4 = 36
7. (d) 36 ÷________ + 4 = 10, As can be seen, we need to get the solution equal to 10 by
adding a number in 4. Now we all know, we get 10 by adding 6 to 4. And to obtain 6,
we must divide 36 by 6. Thus, the equation becomes 36 ÷ 6 + 4 = 10.
8. (d) 24 ÷ 4 - _____ = 5. Solving the equation, we get, 24 ÷ 4 = 6, Thus 6 -__ = 5. To
get 5 as the answer, we must subtract 1 from 6. Thus the missing number is 1 and
the equation becomes 24 ÷ 4 -1 = 5.
9. (a) 23 + 28 ÷ ____ = 30. In the equation, we need to divide a number by 28 and then
add it to 23 to obtain the result equal to 30. Now in order to get 30, we need to
add 7 to 23, which can be obtained by dividing 28 by 4. Thus the equation should
be 23 + 28 ÷ 4 = 30.
10. (b) 23 × 3 + _____= 90, Solving the equation, we have 69 + __ = 90. Now to equate this
equation we must add 21. Thus the new equation becomes 23 ×3 + 21 = 90.
11. (b) 45 - 38 +____= 21, Solving the equation, we get 7 + ___ = 21. Now, to equate the
equation we must add 14. Thus the new equation becomes 45 - 38 + 14 = 21.
12. (c) 56 ÷ 4 × ____= 56, Solving the equation, we get 14 × __ = 56. Now to equate the
equation we must multiply it by 4.
Thus the new equation becomes 56 ÷ 4 × 4 = 56.
13. (d) ____ - 2 × 14 = 4. Applying BODMAS and simplifying the equation, we get, ___ - 28
= 4. Now to equate the equation, we must subtract 28 from 32. Thus the number
is 32. And the equation is 32 - 2 × 14 = 4.

10 10
14. (a) -3 . Applying BODMAS, we will first divide 2 and then subtract 3 from the
2
10
result. Thus = 5 and 5 - 3 = 2.
2
15. (a) 2 x 4 + 4 x 2 + 1; Applying BODMAS, and solving multiplication first, we get

8 + 8 + 1 = 17.
Bodmas Application 129
16. (d) 50 - 5 x 5 ⇒ 50 - 25 = 25
17. (c) 3+6÷3×2⇒3+2×2⇒3+4=7
18. (c) 22 × 1 – 10 + 12 = 34 – 10 = 24
19. (d) (7386 + 3333) – 10010 = 709
20. (b) 100 – 7 × 8 + 4 ÷ 2 = 100 – 56 + 2 = 44 + 2 = 46
21. (d)
22. (b) 244 + 8 × 4 – 318 ÷ 6 = 244 + 32 – 53
= 276 – 53 = 223

23. (a) 3 × 10 + (9 × 2) = 30 + 18 = 48
24. (c) (6 + 15 x 2) / 6; (6 + 30)/6; 36/6 = 6
25. (d) 16 ÷ (4 ÷ 4) = 16 ÷ 1 = 16

26. (c) 16 ÷ 8 + 4 × 2

= 2 + 4 × 2 = 8 + 2 = 10

27. (b) 30 × 8 ÷ 2 + 62 – 24 = 30 × 4 + 62 – 24 = 120 + 62 – 24 = 182 – 24 = 158

28. (c) 60 × 2 + 12 = 120 + 12 = 132

29. (b) 5 + 2 x 7 = 5 + 14 = 19

30. (b) 1 + 2 x 3 + 3 x 4; Applying BODMAS, and solving multiplication first, we get

1 + 6 + 12 = 19.

31. (d) 15/3 + 2 × 8 ; 5 + 2 × 8 ; 5 + 16 = 21

32. (a) (24 + 20 - 7)/2 = (44 - 7)/2 = 37/2 = 18.5

33. (c) 15 - (6 + 36)/(9 + 5) ; 15 - 42/(9 + 5); 15 - 42/14; 15 - 3 = 12

34. (b) (20 - 10) × 1 = 2 × 1 = 2.

35. (a) 9 + 5 × 3 – 9 × 3 + 4 × 2 = 9 + 15 – 27 + 8 = 5

36. (d) (6 - 5) × (15 - 5) × 10 - 99 = 1 × (15 - 5) × 10 - 99 = 1 × 10 × 10 - 99

= 10 × 10 - 99 = 100 - 99 = 1

37. (b) 3 × 2 + 8 ÷ 4 = 3 × 2 + 2 = 6 + 2 = 8

38. (d) 16 ÷ 4 + 24 ÷ 6 = 4 + 4 = 8

39. (b) 4(5 x 3 - 10) = 4(15 - 10) = 4 × 5 = 20


EBD_7364
130 Olympiad Champs–Mathematics
40. (a) 30 ÷ 2 - 18 ÷ 3 = 15 - 6 = 9
1. 2. 3.
3 5 × 6 = 33

+ × x

12 5 3

÷ – +

3 10 5

= × =

7 2 23

LEVEL 2

41. (c) Applying the rules of BODMAS.

42. (b) Both the statements are false. A: 10/ 2 = 5 whereas 2/10 = 16 which are not equal.
Whereas B = 12 + 8 /2 = 12 + 4 = 16 and not 10.

43. (a)

List I List II
6 x 3 - 6 = 12 5 x 4 - 8 = 12
3x3-9=0 5 x 6 - 30 = 0
2 x 9 - 2 = 16 3 x 7 - 5 = 16
8 x 5 - 5 = 35 4 x 9 - 1 = 35

44. (a) 5 + 5/5 - 5/5 ⇒ 5 + 1 - 1 ⇒ 5 + 0 = 5

45. (d) (9929 - 9917)/(12 - 8) = 12/4 = 3

46. (b) Hubert thinks that 5 + 2 × 3 = 21, But, 5 + 2 × 3 = 11. Thus we need to add 10 to
make the left hand side equal to 21.

47. (d) 5 × 3 - 12 ÷ 4 + 8 = 15 - 3 + 8 = 12 + 8 = 20

48. (c) 3 + A x 6 = 33; 3 + 6A = 33; 6A = 33-3; 6A = 30; A = 30/6=5

49. (b) 3 + 12/3 = B; 3 + 4 = 7; Thus B = 7.


Bodmas Application 131
50. (b) A x 5 D 5 x 4 = 5. From above question, we know A = 5. Thus the equation becomes
25 D 20 = 5. Now as can be seen, D is a mathematical operation. The result is 5
which can be obtained only if D is a subtraction operation. Thus D = (-)

51. (c) 6 C 3 + 5 = 23. Clearly C is a mathematical operation. The equation can also be
written as 6 C 3 = 23 - 5 ⇒ 6 C 3 = 18. To equate both the sides, we must multiply
6 and 3. Thus C is a multiplication operation, C = ×.

52. (c) Both these statements are true.

53. (a) 44 – 6 + 43 × 2 – 87

⇒ 44 – 6 + 86 – 87 ⇒ 130 – 93 = 37

5 2 2 6
54. (a) × + ÷
6 9 9 7

5 2 2 7 10 14 24
× + × == + = = 0.44
6 9 9 6 54 54 54
55. (d) 50 + [100 – {35 – (30 – 20)}] × 2

50 + [100 – {35 – 10 }] × 2

50 + [100 – 25] × 2

50 + 75 × 2 = 50 + 150 = 200

56. (c) 3 3 3 3 2 3
÷ ×2+ = × ×2+
2 2 2 2 3 2
3 4+3 7
= 2 + = =
2 2 2

57. (b) (4 – 3) x 4 = 1 × 4 = 4

58. (b) (7 + 3) x (3 + 4) = 10 × 7 = 70

59. (d) 3 x 4 - 2(5 - 2) ⇒ 12 -2(3) ⇒ 12 - 6 = 6

60. (c) 15 - (99 - 93) ⇒ 15-6 = 9

61. (c) 600 + 1200 ÷ 6 × 2 + 400 × 2 = 600 + 400 + 800 = 1800

(11 + A)
62. (a) = 5 . Solving the denominator, we get 97 - 94 = 3. Now to get the result
(97 − 94)
equal (97 - 94) to 5, 3 must be divided by 15. And to obtain 15 in the numerator,
we must add 4, such that 4 + 11 = 15. Thus A = 4.
EBD_7364
132 Olympiad Champs–Mathematics
63. (a) ? × (3 + ? ) = 14. The numbers should be 2 and 4. 14 can be obtained by multiplying 2
and 7. Thus to make the numbers in bracket equal to 7, we should 4 add to 3. And
then multiply the sum of 7 with 2, to obtain 14.
64. (c) 145 + (120 ÷ 20 + 6) = 145 + (6 + 6) = 145 + 12 = 157
65. (a) 5 × 4 – 2 × 3 + 16 ÷ 4
= 20 – 6 + 4 = 14 + 4 = 18
66. (c) (9 + 7) 4 × 5 ⇒ 16 ÷ 4 × 5 = 4 × 5 = 20

67. (b) 1 + 7 ÷  7 × 1 1  = 1 + 7 ÷  7 × 5 
3 9  10 4  3 9  10 4 
1 7 7 1 7 8
= + ÷ = + ×
3 9 8 3 9 7

= 1 + 8 = 3 + 8 = 11 = 1 2
3 9 9 9 9

68. (b) 11.2 – 0.144 + 6 = 17.056

1 1
69. (b) 2.25 ÷ 9 + 1 – (6.5 × 0.2) = ____ ×
8 4
1
⇒ 0.25 + 1.125 – 1.30 = 1.375 – 1.30 = 0.075 = 0.3 ×
4
7 19 9 5 7 19 9 5
70. (c) + × × ÷2 = + × ×
2 7 26 2 2 7 26 4

7 855 2548 + 855 3403 491


=+ = = =4
2 728 728 728 728
71. (c) 0.11 × 33 + 0.33 – 3.3 = 0.66

1
72. (c) 100 ÷ 200 × 300 + 400 – 500 = × 300 + 400 – 500
2
= 150 + 400 – 500 = 50

73. (a) 1 5 7 1
3 +2 × – ÷2
2 7 19 2
7 19 7 1
= + × –
2 7 19 4
7 1 9 1 17 1
=+ 1 – =– = = 4
2 4 2 4 4 4
74. (d) Step II and IV are wrong

75. (d) As 2x +1.5 =3.5 ⇒ X = (3 × 3.5 – 1.5) ÷ 2 ⇒ x = 9 ÷ 2 = 4.5


3
133
Ratio

CHAPTER FOREWORD
Consider the following case :
After celebrating her thirteenth birthday, Madhu felt stomach pain, loss of appetite and
weakness in the night. As all the symptoms indicated diarrhoea, so to prevent Madhu from
dehydration, her mother quickly made lemonade for her as given below, before taking her to
the doctor.

I. She took 1 large glass of water.

II. Then added juice of 1 full lemon.

III. Added 2 teaspoons of salt and 1 teaspoon of sugar.

IV. Mixed it well.

• What is the ratio of water, lemon juice, salt and sugar in the above case?

Here mixing a particular amount of different ingredient is very important. This situation
involves concept of Ratio.

Ratio is a comparison between two or more than two things. It is represented by (:) symbol.

4:2

2:1
Symbol of ratio.
After reading this chapter, you will learn about the simplification of ratio and equivalent ratio.
EBD_7364
134 Olympiad Champs–Mathematics

8
Chapter Ratio

LEARNING OBJECTIVES
Real-Life Examples
This lesson will help you to:—
v The concept of ratio is very v understand the concept of ratio.
commonly used in cooking .
v simplify ratio and understand equivalent ratio.
A recipe has ingredients in a
certain fixed ratio. For example; v determine and define unit rate.
The ratio of flour to sugar is v use scale drawings to measure.
3 : 1.
v Builder use ratio in constructing QUICK CONCEPT REVIEW
a building. The raw material
used should be in fixed ratio to Ratio between two or more things of same kind is a way is
construct a strong building. which things are compared to each other.
For example : 60 km per hour is a ratio or 12 girls to 13
boys in a class is a ratio.

PROPERTIES OF RATIO
a
v There are 3 ways to write a ratio, a to b; and
a : b. b
Do You Know?
v A ratio can be scaled up.
v The unique number For example:
1.6180339887 is called as
Golden ratio in mathematics.

The ratio of checkered ball : striped balls is 2 : 1.

Antecedent and consequent.


In a ratio a : b, a is called antecedent and b is called as
consequent. For example in 3:1, 3 is antecedent and 1 is
consequent
Equivalent Ratio:
Two ratios which are equal are called equivalent ratios.
For example 1:5 = 2:10 = 3:5 = 4:20
135
Ratio

Amazing Facts
Amazing Facts
v The ratio of length to breadth
of Indian flag is 3 : 2.

The ratio of checkered ball : striped balls is 6 : 3 =


2 : 1, even though there are more balls.

PLAY TIME v The ratio of speed of sound
underwater to air is 5 : 1. i.e.,
Have a bag of red balls and blue balls. Divide your sound travels five times faster
friends in teams. Distribute same number of red and underwater than in air.
blue balls to all the teams . Ask the team to divide the
v The ratio of width of the base
balls in asked ratio. of the nose and the width of

the eye is equal.
Proportion : It is equality of two ratios. v The ratio of width of the mouth
For example : a : b = c : d is called a proportion. and the width of two eyes is
So that ad = bc equal.
Preposition is represented by the symbol : :
Properties of a Ratio and Proportion
(a) Invertendo : If a : b is equal to c : d then b : a is
also equal to d : c
(b) Alternendo : If a : b = c : d, then a : c = b : d
(c) Componendo : If a : b = c : d, then
(a + b) : b = (c + d) : d Try It!

(d) Dividendo : If a : b = c : d, then Example : Convert 14 : 21 as a


(a – b) : b = (c – d) : d fraction in lowest terms
14 2
(e) Componendo–Dividendo : If a : b = c : d, Solution : 14 : 21 = =
21 3
Which is a production in lower
a+b c+d terms
then =
a+b c–d
EBD_7364
136 Olympiad Champs–Mathematics

Multiple Choice Questions


LEVEL 1
?
1. What does ________ stand for?

?
height : feet : : age : ________

(a) years (b) inches (c) metres (d) kilograms


?
2. What does ________ stand for? (Mental Mathematics)

?
5 : 35 : : 10 : ________

(a) 7 (b) 70 (c) 10 (d) 5

3. What is the ratio of to ? [Mental Mathematics]

(a) 3 : 1 (b) 1 : 3 (c) 1 : 2 (d) 2 : 1


4. Sheena is making a mocktail for a party. She wants to mix apple juice and orange
juice in the ratio of 4 : 7. If Sheena wants to make 22 litres of mocktail, how
much apple juice will she need for the drink? (Mental Mathematics)
(a) 8 litres (b) 14 litres (c) 11 litres (d) 7 litres
5. A shopkeeper sold 60 apples and mangoes in a day. The ratio of number of apples
sold to the number of mangoes sold is 5 : 7. How many mangoes are sold?
(Mental Mathematics)

(a)
25 (b)
50 (c)
70 (d)
35
6. A recipe needs sugar and flour in the ratio 1 : 3. If 750 grams of flour is used in
the recipe, how much sugar is needed? (Mental Mathematics)
(a) 500 grams (b) 250 grams (c) 700 grams (d) 200 grams

7. Priya bought a book for ` 30/- and Shreya bought a book for ` 150/-. How many
times is the cost of Shreya’s book to that of Priya’s book?
(a) 5 times (b) 4 times (c) 3 times (d) 2 times
8. The length of a room is 6 metres and the breadth is 4 metres. What is the ratio
of length to the breadth of the room? (2017)

(a) 2 : 3 (b) 2 : 5 (c) 3 : 2 (d) 5 : 3


137
Ratio
9. What is the ratio of ( – ) to ? (Tricky)



(a) 5 : 3 (b) 3 : 5 (c) 2 : 5 (d) 5 : 2
10. Richa has 100 cm of ribbon and cuts it in the ratio of 3 : 7. How much shorter is
the shorter piece from the longer one? (2010)
(a) 30 cm (b) 70 cm (c) 50 cm (d) 40 cm
11. Which of the following statements are true? (Critical Thinking)
(A) 1 : 3 is same as 2 : 9.
(B)
` 30 divided is the ratio of 1 : 2 , will have ` 10 is the larger part.
(C) 2 : 8 is same as 1 : 4.
(D) 6 : 18 : : 1 : 3.

(a)
TFTF (b)
TTFF (c)
FFTT (d) FTFT
12. There are 20 books kept in a pile. The ratio of fiction books to the remaining books
is 2 : 3. Which of the following statements are true? (2008)
(A) There are 12 fiction books.
(B) The ratio of fiction to remaining kind of books is 4 : 6.
(C) There are 8 fiction books.
(D) The ratio of remaining to fiction books is 9 : 6.

(a)
FTTF (b)
FFTT (c)
TFTF (d)
TTFF
13. How many of the below are same as 4 : 5 ?
(1) 8 : 10 (2) 6 : 30 (3) 16 : 20 (4) 20 : 25
(a) 0 (b) 1 (c) 2 (d) 3
14. Fill in the blank
15 –
=
18 6
(a) 5 (b) 12 (c) 16 (d) 3
15. True or false – 7th is the third part of 21. Inversely 21 is three times 7.
(a) True (b) False (c) Partially true (d) None
16. True or false – 30 is 6 times of 6. Inversely, 6 is the 6 part of 30. (2015)
(a) True (b) False (c) Partially true (d) None
EBD_7364
138 Olympiad Champs–Mathematics
17. The distance between two towns A and B is 300 km. What is the distance on a map,
if 30 km is shown by 1 cm?
(a) 10 cm (b) 5 cm (c) 3 cm (d) 50 cm
18. Which of the below is odd one out? (2017)
(a) 1 : 3 (b) 2 : 6 (c) 2 : 5 (d) 4 : 9
19. Which is same as the ratio 2 metres to 700 cm ?
(a) 2 : 5 (b) 7 : 2 (c) 5 : 2 (d) 2 : 7
20. Which shows two equivalent ratios of 24 : 8? (2015)
(a) 18 : 6 ; 15 : 5 (b) 12 : 3 ; 6 : 4
(c) 30 : 5 ; 40 : 10 (d) 8 : 2 ; 27 : 9

LEVEL 2
21. In an ice cream shop, the ratio of the number of chocolate ice creams sold to the
number of vanilla ice cream sold is 1 : 4. If 48 vanilla ice creams were sold, how
many chocolate ice creams were sold ? (2004)

(a)
14 (b)
10 (c)
12 (d)
16
22. Seema had ` 150/- for shopping. She spent ` 20/- on stationary, ` 60/- on books
and ` 40/- on food. What is the ratio of the money spent to the money he had for
shopping?
(a) 5 : 4 (b) 2 : 3 (c) 3 : 2 (d) 4 : 5
23. The table below shows the number of fruits is a fruit stall. (2009)

Fruit Number of fruits


Apples 70
Bananas 50
Oranges 85
Mangoes 45
What is ratio of mangoes to the total number of fruits?
(a) 9 : 50 (b) 17 : 50 (c) 50 : 9 (d) 50 : 17
24. The table below shows the number of animals in a zoo. (2015)
Animals Number of animals
Lion 3
Zebra 9
Elephant 12
Monkey 12
Giraffe 6
What is the ratio of number of giraffes and zebras to the number of monkeys?
(a) 2 : 3 (b) 3 : 2 (c) 4 : 3 (d) 5 : 4
139
Ratio
25. What is the actual length of the rectangle drawn below?


Scale : 1 unit = 2 metres
(a) 9 metres (b) 12 metres (c) 6 metres (d) 8 metres
26. Which of the following is a better buy? (Tricky)
(A) 6 apples for ` 30 (B) 4 apples for ` 16
(C) 5 apples for ` 25 (D) 8 apples for ` 48
(a) A (b) B (c) C (d) D

27. Match the following: (Tricky)

List I List II
A. 3 m to 200 cm 1. 3:5
B. 21 days to 5 weeks 2. 3:4
C. 120 seconds to 8 min 3. 3:2
D. 1500 ml to 2l 4. 1:4
A B C D
(a) 1 2 3 4
(b) 3 1 4 2
(c) 2 3 4 1
(d) 4 2 1 3

28. Match the following: (Critical Thinking)

List I List II
A. 12 stamps on 3 pages 1. 5 stamps per page
B. 18 stamps on 6 pages 2. 7 stamps per page
C. 25 stamps on 5 pages 3. 4 stamps per page
D. 49 stamps on 7 pages 4. 3 stamps per page

A B C D
(a) 1 4 2 3
(b) 1 3 4 2
(c) 2 1 4 3
(d) 3 4 1 2
EBD_7364
140 Olympiad Champs–Mathematics
29. If three numbers are in the ratio of 1 : 2 : 3 and their HCF is 12, then numbers
are : (2008)

(a) 4, 8, 12 (b) 12, 24, 36 (c) 12, 24, 30 (d) 5, 12, 15

30. How many of the below shows the ratio of balls to the ratio of toy cars as
2 : 3 ?

(A)

(B)

(C)

(D)

(a) 0 (b) 1 (c) 2 (d) 4


31. Suhana earn ` 800/- for 8 hours work. How long will she have to work to earn
` 2000/- ?
(a) 12 hrs (b) 24 hrs (c) 16 hrs (d) 20 hrs
32. The table below shows the time taken and the number of kilometres Hari cycled in
last 6 weeks? (2014)

Km. 8 10 12 14 16 18
Time taken (in min.) 40 50 60 70 80 90

How much time will Hari take to cycle 22 km ?

(a) 100 min (b) 120 min (c) 110 min (d) 115 min
141
Ratio
33. Sameer needs to fence the field shown is the figure below. How much actual wire
will be needed to fence the field?

1 unit = 3 metres
(a) 10 metres (b) 80 metres (c) 90 metres (d) 60 metres

34. A car travels 360 km in 3 hours. How many kilometres will it travel is 5 hours?

(a) 500 km (b) 550 km (c) 600 km (d) 450 km

35. Express the ratio 15 red apples out of 50 apples as a fraction in simplest form.
10 8 15 50
(a) (b) (c) (d)
3 10 50 15
36. Richa stitches a curtain that is 7 feet long and 2 feet wide. She stitches another
curtain of with same ratio. If the breadth of the curtain is 4 feet, what is the
length of the curtain.

(a) 16 feet (b) 10 feet (c) 12 feet (d) 14 feet

Directions (Qs 37 and 38) : A soccer team played 25 games and won 17. Answer the
following questions.

37. What is the ratio of the number of wins to number of loses?

(a) 8:25 (b) 8:17 (c) 17:8 (d) 17:25

38. What is the ratio of the number of games played to the number of games won?


(a)
17:25 (b)
25:17 (c)
25:8 (d)
8:25

39. This ratio in the simplest form : (Tricky)

1 5
1 :2
6 6

(a) 6 : 17 (b) 7:17 (c) 17:7 (d) 17:5


40. Two numbers are in a ratio of 5 :3. Their sum is 80. Find the largest number.
(2011)


(a)
40 (b)
30 (c)
70 (d)
50
EBD_7364
142 Olympiad Champs–Mathematics
41. The ratio of doves to parrots in a cage is 13 to 12. If there are 108 parrots, then
how many doves are present in the cage?

(a) 120 (b) 117 (c) 110 (d) 95

42. The ratio of two lengths x : y is 5 : 6. If y is 54 cm long, what is the length


of x ? (2012)

(a) 40 cm (b) 45 cm (c) 50 cm (d) 55 cm

43. The venn diagram below shows the kind of movie 50 people would like to watch.
(Tricky)

A 14 B
16
2
1 3

14 C

A → Romance B → Comedy C → Action

What is the ratio of the number of people who like to watch only action movies to
the number of people who like to watch only comedy movies ?

(a) 7 : 8 (b) 2 : 7 (c) 1 : 1 (d) 1 : 2


44. A group of 100 students were asked which hobby do they like. The venn diagram
shows the data (2014)

A 25 B
20
5
10 10

30 C

A → Music B → Dance C → Dramatics

What is the ratio of number of students who like both music & dance but not
dramatics to the number of students who like only dramatics ?

(a) 6 : 1 (b) 1 : 6 (c) 1 : 3 (d) 3 : 1


143
Ratio
45. The table below shows the money saved by Amit is last 6 months. (Tricky)

Month Jan. Feb. Mar. Apr. May Jun.


Money saved
550 200 330 450 600 700
(in `)

What is the ratio of money saved in Jan and Feb to the money saved in May and
June ?
(a) 15 :26 (b) 26 : 15 (c) 11 :23 (d) 23 : 11
46. Jim and Maria saved money in the ratio 8 : 12. Maria's saving as a fraction of
Jim's saving is (2010)
2 1 3 1
(a) (b) (c) (d)
3 3 2 2
47. The ratio is said to be in simplest form if common factor is _________.

(a) 1 (b) 0 (c) 2 (d) None of these


48. A sum of money is to be distributed among A, B, C, D in the proportion of
5 : 2 : 4 : 3. If C gets `1000 more than D, what is B's share? (2016)

(a) ` 500 (b)


`1500 (c)
`2000 (d) None of these
49. True or false : Two quantities can be compared only if they are in the same
units. (Critical Thinking)

(a) True (b) False (c) Partially true (d) None of these
50. In a recipe, you have to add one cup of sugar for every four cups of flour. What
is the ratio of sugar to flour? (2016)

(a) 4:1 (b) 2:4 (c) 1:2 (d) 1:4

RESPONSE GRID
1. a b c d 2. a b c d 3. a b c d 4. a b c d 5. a b c d
6. a b c d 7. a b c d 8. a b c d 9. a b c d 10. a b c d
11. a b c d 12. a b c d 13. a b c d 14. a b c d 15. a b c d
16. a b c d 17. a b c d 18. a b c d 19. a b c d 20. a b c d
21. a b c d 22. a b c d 23. a b c d 24. a b c d 25. a b c d
26. a b c d 27. a b c d 28. a b c d 29. a b c d 30. a b c d
31. a b c d 32. a b c d 33. a b c d 34. a b c d 35. a b c d
36. a b c d 37. a b c d 38. a b c d 39. a b c d 40. a b c d
41. a b c d 42. a b c d 43. a b c d 44. a b c d 45. a b c d
46. a b c d 47. a b c d 48. a b c d 49. a b c d 50. a b c d
EBD_7364
144 Olympiad Champs–Mathematics

Solutions with Explanation


LEVEL 1
1. (a) Feet is the unit for height.
The unit for age is years.
2. (b) 5 × 7 = 35
10 × 7 = 70
So, 70 is the correct answer.
3. (c) No. of squares = 9
No. of triangles = 18
Ratio of to ∆ = : ∆ = 9 : 18 = 1 : 2.
4
4. (a) Quantity of apple juice = × 22
4+7
4
= × 22 =4 × 2 = 8l
11
5. (d) Numbers of mangoes sold
7 7
= 60 ×= 60 × = 35
5+7 12
6. (b) Let x grams of sugar is needed.


x 1
⇒ x = 250 grams.
=
750 3
7. (a) Cost of Shreya’s book to cost of Priya’s book

150 5 = 5
= 150 := 30 =
30 1
6 3
8. (c) Ratio of length to breadth= 6 : 4
= = .
4 2
9. (b) Number of circles = 20.

Number of squares = 8.

– = 12

( – ): = 12 : 20

= 3 : 5.
7 7
10. (d) Longer piece is = 100 × = 100 × = 70 cm
3+ 7 10
Shorter piece =(100 – 70)cm = 30 cm long.
Difference = 70 – 30 = 40 cm.
145
Ratio

1 1×3 3
11. (c) (A) = = ; a false.
3 3×3 9
(B) x + 2x = 30 ⇒ x = 10; ` 10 and ` 20; (B) is false.

(C) 2 = 1 ; (C) is true.


8 4
6 1
(D) = ; (D) is true.
18 3
2 2
12. (a) Number of fiction books = 20 × = 20 × = 8
2+3 5
Number of remaining kind of books = 20 – 8 = 12
\ Ratio of fraction books to the remaining books = 8 : 12 = 4 : 6
and ratio of remaining books to fiction books = 6 : 4

4 4 × 2 8 4 4 × 4 16
13. (c)
= = ;
= = ;
5 5 × 2 10 5 5 × 4 20
4 4 × 6 24 4 4 × 5 20
= = ;
= =
5 5 × 6 30 5 5 × 5 25
14. (a) 5
15. (a) 16. (b)

17. (a) Let x be the distance on the map

300 : x = 30 : 1

300 30
= ⇒ x = 10 cm.
x 1
2 1
18. (b) Only b can reduced further 2 : 6= = all the remaining ratios are in simplified
6 3
form.

19. (d) 2 metres = 200 cm

200 2
200 : 700
= = = 2:7
700 7

24 3
20. (a) 24 : 8=
= = 3:1
8 1
3 3 × 6 18 3 3 × 5 15
(a) = = = 18 : 6; = = = 15 : 5 ; true
1 1×6 6 1 1×5 5
3 3 × 4 12
(b) = = = 12 : 4 ; false
1 1×4 4
3 3 × 10 30
(c) = = = 30 : 10 ; false
1 1 × 10 10
3 3 × 9 27 8 4
(d) = = = 27 : 9; = = 4 : 1 ; false
1 1×9 9 2 1
EBD_7364
146 Olympiad Champs–Mathematics
LEVEL 2
21. (c) If x is the number of chocolate ice creams sold then

1 x
= ⇒ x = 12.
4 48

22. (d) Money spent on shopping = 60 + 40 + 20 = 120

Ratio = 120 : 150 = 4 : 5.

23. (a) Total number of fruits = 70 + 50 + 85 + 45 = 250

45 9
Ratio
= 45 : 250
= = = 9 : 50.
250 50
24. (d) No. of giraffes and zebras = 6 + 9 = 15.

Ratio of number of giraffes and zebras to the number of monkey = 15 : 12

15 5
= = = 5 : 4 .
12 4
25. (b) Length of the rectangle is the figure = 6 units.

Actual length = 6 × 2 = 12 metres.

30 16
26. (b) (a) Cost of 1 apple = = ` 5 (b) Cost of 1 apple = =`4
6 4
25 48
(c) Cost of 1 apple = = ` 5 (d) Cost of 1 apple = =`6
5 8
300 3
27. (b) 3 m to 200 cm = 300 cm to 200 cm= = = 3:2
200 2
321
21 days to 5 weeks = 21 days to 35 days= = 3:5
=
535
2 1
(c) 120 seconds to 8 min = 2 min to 8 min= = = 1 : 4
8 4
1500 3
(d) 1500 ml to 2l = 1500 ml to 2000 ml = = = 3: 4
2000 4
12 18
28. (d) (A) = 4 stamps / page (B) stamps / page
3
25 49
(C) = 5 stamps / page (D) = 7 stamps / page
5 7
29. (b)

6 2 4 12 3 10 5
30. (b) = = 2 : 3; = 4 : 5; = = 3 : 2; = = 5 : 6
9 3 5 8 2 12 6
31. (d) 8 : 800 = x : 2000 (where x is the number of hours Suhana needs to work)

8 x 8 × 2000
= x
⇒= ⇒ 20 hours.
800 2000 800
147
Ratio

32. (c) 8 22 1 22 ⇒ x = 22 × 5 = 110 minutes


= ⇒ =
40 x 5 x
(Where x is time taken to cycle 22 km.)

33. (c)
6

2

2
6
3
2
7

(show 10 × 10 grid)

Units around the field = 6 + 2 + 2 + 2 + 3 + 2 + 7 + 6 = 30.

1 30
Length of wire needed ⇒ = ⇒ x = 3 × 30 = 90 metres.
3 x
360 x
34. (c) = ⇒ x = 120 × 5 = 600 km.
3 5
(x is no. of km to travel in 5 hours)

15
35. (c)
50
7 x
36. (d) = ⇒ x = 14 feet.
2 4
(Where x is the length of the second curtain)

37. (c) 38. (b)

7 17
39. (b) The ratio changes to :
6 6
7 6
× 7 : 17
=
6 17
40. (d) Let first number be 5x.
Let second number be 3x.
5x + 3x = 80
8x = 80
x = 10
substitute x to find largest number i.e., 5(10) = 50.
41. (b)
EBD_7364
148 Olympiad Champs–Mathematics

5 x
42. (b) = ⇒ x = 9 × 5 = 45 cm
6 54
43. (c) Number of people who like to watch only action movies = 14.

Number of peoples who like to watch only comedy movies = 14.

Ratio = 14 : 14 = 1 : 1

44. (b) Number of students who like music and dance = 5 and not dramatics

Number of students who like dramatics only = 30.

Ratio = 5 : 30 = 1 : 6.

45. (a) Money saved in Jan and Feb = 550 + 200 = 750.

Money saved in May and June = 600 + 700 = 1300.

750 75 15
Ratio
= 750 : 1300
= = = = 15 : 26 .
1300 130 26
46. (c)
47. (a) 1
48. (c) Let shares of A, B, C and D be `5x, `2x, `4x and `3x
Then 4x – 3x = 1000
x = 1000
B's share = `2000.
49. (a) True
50. (d) 1 : 4
149
Temperature

CHAPTER FOREWORD
Consider the following case

Nim was carefully watching what his father was doing for her sick brother.

→ Her father brought a thermometer.

→ Then he placed the tip of this thermometer under the tongue of Nim's brother.

→ He held the thermometer in the same spot for 40 seconds.

→ Then came a beeping sound and her father saw the tube. He said " the temperature is

above the normal temperature so we need to go the doctor immediately".

Nim wondered what happened.

• What is this normal temperature?

The word temperature refers to the degree of hotness or coldness of any object or atmosphere.
There are mainly two units of temperature:
Units of Temperature

Fahrenheit Celsius
(Invented by Daniel Gabriel (Invented by Anders
Fahrenheit (1686-1736)) Celsius (1701 – 1744))

Now try answering this question:

• What is the difference between celsius and centigrade?

After reading this chapter, you will learn about the above mentioned units and their conversion
into each other.

Note that temperature always flows from higher level to lower level until the temperature of
both the bodies become equal. At that temperature , the two bodies are said to be in thermal
equilibrium.
EBD_7364
150 Olympiad Champs–Mathematics

9
Chapter
Temperature

LEARNING OBJECTIVES
Real-Life Example
This lesson will help you to:—
v We use thermometers at our
home to check the temperature v understand the concept of temperature.
of our bodies during fever. We v learn and study about various measuring scales of
place the thermometer either temperature.
in the mouth or under the
arm-pits to measure the body v study and learn about the conversion from one scale
temperature. to another.

QUICK CONCEPT REVIEW

TEMPERATURE
v It is the degree of hotness or coldness of a body or
environment.
Amazing
Amazing Facts
Facts
v Temperature is measured in units called degrees.
v An interesting temperature There are a few different temperature scales,
related fact is that Fahrenheit including degrees Fahrenheit and degrees Celsius
and Celsius are equal at -40 represented as °F and °C respectively.
degrees.
v The hottest temperature ever v Temperature is measured using a thermometer.
recorded on Earth is 57.8 °C v When you boil water, it measures 100° in Celsius, but
(136 °F), recorded in Al
212° in Fahrenheit.
'Aziziyah, Libya on September
13, 1922. v When you freeze water, it measures 0° in Celsius,
v The coldest temperature ever but 32° in Fahrenheit.
recorded on Earth is −89.2 °C
(−128.6 °F), recorded at Vostok v So the difference between freezing and boiling is
Station, Antarctica on July 21, 100° in Celsius, but 180° in Fahrenheit:
1983.
v The temperature of our bodies is about 37°C or
98.6°F.
151
Temperature
C F
Boil 100 220
210 212
90 200

80
190
180
Historical
Amazing Preview
Facts
170
70 160
60
150
140 v One of the early scientists
100 50
130
120 180 to start developing a way of
40 110
100 measuring temperature was
30 90
80 Galileo Galilei. These devices
20 70
60 were called “thermoscopes”
10 50
40 because they did not actually
0
Freeze 30
20
32 have a scale which measured
-10 10
0 temperature. However, records
-20
-30
-10
-20 from this time period do allow
scientists to reconstruct world
temperatures much more
accurately. Galileo invented the
Temperature Conversion first documented thermometer
in about 1592.
By looking at the diagram, it can be seen that:
v By the early 18th century,
v The scales start at a different number (0 v/s 32), so
as many as 35 different
we will need to add or subtract 32. temperature scales had been
v The scales rise at a different rate (100 v/s 180), so devised.
we will also need to multiply.
And this is how it works out:
v To convert from Celsius to Fahrenheit, first multiply
by 180/100, then add 32 Misconcept/Concept

v To convert from Fahrenheit to Celsius, first subtract Misconcept : All liquids boil at
32, then multiply by 100/180 100°C (212°F) and freeze at
0° C (32°F).
Note: 180/100 can be simplified to 9/5, and likewise
Concept : Not all liquids boil at
100/180=5/9, so this is the easiest way: 100°C and freeze at 0°C. This is
°C to °F Multiply by 9, then divide by 5, then add 32 the melting and freezing point
of water only. Each and every
°F to °C Deduct 32, then multiply by 5, then divide liquid has different properties
by 9 and thus melt and freeze at
different temperatures.
We can write that as a formula like this:
Celsius to Fahrenheit (°C × 9/5) + 32 = °F
Fahrenheit to Celsius (°F - 32) x 5/9 = °C

THERMOMETER
v We can use a thermometer to measure the
temperature. Today, thermometers no longer contain
mercury due to potential health risks; they are filled
EBD_7364
152 Olympiad Champs–Mathematics
with a combination of mineral spirits or alcohol mixed
Shortcut to Problem Solving with red dye. In these thermometers, the red liquid
rises and falls as it gets hotter or cooler. The hotter
To remember 9/5 for °C to °F
the temperature, the higher the liquid climbs up the
think "F is greater than C, so
there are more °F than °C" thermometer. The lower the temperature, the lower
it goes down the thermometer.
v Certain thermometers are made of glass, so they
should handle them carefully to avoid breaking. On
some thermometers, the numbers go up by ten, while
Kelvin Scale of Temperature
on others they go up by five.
S.I. unit of temperature is v To find the temperature, look at where the
Kelvin. Celsius temperature can
special liquid stops along the number line. On some
be converted to Kelvin scale of
temperature by adding 273 to thermometers the marks need to be skip-counted
celsius temperature . by twos, while on others they are simply counted by
For example 32°C can be ones.
converted to Kelvin. Scale by
adding 273 to it. SCOPE OF MEASURING TEMPERATURE
So 32° C = (32 + 273)° K
v We use thermometers to measure the temperature
= 305° K
Relation between celsius and outside and inside, and to check the temperatures
Fahrenheit can be obtained by of our bodies.
C – 0 F –32 v We set the temperature on ovens and measure the
=
100 180 temperature of food.
Where 0°C or 32°F is freezing
point of water and 100°C or 180°F
is boiling point of water

Example : Convert 10° into °F


9
Solution : 10 × + 32
5
= 18 + 32 = 50°F
Example : Convert 59°F into °C
5
Solution : (59 –32) ×
9
5
= 27 ×
9
= 3 × 5 = 15°C
153
Temperature

Multiple Choice Questions


LEVEL 1
1. What is the temperature outside on a snowy day? Choose the more reasonable
answer.
(a) 40°C (b) 0°C (c) 50°C (d) 63°C
2. Pick the odd one out.
(a) Metre (b) Centimetre (c) Degree (d) Kilometre
3. What is the temperature of a hot barbecue grill? Choose the more reasonable
answer. (Mental Mathematics)
(a) 260° C (b) 40° C (c) – 10° C (d) 10° C
4. Which thermometer best represents the temperature 65° F? (Mental Mathematics)
A B C D
C F C F C F C F
40 40 40 40
100 100 100 100
90 90 90 90
30 30 30 30
80 80 80 80

20 70 70 20 70 20 70
20
60 60 60 60
10 50 10 50 10 50 10 50
40 40 40 40
0 30 0 30 0 30 0 30
20 20 20 20
-10 -10 -10 -10
10 10 10 10
0 0 0 0
-20 -20 -20 -20
-10 -10 -10 -10
-30 -20 -30 -20 -30 -20 -30 -20

(a) A (b) B (c) C (d) D


5. __________ is the instrument that is used to measure the temperature.
(a) Thermo-scope (b) Thermometer (c) Thermo-bar (d) Thermo-scale

6. Metre: Length::_________: Temperature.


(a) Celsius (b) Fahrenheit (c) Degree (d) Gram
Directions (Qs. 7 to 11) : Convert the following given temperatures from degree Fahrenheit
to degree Celsius.
7. 77 °F = __________°C (Mental Mathematics)
(a) 28 (b) 25 (c) 26 (d) 45
EBD_7364
154 Olympiad Champs–Mathematics
8. 203 °F = __________°C
(a) 95 (b) 75 (c) 85 (d) 78
9. 86 °F = __________°C
(a) 20 (b) 30 (c) 62 (d) 35
10. 176 °F = __________°C
(a) 85 (b) 78 (c) 65 (d) 80
11. 212 °F = __________°C
(a) 102 (b) 103 (c) 101 (d) 100

12. The water in Megha’s watering bucket is frozenn. Which would most likely be the
temperature of the water? (2010)
(a) 0ºC (b) 100ºC (c) 50ºC (d) 25ºC
13. Clinical thermometers are marked in _________ scale.
(a) Celsius (b) Fahrenheit (c) Both A and B (d) None of these
14. The scales of temperature are named after:
(a) instruments (b) scientists (c) places (d) none of these
15. Normal human body temperature is _______ °F.
(a) 37 (b) 98.8 (c) 98.6 (d) 98.7
16. Number of equal divisions in the Fahrenheit scale is :
(a) 100 (b) 108 (c) 180 (d) 212
17. Choose the correct option. (2015, Tricky)
(a) 85°C = 105°F (b) 50°C =120°F
(c) 40°C = 95°F (d) 80°C = 176°F
18. 75°C is _______ °F
(a) 176 (b) 167 (c) 107 (d) 112
19. 302°F is _______ °C. (2015)

(a) 105 (b) 155 (c) 150 (d) 205


20. Choose the correct option.
155
Temperature
(a) The temperature in the thermometer is the normal human body temperature.
(b) The temperature in the thermometer is below the normal human body temperature.
(c) The temperature in the thermometer is above the normal human body temperature.
(d) None of these.

LEVEL 2
21. Match the following:

List I List II
A. Temperature of boiling water 1. 98.6°F
B. Temperature of freezing water 2. 100°C
C. Normal temperature of human body 3. 32°F
A B C
(a) 1 2 3
(b) 3 2 1
(c) 2 3 1
(d) 1 3 2
Directions (Qs. 22 to 24) : By studying the temperature chart for the month of September,
answer the following questions. (Critical Thinking)

September Mon. Tues. Wed. Thurs. Fri. Mon. Tues. Wed. Thurs. Fri. Mon.
6 7 8 9 10 13 14 15 16 17 20

Hot

Warm

Cold

22. The coldest day of the month is


(a) Monday (b) Tuesday (c) Wednesday (d) Thursday
23. Second highest temperature was measured on
(a) 6th September (b) 7th September
(c) 8th September (d) 9th September
EBD_7364
156 Olympiad Champs–Mathematics
24. Which of the following is correct with respect to the graph.
(a) Gradual increase in temperature (b) Gradual decrease in temperature
(c) No change in temperature (d) None of these
25. Read the statement and choose the correct option. (Critical Thinking)
Statement A: (°C × 9/5) + 32 = °F.
Statement B: (°C × 9/5) - 32 = °F.
(a) Only statement A is true. (b) Only statement B is true.
(c) Both A and B are true. (d) Both A and B are false.
26. An industrial machine heats up to 161°F when it's being used. After being unused
for an hour the temperature drops to 144°F. How much did the machine cool
off? (2009)
(a) 17°F (b) 16°F (c) 18°F (d) 19°F
27. What is the difference between the temperature of day 3 and day 1?
C C
C 50 50
50

40 40
40

30 30
30

20 20
20

10 10 10

0 0 0


Day 1 Day 2 Day 3

(a) 10° (b) 15° (c) 25° (d) 55°


Directions (Qs. 28 to 31) : Convert the following given temperatures from degree Celsius
to degree Fahrenheit.
28. 80°C =__________°F (2008)
(a) 175°F (b) 80°F (c) 60°F (d) 176°F

29. 95°C =__________°F


(a) 203°F (b) 200°F (c) 204°F (d) 209°F

30. 30°C =__________°F (2010)


(a) 65°F (b) 78°F (c) 86°F (d) 75°F

31. 15°C =__________°F (2012)


(a) 59°F (b) 79°F (c) 47°F (d) 76°F
32. Megan measured the temperature of her soda and found that it was 47°F. After
sitting out for an hour it had warmed by 21°F. What was the temperature of soda
after an hour? (2017)
(a) 69°F (b) 58°F (c) 26°F (d) 68°F
157
Temperature
Directions (Qs. 33 to 38) : Choose the correct option by studying the readings on the
thermometer.
100
33. What temperature is shown on the thermometer? (2017) 90
80
(a) 33 degree 70
60

(b) 36 degree 50
40
30
(c) 38 degree 20
10
0
(d) 37 degree

34. What temperature is shown on the thermometer? (2015) 50

40
(a) 4 degree
30

(b) 3 degree 20

10
(c) 2 degree
0

(d) 8 degree

35. What temperature is shown on the thermometer? (2016) 50

40
(a) 39 degree
30

(b) 40 degree 20

10

(c) 42 degree 0

(d) 50 degree

36. What temperature is shown on the thermometer? 100


90
80

(a) 97 degree 70
60
50
40
(b) 95 degree 30
20
10
(c) 94 degree 0
-10
-20
(d) 100 degree
EBD_7364
158 Olympiad Champs–Mathematics
37. What temperature is shown on the thermometer?
100
90
80
(a) 69 degree 70
60
50
40
(b) 70 degree 30
20
10
0
(c) 68 degree -10
-20

(d) 66 degree

38. What temperature is shown on the thermometer? (2014)


50

40
(a) 30 degree
30

(b) 28 degree
20

10

(c) 24 degree 0

(d) 32 degree

39. Braiden heated up some pizza in the microwave. Before he put it in, it was 22° F.
When he got it out it was 77°F. How much did the microwave heat it up? (2015)
(a) 55°F (b) 56°F (c) 54°F (d) 57°F
Directions (Qs. 40 to 42) : Below is the bar graph of the average monthly temperature.
Study the graph and answer the questions. (Critical Thinking)

100

80

60

40

20

0
Jan Feb March April May June July Aug Sep Oct Nov Dec

40. Which is colder ________ March or April? (2011)


(a) March (b) April
(c) Both are equally cool (d) None of these
41. July is ________ degree warmer than April.
(a) 30 (b) 40 (c) 38 (d) 42
159
Temperature
42. Which month had an average temperature of 78 degree?
(a) May (b) June (c) July (d) September
43. Match the equivalent value of Fahrenheit scale with the Celsius scale.
[Critical Thinking]

List I List II
A. 167 °F 1. 65° C
B. 149 °F 2. 55° C
C. 113 °F 3. 75° C
D. 131 °F 4. 45° C

A B C D
(a) 3 1 4 2
(b) 2 1 3 4
(c) 1 2 3 4
(d) 4 1 3 2
44. James was sitting inside on snowy day and noticed it was 12°F outside. When he
looked a few hours later it was 21°F. How much did the temperature rise? (2016)
(a) 7°F (b) 11°F (c) 9°F (d) 33°F
45. The temperature at 7:00 AM was 61°F. The temperature at 11:30 AM was 38°F.
What was the change in temperature between 7:00 AM and 11:30 AM?
(a) 46°F (b) 99°F (c) 25°F (d) 23 °F
46. The average temperature for January was 35°F. The average temperature for
February was 9° warmer. What was the average temperature for February? (2011)
(a) 37°F (b) 24°F (c) 44°F (d) 48°F
47. Joey went to the park at 2:30 and it was 33°F. By the time she left at 4:30, it
was 18° warmer. What was the temperature when she left?
(a) 51°F (b) 50°F (c) 15°F (d) 49°F
48. The body temperature of a patient is 5.4 °F above the normal temperature. His
body temperature now in °C is: (2014)
(a) 38°C (b) 98°C (c) 50°C (d) 40°C
49. The temperature of first object is 36°C and that of second object is 36 °F. Which
is hotter than the other?
(a) First object (b) Second object
(c) Both have same hotness (d) Cannot be determined
50. Bianca made herself a cup of hot chocolate that was 79°F. After she put it in
the microwave the temperature rose by 38°. What was the temperature of hot
chocolate after she heated it? (2011)

(a) 111°F (b) 112°F (c) 117°F (d) 115°F


EBD_7364
160 Olympiad Champs–Mathematics

51. To convert from celsius to Fahrenheit, first ______________ by 180 , then add
100
_________
(a) multiply, 9/5 (b) multiply, 32 (c) add, 32 (d) convert, 32
52. The temperature of water in a swimming pool is 51ºF. Since the freezing point of
water is 32ºF, how many degrees would the temperature of the water have to drop
to reach the freezing point? (2010)
(a) 9ºF (b) 21ºF (c) 2ºF (d) 19ºF
53. The temperature of an object is found to be 167°F. What is its temperature in
°C?
(a) 102 °C (b) 75 °C (c) 167 °C (d) 98 °C
54. The maximum temperature on a day is 35 °C and the minimum temperature is
25 °C. The difference of these temperatures, on Fahrenheit scale is :
(a) 25°F (b) 75°F (c) 50°F (d) 100°F
55. The highest temperatures recorded in 4 different countries are listed below
(Tricky)

Highest temperature Country


127.6°F India
114.8°F Nepal
136.4°F Srilanka
134.0°F Pakistan

What is the highest temperature listed above?


(a) 127.6 °F (b) 114.8 °F (c) 136.4 °F (d) 134.0 °F
56. The temperature first rises by 18°C and then falls by 23°C. If the initial
temperature is 27°C. What is final temperature? (2017)
(a) 22°C (b) 23°C (c) 24°C (d) 25°C

RESPONSE GRID
1. a b c d 2. a b c d 3. a b c d 4. a b c d 5. a b c d
6. a b c d 7. a b c d 8. a b c d 9. a b c d 10. a b c d
11. a b c d 12. a b c d 13. a b c d 14. a b c d 15. a b c d
16. a b c d 17. a b c d 18. a b c d 19. a b c d 20. a b c d
21. a b c d 22. a b c d 23. a b c d 24. a b c d 25. a b c d
26. a b c d 27. a b c d 28. a b c d 29. a b c d 30. a b c d
31. a b c d 32. a b c d 33. a b c d 34. a b c d 35. a b c d
36. a b c d 37. a b c d 38. a b c d 39. a b c d 40. a b c d
41. a b c d 42. a b c d 43. a b c d 44. a b c d 45. a b c d
46. a b c d 47. a b c d 48. a b c d 49. a b c d 50. a b c d
51. a b c d 52. a b c d 53. a b c d 54. a b c d 55. a b c d
56. a b c d
161
Temperature

Solutions with Explanation


LEVEL 1

1. (b) Since there is snow outside, the most reasonable temperature would be the
freezing temperature of water i.e. 0°C.
2. (c) Degree is the odd one out as it is the unit of measurement of temperature whereas
all other are units of measurement of length.
3. (a) The temperature in the hot barbecue grill is very hot and intolerable by humans.
Thus, 260°C is the most reasonable answer.
4. (a) 65°F can be seen in the first thermometer.
5. (b) Thermometer is the instrument that is used to measure the temperature.
6. (c) As meter is the unit of measurement of length, similarly degree is the unit of
measurement of temperature. Celsius and Fahrenheit are the scales in which
temperature is measured.
7. (b) 77 - 32 = 45; 45 × 5 = 225 ; 225 ÷ 9 = 25°C
8. (a) 203 - 32 = 171; 171 × 5 = 855 ; 855 ÷ 9 = 95°C
9. (b) 86 - 32 = 54; 54 × 5 = 270; 270 ÷ 9 = 30°C
10. (d) 176 - 32 = 144; 144 × 5 = 720; 720 ÷ 9 = 80°C
11. (d) 212 - 32 = 180; 180 × 5 = 900 ; 900 ÷ 9 = 100°C

12. (a) 0 °C
13. (c)
14. (b) Scientiests
15. (c) 98.6°C
16. (c) 180

17. (d) 80°C = 80 × 9 = 720; 720 ÷ 5 = 144; 144 + 32 = 176°F.


9
18. (b) 75 × + 32 = 167°F
5
5
19. (c) (302 – 32) × = 150°C
9
20. (b) The temperature in the thermometer is below the normal human body temperature
because the normal body temperature of human body is 37°C and the temperature
in the thermometer is 33.6°C.
EBD_7364
162 Olympiad Champs–Mathematics

LEVEL 2

21. (c)

22. (a) As can be seen in the diagram, the minimum temperature is on 20 September i.e.
on Monday.

23. (c) The second highest peak is on 8th September, thus it is the second most hottest
day.

24. (b) As seen in the graph, the temperature is decreasing gradually and the days are
becoming cooler, thus there is a gradual decrease in temperature.

25. (a) (°C × 9/5) + 32 = °F is the correct conversion equation to convert Celsius into
Fahrenheit.

26. (a) temperature of machine when it is being used= 161°F; temperature of machine
when left unused for an hour=144°F, Drop in temperature of the machine = (161-
144) = 17°F.

27. (c) 25°

28. (d) 80 × 9 = 720 ; 720 ÷ 5 = 144 ; 144 + 32 = 176°F

29. (a) 95 × 9 = 855; 855 ÷ 5 = 171; 171 + 32 = 203°F

30. (c) 30 × 9 = 270 ; 270 ÷ 5 = 54; 54 + 32 = 86°F

31. (a) 15 × 9 = 135 ; 135 ÷ 5 = 27; 27 + 32 = 59°F

32. (d) Temperature of soda earlier = 47°F, temperature of soda after an hour = 21°
warmer = (47+21) °F = 68°F.

33. (b) The tip of the fluid shows the temperature. The level of fluid is between 30 and
40. And as seen there are 5 scales in between it, meaning each scale = 2 degree.
Thus the temperature as seen in the thermometer is 36 degree.

34. (a) The tip of the fluid shows the temperature. The level of fluid is between 0 and 10.
And as seen there are 10 scales in between it, meaning each scale= 1 degree. Thus
the temperature as seen in the thermometer is 4 degree.
163
Temperature
35. (b) The tip of the fluid showing the temperature is at 40 degree. Thus the temperature
that is shown by the thermometer is 40 degree.

36. (a) The tip of the fluid shows the temperature. The level of fluid is between 90 and
100. And as seen there are 5 scales in between it, meaning each scale= 2 degree.
And the fluid ends between the 3rd and 4th scale i.e. between 96 and 98 degree.
Thus the temperature as seen in the thermometer is 97 degree.

37. (c) The tip of the fluid shows the temperature. The level of fluid is between 60 and
70. And as seen there are 5 scales in between it, meaning each scale= 2 degree.
Thus the temperature as seen in the thermometer is 68 degree.

38. (b) The tip of the fluid shows the temperature. The level of fluid is between 20 and
30. And as seen there are 10 scales in between it, meaning each scale= 1 degree.
Thus the temperature as seen in the thermometer is 28 degree.

39. (a) temperature of pizza before heating = 22° F; Temperature of pizza after heating
= 77° F; The microwave heat up the pizza by (77-22) ° F = 55° F.

40. (a) March is colder than April since its average monthly temperature is less than that
of April.

41. (b) The temperature of July is 80 degree and that of April is 40 degree. Thus July is
40 degree warmer than April.

42. (b) June had average monthly temperature of 78 degree as it can be seen that it is
slightly below 80 degrees.

43. (a)

44. (c) Rise in temperature= (21-12) °F = 9°F.

45. (d) change in temperature between 7:00 AM and 11:30 AM= Temperature at 7:00 AM
– Temperature at 11:30 AM = (61-38) °F = 23°F.

46. (c) The average temperature for January = 35°F.;The average temperature for
February = 9° warmer than the temperature of January. Average temperature for
February = (35 +9) °F = 44°F.

47. (a) Temperature at 2:30 = 33°F; temperature at 4:30 = 18° warmer than the
temperature at 2: 30 = (18 + 33) °F = 51°F.
EBD_7364
164 Olympiad Champs–Mathematics
48. (d) Temperature = 98.6°F + 5.4°F

= 104.0°F

= 104°F
5
(104 – 32) ×
= = 40°C
9
49. (a)

50. (c) Temperature of hot chocolate= (79 + 38) °F = 117°F.

51. (b)

52. (d)
5
53. (b) (167 – 32) ×
9
5
= 135 × 9 = 75°C

54. (c) 35°C – 25°C = 10°C


 9
10°C=  10 ×  + 32
 5

= 18 + 32 = 50°F.

55. (c) 136.4° F

56. (a) 22°C


Area and Perimeter 165

CHAPTER FOREWORD
Consider the following scenario :

Puja - "Only few days are left for Diwali. So we are planning to change the curtains, carpet
and paint of our house".

• What should be the required length of carpet and curtains?

• How much paint do we need?

• "These two questions need to be addressed before we start with the work."

Here, using curtain of area according to the shape of window, walls and floor would be beneficial.

The quantity of paint can also be calculated by knowing the total surface area of walls and
roofs.

Area refers to the quantity that represents the extent of a two-dimensional figure or shape
or planar lamina in the plane.

The word 'Perimeter' comes from the greek words 'peri' (around) and 'meter' (measure). It
is the distance around a two-dimensional shape, or the measurement of the distance around
something.

Area

Perimeter

After reading this chapter, you will learn how to calculate the area and perimeter of square,
rectangle, triangle, etc.
EBD_7364
166 Olympiad Champs–Mathematics

Chapter
10 Area and Perimeter

LEARNING OBJECTIVES
This lesson will help you to:—
Real-Life Example v understand the concept of area and perimeter of
polygons.
v Whenever we have to pave any
surface with tiles, we calculate v calculate the area, perimeter and volume of two
the area to be paved and the dimensional and three dimensional shapes.
area covered by one tile. This
gives the number of tiles to be
used. In fencing a garden we Q U I C K C O N C E P T R E V I E W
find the perimeter of the garden
which gives us the length of wire AREA is the space occupied by a closed figure.
to be used.
For example: the area covered by a carpet on the floor.
PERIMETER is the distance around a closed figure.
For example: when you take a round around your play
ground the distance covered by you in taking one round is
called the perimeter.
Area of Square
As we know that the square has all its four sides equal.
The formula for area of a square is side × side.
Side
Historical
Amazingpreview
Facts

v The uniform systems of Side Side


measurements seem to have
been created by Egypt,
Mesopotamia and Indus valley Side
people. Indus valley people For example: Calculate the area of a square with each
achieved great accuracy in
side of 5 cm.
measurement of length, time
etc. Then to calculate the area of the square we apply the
formula side × side.
5 × 5 = 25, so the area of the square will be 25.
Area and Perimeter 167
UNITS: While calculating the area of square we are
multiplying two numbers with same unit. Like in the
example above, 5 cm is multiplied by 5 cm so the
result will be in cm2. This means the area will be 25 Amazing
Amazing Facts Facts
2
cm . Whenever we find area of the square the units
will be square units, like m2, cm2 or inch2 etc. v If base of a triangle is doubled,
the area of the triangle also
Perimeter of Square gets doubled.
When we find the perimeter of square we add all the sides v When there are two rectangles
or we multiply the length of one side by 4. of same perimeter, closer the
For example: Calculate the perimeter of a square with length and the width, greater is
each side of 4 units. the area.
Perimeter of square = 4 × side For example, if there are three
So, 4 × 4 = 16 ⇒ The perimeter of square is 16 units. rectangles with sides:
The units of perimeter of a figure will be the same as that  l = 1; b = 5 perimeter is
of units of side of the figure. Like cm, m and inches. 2(1 + 5) = 2 x 6 = 12 units;
area is 1 × 5 = 5 sq units.
Area of Rectangle
 l = 2; b = 4 perimeter is
As we know there is length and breadth in a rectangle.
2(2 + 4) = 2 × 6 = 12 units ;
So, to calculate area of rectangle we multiply length by
area is 2 × 4 = 8 sq units.
breadth.
Length  l = 3; b = 3 perimeter is
2(3 + 3) = 2 × 6 = 12 units ;
area is 3 × 3 = 9 sq units.
In all of the above given
Breadth Breadth
rectangles, perimeter is same.
The rectangle in which the
difference between the sides
Length is least (case iii), that rectangle
So area of a rectangle is length × breadth has the greatest area.
For example to calculate the area of a rectangle with [Difference in sides — (i) 5 – 1 =
length = 5 cm and breadth = 9 cm, we multiply 5 by 9 4, (ii) 4 - 2 = 2, (iii) 3 - 3 = 0]
5 x 9 = 45, so the area of rectangle is 45 cm . 2 v Similarly if the areas of three
rectangles are equal, closer
Perimeter of Rectangle
the length and width smaller
If we want to calculate the distance around a rectangle we is the area. You can try this
shall add both length and breadth twice. So to calculate yourself!!!!!
the perimeter of a rectangle we add all the four sides.
⇒ Perimeter of rectangle = l + b + l + b where l = length
and b = breadth
= 2l + 2b
= 2(l + b)
So, the perimeter of rectangle = 2(l + b)
EBD_7364
168 Olympiad Champs–Mathematics
Triangle
1
Example: Find the area of a triangle Area of triangle =× base × height .
2
whose base is 6 m and the Height : In a triangle, the length of the perpendicular
corresponding altitude is 3 m. which is drawn from vertex to the opposite side is called
Solution : Area of the triangle height of the triangle.
Base : In a triangle, the length of the side of the triangle
1
= × 6 × 3 = 9 m2 on which perpendicular is drawn is called base.
2
Perimeter of a triangle is sum of lengths of three sides.
Units
Area ---sq units ---- cm2, m2 etc.
Perimeter ---single units ---cm, m etc.

VOLUME
Point to remember Holding capacity of a container is called volume.
v Volume of Cuboid
We can also calculate the length
= length × breadth × height
of sides if perimeter or area is
given. v Volume of Cube
Say we know the area of = side × side × side
rectangle and its length, then
to calculate breadth: v Volume is measured in cubic units.
Area = length × breadth Circumference of a Circle
⇒ Area/length = breadth The perimeter of a circle is called the circumference.
Similarly we can find the The distance between the centre and any point on the
other missing dimensions.
circle is called its radius. And the line segment passing
through the centre of the circle, whose end points lie on
the circle is called the diameter of the circle. The diameter
of a circle is twice the radius.

Finding Circumference
To find the circumference of a circle we must remember
Circumference 22
= 3.14 or .
diameter 7
This makes it much easier to find the circumference of a
circle. Measure the diameter of the circle and multiply it
22
by 3.14 or to get the circumference.
7
For example: The diameter of a circle is 5cm. Find the
circumference.
Circumference = 5 × 3.14 cm = 15.7 cm
5 cm
Area and Perimeter 169
Area of Some Common Figure
D C

(a) Parallelogram A B

= base × height
a
(b) Area of Trapezium
1 h
= [Sum of parallel sides] height
2
1
= (a + b)h b A
2
(c) Rhumbus

1
Area of rhumbus = (Product of diagonals) B D
2 O
1
= (AC) (BD)
2

C
EBD_7364
170 Olympiad Champs–Mathematics

Multiple Choice Questions


LEVEL 1
1. Calculate the area of rectangle if one square represents an area of 1 cm2.
(Mental Mathematics)


(a) 2 cm2 (b) 9 cm2 (c) 50 cm2 (d) 24 cm2
2. What is the area of rectangle if (Mental Mathematics)
Length = 2 m, Breadth = 5 m
(a) 12 m2 (b) 14 m2 (c) 10 m2 (d) 18 m2
3. Each side of square is of 10 m. What will be the area of the square?
(Mental Mathematics)
2
(a) 100 m (b) 90 m 2 2
(c) 34 m (d) 115 m2
4. The third side of the triangle if the perimeter of a triangle is 20 m and two of its
sides are 5 m and 4 m, is (Mental Mathematics)
(a) 11 m (b) 10 m (c) 12 m (d) 5 m
5. Find the missing side in the figure given below if the perimeter of the figure is 28 m.
(Mental Mathematics)
8m

8m
(a) 12 m (b) 3 m (c) 9 m (d) 6 m
6. Find the area of shaded region. (Each square = 1m2)

(a) 21 m2

(b) 35 m2

(c) 34 m2

(d) 13 m2
Area and Perimeter 171
7. The length and breadth of a rectangle are (3a + 2) and (2a – 1). Which of the
following represents its perimeter? (2008, Tricky)
(a) 2(5a – 1) (b) (5a + 1) (c) (5a – 1) (d) 2(5a + 1)
8. The difference between the length and the breadth of a rectangle is 8 cm and the
perimeter is 64 cm. Which of the following can be the length and breadth of this
rectangle?
(a) L = 12 cm, B = 4 cm (b) L = 20 cm, B = 8 cm
(c) L = 20 cm, B = 12 cm (d) L = 12 cm, B = 8 cm

9. The area of a triangle whose base is 12 cm and the Height twice the base, is
(2017)
(a) 144 sq. cm (b) 288 sq. cm
(c) 289 sq. cm (d) Can not be determined
10. How many small cubes of side 2 cm can be put in a cubical box of side 6 cm?
(Tricky)
(a) 9 (b) 12 (c) 27 (d) 611.
11. The breadth of a rectangle is increased by 2 units. Its perimeter is now increased
by? (2013, Critical Thinking)
(a) 2 units (b) 4 units (c) 8 units (d) 16 units
12. The area of a square is equal to the area of a rectangle of l = 8 cm and b = 2 cm.
What is the side of the square?
(a) 6 cm (b) 4 cm (c) 3 cm (d) 8 cm
13. The area of a square is 100 sq. cm. If the sides this square are increased by 10%
then what will be the area of new square? (2012)
(a) 121 sq. cm (b) 101 sq. cm (c) 81 sq. cm (d) 169 sq. cm

14. Area of a square is : (Mental Mathematics)


(a) product of all sides (b) sum of all sides
(c) side × side (d) 2 × side

15. A cuboid measures 24 m × 12 m × 16 m. How many cubes of side 8 m can fit in the
box? (2010, Tricky)
(a) 9 (b) 16 (c) 15 (d) 24
16. The area of a square is 25 square units and its side is 5. Find its perimeter. (2018)
(a) 25 (b) 20 (c) 50 (d) 40
17. What is the perimeter of a square if each side is 8.2 centimetres long ?
(a) 64.16 cm (b) 67.24 cm (c) 32.8 cm (d) 16.2 cm
EBD_7364
172 Olympiad Champs–Mathematics
18. Which of the following statement is INCORRECT? (2014, Tricky)
(a) A square has 5 equal sides
(b) A rectangle has 2 pair of equal sides
(c) A square has 4 right angles
(d) A rectangle has only 2 right angles

19. If a figure has sides 4 m, 2 m, 4 m and 2 m then which type of figure is this?

(a) Circle (b) Rectangle (c) Square (d) Triangle

20. Which geometric figure has six square faces? (2010)


(a) Cube (b) Cone (c) Cylinder (d) Sphere
(e) None of these

LEVEL 2
21. Calculate the area of rectangle given below if each square has an area of 4 m2.

(a) 80 m2 (b) 64 m2 (c) 40 m2 (d) 15 m2

22. Which of these figures represents the area of 24m2 if given that each block
= 2 m2

(a) (b)

(c) (d)

23. Area of rectangle is 32 m2. If the length is 8 m then breadth = __________ (2015)
(a) 20 m (b) 4 m (c) 10 m (d) 12 m
24. Match the following : (Critical Thinking)

List-I List-II
A. Area of rectangle 1. Side × side
B. Perimeter of triangle 2. Length × breadth
C. Area of square 3. 4 × side
D. Perimeter of square 4. Sum of the lengths of three sides
Area and Perimeter 173
A B C D
(a) 2 4 3 1
(b) 2 4 1 3
(c) 3 4 1 2
(d) 4 1 2 3

25. Find area of unshaded region if each box = 3 m2. (Tricky, 2009)

(a) 88 m2 (b) 60 m2 (c) 84 m2 (d) 180 m2

26. Look at the diagram of Shreya’s yard. Which expression can Shreya use to find the
perimeter of her yard? (2011, Tricky)
8m
4m 4m

4m 4m
8m
(a) (4 × 4) + (2 × 8) (b) (4 × 8) + (2 × 8)
(c) (4 + 4) + (2 + 8) (d) (4 × 8) + (2 × 4)

27. Which of the following statement(s) are true or false? Choose the correct option.
(Critical Thinking)
A. The perimeter of triangle = 2 × sum of three sides.

B. Area of square = side x side.

C. If we want to fence an area we calculate perimeter of that place.

D. A rectangle has all its four sides equal.

(a) TTTT (b) TFFT (c) FTTF (d) TFFT


EBD_7364
174 Olympiad Champs–Mathematics
28. Find the perimeter of the given figure. (2015)

5 cm

4 cm 8 cm
6c m 3 cm
m
4c
5 cm 2 cm

12 cm

(a) 42 cm (b) 45 cm (c) 49 cm (d) 53 cm

Directions (Qs. 29 to 32) : Fill in the blanks in the given passage by choosing the correct
option.

Jayant has a garden of 20feet × 25feet. He wants to have a grass lawn on some part of it.
He wants the grass lawn to be 10feet by 13feet. To find area of grass lawn he should use the
formula ______ (29) he also wants to fence the whole garden. For that Jayant will have to
calculate the ______ (30) of the garden which is = ______ (31) . So he will have a grass lawn
of area ______ (32) sq feet.

29. (a) side × side (b) l + b (c) l × b (d) none of these

30. (a) area (b) perimeter (c) both (d) none of these

31. (a) 2l + b (b) 2b + l (c) 2(l + b) (d) 2l

32. (a) 130 (b) 140 (c) 124 (d) 80


33. Reema wants to spread the carpet on the floor of dimensions 13 m × 14 m. If the
cost of spreading the carpet is ` 14 per sq m. calculate the cost of carpeting the
area. (2011)

(a) 2455 (b) 2548 (c) 2344 (d) 500


34. Which of the following statement(s) are true or false? Choose the correct option.
Statement A : The measure of the amount of surface enclosed by a closed bounded
figure is called its area.
Statement B : The area of a square with each side measuring 1 m is 1 square metre, written as
1 sqm. or 1 m2.
Statement C : A rectangle can always be divided into a number of squares, each having
a unit area.
Statement D : While finding the area of a rectangle, its length and breadth should be
taken in the same units of length. (Critical Thinking)
(a) TTFF (b) FFFF (c) TTTT (d) FFTT
Area and Perimeter 175
35. Find the area and perimeter of the figure: (2015)
6m
A B

10 m

C 16 m D
2m

F E

(a) A = 25 m2, P = 34 m (b) A = 19 m2, P = 34 m

(c) A = 104 m2, P = 68 m (d) none of these

36. Which of the following statement(s) are true or false? Choose the correct option.
Statement A : Volume of a cuboid = length + breadth + height
Statement B : The measure of the amount of space occupied by a solid is called its
volume.
Statement C : Volume of a cube = (side × side × side) cubic units
Statement D : Every solid body occupies a certain amount of space.
(a) FTFT (b) FTTT (c) TFTT (d) TTFT
37. If each square has perimeter 8 centrimetres, then the perimeter of the following
shape is ______. (2011)

(a) 72 cm (b) 54 cm (c) 36 cm (d) 18 cm


38. Consider the following two statements. (Critical Thinking)
Statement A : Perimeter is defined as the length of boundary line of a close geometrical
figure.
Statement B : Holding capacity of a container is called volume.
Which one of the following option is correct about the above statements ?
(a) Only A is correct.
(b) Only B is correct
(c) Both A and B are correct.
(d) Neither A nor B are correct.
EBD_7364
176 Olympiad Champs–Mathematics
39. Rectangle ACEG below is divided into 4 parts. BCDX is square. What is the area of
shaded part? (2014, Tricky)

B
A 2 2 C
16 cm 4 cm
H D
X

2
8 cm
G E
F
(a) 28 cm2 (b) 32 cm2 (c) 48 cm2 (d) 64 cm2
40. Consider the following two statements.
Statement A : If edge of a cube measures 25 m and l = 20 m, b = 20m, h = 30 m are the
measures of a cuboid then cuboid has the larger volume.
Statement B : Dilip breaks a rod of 90 cm into four equal pieces. Taking three parts he
makes a triangle by joining end to end. Perimeter of the triangle is 67.5 cm.
Which one of the following option is correct about the above statements?
(a) Statement A is false and B is correct.
(b) Statement A is true and B is false.
(c) Both statements are true.
(d) Both statements are false.
41. On the basis of following features identify the correct shape. (Tricky, 2015)
(A) It's length and breadth are equal.
(B) It's area is side × side.
(C) It's perimeter is 4 times its side.
(D) It is a four-sided polygon.
(a) Rectangle (b) Parallelogram (c) Square (d) Trapezium
42. The perimeter of an equilateral triangle and of a square measure the same. Which
of the following can be the measures of the side of the equilateral triangle and of
the square?
(a) Equilateral triangle = 9 cm; square = 12 cm
(b) Equilateral triangle = 12 cm; square = 9 cm
(c) Equilateral triangle = 15 cm; square = 10 cm
(d) Equilateral triangle = 6 cm; square = 4 cm
43. Find the area and Perimeter of the given shape. 2 (2014)

(a) Area 40; Perimeter 40


(b) Area 36; Perimeter 28
8
(c) Area 28; Perimeter 36
5
(d) Area 30; Perimeter 40

6
Area and Perimeter 177
44. Find the missing side length, when the area is 90 square units.
(a) 10 ?

(b) 8
Area = 90
10 square unit.
(c) 9

(d) 7
45. On the basis of following features identify the correct shape. (2016, Tricky)
(A) It is a four-sided polygon in which opposite sides are equal.
(B) It's perimeter is twice the sum of its length and breadth.
(C) It's area is calculated by multiply its length and breadth.
(a) Rectangle (b) Parallelogram (c) Square (d) Trapezium
46. Read the statement and choose the correct option. (Tricky)
Statement A : The side of a square is 3. So its area is 9 square unit.
Statement B : The Perimeter of a square is 12 with side length 3.
(a) Statement A is right B is wrong. (b) Statement B is right A is wrong.
(c) Both the statements are wrong. (d) Both the statements are right.
47. The area of a square is 64 cm2. Then its perimeter is (2011)
(a) 64 cm (b) 32 cm (c) 46 cm (d) 48 cm
48. The perimeter of two squares is 12 cm and 24 cm. The area of the bigger square
is how many times that of the smaller?
(a) 5 times (b) 2 times (c) 3 times (d) 4 times
49. Find the difference in perimeters between the two figures. (Figures are not drawn
to scale) (2012, Tricky)

3 cm 4 cm
5 cm 7 cm
6 cm 3 cm
4 cm
5 cm

4 cm

3 cm

(a) 5 cm (b) 6 cm (c) 8 cm (d) 2 cm


50. In the given figure, the area of the shaded square is 196 cm2. What is the
perimeter of the rectangle EFGH? (2014, Critical Thinking)
E F

H G

(a) 90 cm (b) 54 cm (c) 98 cm (d) 49 cm


EBD_7364
178 Olympiad Champs–Mathematics
51. In the figure, side of each square is 1 cm. The area, in square cm, of the shaded
part is

(a) 11

(b) 8

(c) 9

(d) 10
2
52. In the given figure, AB is as long as BC and DE = EF = FG. (2015, Tricky)
3
30 cm
A B C
14 cm

8
6 cm

cm

G F E D

What is the perimeter of BCDE?


(a) 48 cm (b) 50 cm (c) 52 cm (d) 28 cm
3
53. The breadth of a rectangular painting is of its length. If the length of the
4
painting is 80 cm, what is the perimeter of the painting? (2015, Tricky)
(a) 140 cm (b) 190 cm (c) 280 cm (d) 380 cm
54. Find the area and Perimeter of the given shape. (2015)
6

(a) Area 19; Perimeter 20
(b) Area 20; Perimeter 20
4
(c) Area 19; Perimeter 19
1
(d) Area 20; Perimeter 19
1
55. Find the area of the shaded part. (2011, Tricky)

A 4 cm 5 cm B
2 cm X 1 cm

2 cm Y 3 cm

D 6 cm 3 cm C

(a) 9 cm2 (b) 4 cm2 (c) 23 cm2 (d) 36 cm2


Area and Perimeter 179
56. Find the area of rectangular garden of length 18 m and breadth 10 m. Also, find
the cost of manuring the garden at the rate of `5 per m2. (2014)

(a) 180 m2, ` 1800 (b) 180 m2, ` 900

(c) 28 m2, ` 280 (d) 14 m2, ` 450

57. Which shape has the largest area? (2012, Tricky)

(Area of 1 = 3 sq. units)

(a) Q (b) R (c) P (d) S

58. The given figure is made up of identical squares. The total area of the figure is 576
cm2. What is the perimeter of the figure? (2016, Critical Thinking)

(a) 320 cm (b) 160 cm (c) 240 cm (d) 200 cm

59. Find the perimeter of the given figure. (Two rectangles are of same shape and size)
(FG = FH = GH) (2013, Tricky)

10 cm
A B
35 cm

E F
C H
D G

(a) 165 cm (b) 150 cm (c) 135 cm (d) 170 cm


EBD_7364
180 Olympiad Champs–Mathematics
60. The given figure is made up of a square and a rectangle. The breadth of the
rd
1
rectangle is   the length of the edge of the square. If the area of the whole
3
figure is 384 cm2, find the length of the rectangle.
(2016, Tricky)
18 cm

(a) 12 cm (b) 8 cm
(c) 10 cm (d) 9 cm

RESPONSE GRID
1. a b c d 2. a b c d 3. a b c d 4. a b c d 5. a b c d
6. a b c d 7. a b c d 8. a b c d 9. a b c d 10. a b c d
11. a b c d 12. a b c d 13. a b c d 14. a b c d 15. a b c d
16. a b c d 17. a b c d 18. a b c d 19. a b c d 20. a b c d
21. a b c d 22. a b c d 23. a b c d 24. a b c d 25. a b c d
26. a b c d 27. a b c d 28. a b c d 29. a b c d 30. a b c d
31. a b c d 32. a b c d 33. a b c d 34. a b c d 35. a b c d
36. a b c d 37. a b c d 38. a b c d 39. a b c d 40. a b c d
41. a b c d 42. a b c d 43. a b c d 44. a b c d 45. a b c d
46. a b c d 47. a b c d 48. a b c d 46. a b c d 46. a b c d
46. a b c d 46. a b c d 46. a b c d 49. a b c d 50. a b c d
51. a b c d 52. a b c d 53. a b c d 54. a b c d 55. a b c d
56. a b c d 57. a b c d 58. a b c d 59. a b c d 60. a b c d
Area and Perimeter 181

Solutions with Explanation


LEVEL 1

1. (d) There are 6 × 4 = 24 squares and each square has an area of = 1 cm2.

So, the area of rectangle = 6 x 4 × 1 = 24 cm2.

2. (c) The area of rectangle is given by length × breadth. So the area is 2 × 5 = 10 m2.

3. (a) Area of square = side × side = 10 × 10 = 100 m2.

4. (a) Perimeter of a triangle = 1st side + 2nd side + 3rd side

So, 20 = 5 + 4 + 3rd side

3rd side = 20 - 9 = 11 m.

5. (d) Given figure is a rectangle. Also, given l = length = 8 m

Let b be the breadth of the rectangle.

Perimeter of rectangle = 2(l + b) = 28

⇒ l + b = 28/2

⇒ l + b = 14 ⇒ b = 14 - 8 = 6 m.

6. (a) Calculate no. of shaded boxes. It comes out to be 21. And each box has area of
1 m2. So the area of shaded region becomes 21 m2.

7. (d) Perimeter = 2[3a + 2 + 2a – 1]

= 2[5a + 1]

8. (c) Length of rectangle = x cm

Breadth of rectangle = (x – 8) cm

Perimeter of rectangle = 2(x + x – 8)

= 2(2x – 8)

Given 2(2x – 8) = 64

⇒ 2x = 40 ⇒ x = 20

L = 20, B = 20 – 8 = 12
EBD_7364
182 Olympiad Champs–Mathematics
9. (a) Base of ∆ = 12 cm

Height of ∆ = 2 × 12 = 24 cm
1
Area of ∆= × 12 × 24= 144 sq. cm.
2
10. (c) 11. (b) 12. (b)

13. (a) Given area of square = 100

side × side = 100 = 10 × 10

⇒ side = 10 cm

Increased in side of square = 10% of 10 = 1

∴ New side = 10 + 1 = 11

∴ Area of new square = (11)2 = 121 sq. cm.

14. (c) 15. (a) 16. (b)

17. (c) Perimeter of a square = 4 × side

∴ Perimeter = 4 × 8.2 = 32.8 cm.

18. (d) Rectangle has 4 right angles and not 2 right angles

19. (b) There are four sides .So it cannot be circle or triangle. Now, it has 2 pairs of equal
sides. So it is a rectangle as the square has all its four sides equal.

20. (a) Cube has 6 square faces

LEVEL 2
21. (c) There are 2 × 5 = 10 squares and each square has an area of 4 m2

So, the area of rectangle = 2 × 5 × 4 = 40 m2.

22. (c) Given each block is of 2 m2, so there should be 12 blocks in a figure to get an area
of 24 m2. In option (c), there are 2 × 6 = 12 blocks in figure.

23. (b) Given, area = length × breadth ⇒ breadth = area ÷ length

So, the breadth = 32/8 = 4 m.

24. (b)
Area and Perimeter 183
25. (d) No. of shaded boxes = 28,

no. of unshaded boxes = 60

Area of 1 unshaded box = 3 m2

So, area of unshade region = 60 x 3 = 180 m2.

26. (a) (4 × 4) + (2 × 8) represents perimeter of the given figure

27. (c)

28. (c) Perimeter = 12 + 5 + 6 + 4 + 4 + 5 +8 +3 + 2 = 49 cm

29. (c) Since it is a rectangular grass lawn the formula l × b should be used.

30. (b) To calculate length of the wire for fencing perimeter is calculated.

31. (c) Since it is a rectangle, the perimeter of rectangle = 2(l + b).

32. (a) Area of grass lawn = l × b = 10 × 13 = 130sq feet.

33. (b) Area of floor = 13 × 14 = 182 m2

Cost of carpeting = 182 × 14 = ` 2548.

34. (c) All the given statements are true.

35. (c) Calculate the lengths of the sides which are not given :

AB = 6 m, BC = 10 m, CD = 16 m, DE = 2 m

Now, FE = CD + AB = 16 + 6 = 22 m

AF = BC + DE = 10 + 2 = 12 m
6m
A B

10 m

C 16 m D
G
2m

F H E

Required area = Area of rectangle ABCG + area of rectangle GDEF

= 6 x 10 + 2 x 22 = 60 + 44 = 104 m2

PERIMETER = Sum of all sides = 6 + 10 + 16 + 2 + 22 + 12 = 68 m.


EBD_7364
184 Olympiad Champs–Mathematics

36. (b) Only first statement is false. Correct statement is volume of cuboid
= length × breadth × height.

37. (c) Perimeter of one square = 8 cm

8
Side of square = = 2 cm
4
Perimeter of given shape = 18 × 2 = 36 cm.

38. (c) Both statements are correct.

39. (b) Area of shaded part = 4 × 8 = 32 cm2

40. (a) Statement A is false. In statement A cube has the larger volume.
90
Statement B : Each part
= = 22.5 cm.
4
\ Perimeter of triangle = 3 × (22.5) cm = 67.5 cm.

41. (c) Given features indicates that the required shape is square.

42. (b) (a) Perimeter of equil. D = 9 × 3 = 27

Perimeter of square = 12 × 4 = 48.

(b) Perimeter of equil. D = 12 × 3 = 36

Perimeter of square = 9 × 4 = 36.

(c) Perimeter of equil. D = 15 × 3 = 45

Perimeter of square = 40.

43. (b) 44. (c)

45. (a) Given features shows that the shape is rectangle.

46. (d)

47. (b) Area of square = 64

(side)2 = 64

∴ side = 8 cm

Perimeter = 4 × side = 4 × 8 = 32 cm.


Area and Perimeter 185
48. (d) Perimeter of smaller square with side

a1 = 12 cm

⇒ 4a1 = 12

⇒ a1 = 3 cm

Area of smaller square = (a1)2 = (3)2 = 9 cm2

Perimeter of bigger square with side a2 = 24 cm

⇒ 4a2 = 24

⇒ a2 = 6 cm

Area of bigger square = (a2)2 = (6)2 = 36 cm2.

The area of bigger square is 4 times that of smaller square.

49. (b) As 44 – 38 = 6 cm
196
196 14
50. (c) length of side of square = = = 7 cm
22 2
Perimeter of EFGH = 14 × 7 = 98 cm

51. (a)

52. (b) BC = 18 cm, DE = 10 cm , BE = 8 cm, DC = 14 cm

So, perimeter of BCDE = 50 cm

53. (c) Perimeter = 60 + 60 + 80 + 80 = 280 cm

54. (a)

55. (c) Area of rectangle ABCD = (5 + 4) × (3 + 1) = 9 × 4 = 36 cm2

Area of triangle X = × 4 × 2 = 4 cm2

Area of square y = 3 × 3 = 9 cm2.

\ Shaded area = area of ABCD – area of X – area of Y

= 36 – 4 – 9 = 23 cm2.
EBD_7364
186 Olympiad Champs–Mathematics
56. (b) Area = 18 × 10 = 180 m2

cost of manuring = 180 × 5 = `900

57. (a) Q has largest area

58. (b) 160 cm

59. (d) Perimeter = 170 cm

60. (c) 10 cm
187
Geometry

CHAPTER FOREWORD
Consider the following case:
Ravi, Kavi and Lavi have a packet of chips each. But when they opened it, they found that each
packet had chips of different shape.

(Ravi) (Kavi) (Lavi)


Now try answering these questions:
• Do you know name of these shapes?
• Which shape contains same angles?
Like this there are many more examples of 2D as well as 3D shaped figures around us. List 2
shapes of each of the following kind.
2-D shape 3-D shape
1. ____________________________ 1. ________________________

2. ____________________________ 2. ________________________

Geometry is a branch of mathematics concerned with questions of shape, size, relative position
of figures and the properties of space .

2-D shape 3-D shape symmetry

• Can you give an example of real life symmetry?

The word geometry is taken from Greek word 'geometron' in which geo means 'earth' and
metron means measurement.

After reading this chapter, you will learn about different kinds of angles, 2-D and 3-D shapes
and the concept of symmetry
EBD_7364
188 Olympiad Champs–Mathematics

Chapter
11 Geometry
LEARNING OBJECTIVES
Real-Life Examples This lesson will help you to:—

v The intersection of four roads v be able to identify and classify angles as acute, right
on a traffic signal is an example and obtuse angles.
of right angle. v be able to identify right angles in real world
v The Taj Mahal is a fine example situations.
of symmetry. v explore perspective while drawing 3D object in 2D.
v be able to explore rotations and reflection in 2D
shapes.
v be able to explore symmetry and nets in 3D shapes.

QUICK CONCEPT REVIEW

ANGLES

An angle is a measure of a turn. It is measured in degrees


(°). There are different types of angles:

Acute angle : If the measure of an angle is less than 90°,


then it is called an acute angle.
A

Rays and Vertex of an Angle


P
B C

1 ∠ABC is an acute angle.


Q R
Right angle: If the measure of an angle is 90°, then it is
For ∠PQR or ∠RQP or ∠Q or ∠1 called a right angle.
vertex is Q and rays or arms are
QP and QR
189
Geometry

A
Reflex Angle
Reflex angle : An angle greater
than 180° but less than 360°' is
called as reflex angle.
B C A

∠ABC is a right angle.

Obtuse angle: If the measure of an angle is greater than B C


90°, then it is called an obtuse angle.

Amazing
Amazing Facts
Facts

v ARCHITECTURE is based on
B C angles and lines. They use the
concept of angles to design and
∠ABC is an obtuse angle. then bring the buildings to life.
v Things that are shaped like
TRIANGLES
cube are often referred as
A triangle is a closed plane figure, made up of three line 'cubic'.
segments.
v Most dice are in the shape of
A triangle has 3 sides, 3 angles and 3 vertices.
a cube with numbers 1 to 6 on
The sum of the angles of a triangle is always equal to
each face.
180°
Thus, BAC + ABC + ACB = 180° v 20° is approximately the width
of a handspan at arm's length.
BAC, ABC and ACB can also be written as A, B and
C respectively.

A + B + C = 180°
\
A

B C

Types of Triangles Based on Sides


There are three types of triangles based on their sides.

Equilateral triangle:

A triangle whose all the three sides are of equal length is


called an equilateral triangle.
EBD_7364
190 Olympiad Champs–Mathematics
In DPQR, PQ = QR = PR = 2 cm.

Therefore, ∆PQR is an equilateral triangle.

All the angles of an equilateral triangle are equal.


P

2 cm 2 cm

Q R
2 cm
Isosceles triangle:
Historical
Amazing Facts
Preview
A triangle whose only two sides are equal in length is
v The first known instrument called an isosceles triangle.
for measuring angles was the
Egyptian Groma. It consisted In D XYZ, XY = XZ.
of 4 stones hanging by cords
Therefore, ∆ xyz is an Isosceles triangle.
from sticks set at right angles.
X

Y Z

Amazing
Remember
Facts Scalene triangle:

v An equilateral triangle has A triangle whose all the three sides are of different
three sides of equal length and length is called a scalene triangle.
two equal lengths.
Sides AB, BC and AC of ∆ABC are of different lengths.
v The Longest Side of right A
angled triangle is called the
hypotenuse, it is always found
opposite the right angle.

B C
191
Geometry

Types of Triangles Based on their Angles


Acute angled triangle:
A triangle whose each angle is less than 90° is called an Do
Amazing
You Know?
Facts
acute angled triangle.
Each angle of the ∆XYZ is less than 90°, therefore, v A right triangle that has its
∆XYZ is an acute angled triangle. two legs equal in length is called
an isosceles right triangle.
X

Y Z

Right angled triangle:


A triangle whose one angle is 90° is called a right-angled
triangle.
In ∆ ABC, ∠B = 90°, therefore, D ABC is a right-angled
triangle.
A Amazing
Remember
Facts

v The sum of the lengths of any


two sides of a triangle is always
greater than the length of the
third side.
90°
B C

Obtuse-angled triangle:

A triangle whose one angle is greater than 90° is called


an obtuse angled triangle.
P

140°

Q R
In ∆PQR, ∠PQR is greater than 90°.

Therefore, ∆PQR is an obtuse angled triangle.


EBD_7364
192 Olympiad Champs–Mathematics
Example: Find the missing angle in the following triangle.
A

70°

Amazing
Try It!Facts
60° ?
v Which of the following cannot B C
be the measure of three angles
of a triangle? Solution: Sum of angles of a triangle = 180°
(i) A = 60°, B = 60°, C = 60° or  ∠A + ∠B + ∠C = 180°
(ii) A = 90°, B = 90°, C = 90°
⇒  70° + 60° + ∠C = 180°
(iii) A = 70°, B = 20°, C = 100°
Solution : ⇒  130° + ∠C = 180°
(i) Sum = 60° + 60° + 60° = 180° ⇒  ∠C = 180° – 130° = 50°
These can be the angles of a
triangle. Example: Check whether the following triangle is
(ii) Sum = 90° + 90° + 90° = 270° possible or not:
These cannot be the angles A
of a triangle.
(iii) Sum = 70° + 20° + 100° = 190°
40°
These cannot be the angles
of a triangle.
120°
30°
C B

Solution: Here, ∠A + ∠B + ∠C = 40° + 120° + 30° = 190°


But, sum of angles of a triangle is always 180°.

DABC is not possible.

SYMMETRY

Symmetry is when one shape becomes exactly like another


if it is rotated, reflected or translated.
193
Geometry
The vertical line in the figure above is called line of
symmetry.

3D shapes have faces, edges and vertices. A net of a


3D shape is a figure which folds up to form a 3D shape.
For example:
Do You Know?
Cuboid is similar to the cube in the
sense that it also has 6 faces but
it is different from cube, as cube
has 6 square faces but cuboid has
6 rectangular faces.

Net of a cube folds up to form a cube.

Properties :
v There are 360° in a full turn.
v A protractor can be used to measure an angle.
v Angles can be observed all around us.
v A shape can be turned clockwise or counter clockwise
about a point. It is then said to be rotated. Misconcept/Concept
v A shape can be reflected in a mirror line.
Misconcept: Students tend to find
v There are several possible nets of a 3D shape.
more lines of symmetry than
For example : actually exist.
Concept: The line of symmetry
divides the object such that it
forms two parts that are mirror
image of each other.
Misconcept: Children associate the
word 'right' with directional
language, i.e., a right angle and
These are two different nets that form cubes. a left angle.
Concept: An angle of measure 90°
GAME is called right angle. It has
nothing to do with direction.
1. Get two match sticks from a match box. The
teacher creates angles of different measures from
the sticks. The teacher then asks the students to
classify the angles as acute, right or obtuse angles.
EBD_7364
194 Olympiad Champs–Mathematics
2. The teacher will show different objects in class such as pencil box, eraser, book etc., and
ask the students if the objects have a line of symmetry.
Cylinder, Cone and Sphere

Cylinder has 2 circular


faces and no vertex

A Cone has one vertex

A Sphere has no vertex


195
Geometry

Multiple Choice Questions


LEVEL 1
1. Triangle : Isosceles :: Angle : __________ (Mental Mathematics)
(a) Scalene (b) Acute (c) Square (d) Ray
2. Polygon : Square :: Symmetry : __________ (Mental Mathematics)
(a) Rotational (b) Shape (c) Angle (d) Line
3. Which of the following has a right angle? (Mental Mathematics)

(a) (b)

(c) (d)

4. Which of the following road symbols has a right angle? (Mental Mathematics)

(a) (b)

(c) (d)

5. Which shows reflection for the figure given below? (Mental Mathematics)

(a) (b)

EBD_7364
196 Olympiad Champs–Mathematics

(c) (d)

6. Which shows rotation for the figure given below?

(a) (b)

(c) (d)

7. The shape below is rotated in counter clockwise direction. Which of the following
1
shows figure A after turn? (Mental Mathematics)
4

Figure A

(a) (b) (c) (d)


8. How many lines of symmetry does the figure A have? (Mental Mathematics)

Figure A
(a) 1 (b) 2 (c) 3 (d) 4
197
Geometry
9. How many lines of symmetry does figure A has? (Mental Mathematics)

(a) 0 (b) 1 (c) 2 (d) 3


10. Which is odd one out?


Figure A Figure B Figure C Figure D
(a) Figure A (b) Figure B (c) Figure C (d) Figure D
11. Which is the closest measure of angle shown below?

12º

(a) 170° (b) 160° (c) 10° (d) 20°


12. What is the closest measure of the angle shown?
104º

(a) 70° (b) 80° (c) 110° (d) 100°


13. What type of angle is shown below?

B C

(a) Acute angle (b) Obtuse angle


(c) Right angle (d) None of these
EBD_7364
198 Olympiad Champs–Mathematics
14. What type of angle is shown below? (2008)

(a) Acute angle (b) Obtuse angle


(c) Right angle (d) None of these
15. Which holds true for angle below? (2010, Mental Mathematics)

B C

(a) ∠ABC is an obtuse angle and is greater than 90°.

(b) ∠ABC is a right angle and is equal to 90°.

(c) ∠ABC is an obtuse angle and is less than 90°.

(d) ∠ABC is an acute angle and is less than 90°.

16. Ravi makes some frame boards. In which board has he identified angle less than
90°?(2016)

(a) (b)

(c) (d)


17. Richa and Reena share a room that is regular hexagon in shape. What type of angle
is x?

(a) Acute angle (b) Obtuse angle


(c) Right angle (d) None of these
199
Geometry
18. Which of the following net will make a cone? (2011)

(a) (b) (c) (d)


19. Which of the following figure/net folds up to form a cube? (2011, Tricky)

(a) (b)

(c) (d)

20. Which of the following figure/net will fold up to form a cuboid?

(a) (b)

(c) (d)

21. Which of the following shape will be formed from the given net. (2013)

(a) Cube (b) Cylinder (c) Cone (d) Cuboid


EBD_7364
200 Olympiad Champs–Mathematics
22. What is the top view of the can shown below?

(a) (b)

(c) (d)

23. Which figure has no line of symmetry? (2017)

(a) (b) (c) (d)


24. Which shows all right angles of figure A below?

Figure A

(a) (b) (c) (d)


25. Which figure has only one base and one vertex? (2011)

(a) (b) (c) (d)


26. How many pictures have symmetry? (2017)


Figure A Figure B Figure C Figure D
(a) 0 (b) 1 (c) 2 (d) 3
201
Geometry
27. The given net represents which of the following shape? (2012)

(a) (b) (c) (d)


28. How many pictures does not have symmetry?


Figure A Figure B Figure C Figure D
(a) 0 (b) 1 (c) 2 (d) 3
29. How many of the given figures have right angles? (2013)

(a) 2 (b) 3 (c) 4 (d) 5


30. How many lines of symmetry does the figure have? (2013)

(a) 0 (b) 1 (c) 2 (d) 4

LEVEL 2
31. Shekhar wants to cut his chocolate brownie into two rectangles. What kind of
angles will Shekhar form?

(a) Acute angles (b) Obtuse angles (c) Straight angle (d) Right angles
EBD_7364
202 Olympiad Champs–Mathematics
32. State which of the following statements are TRUE or FALSE? (Critical Thinking)
Statement A: Acute angles are greater than obtuse angles in measure.
Statement B: Right angles are greater than 90°.
Statement C: Acute angles are smaller than right angles in measure.
Statement D: Obtuse angles and right angles are equal in measure.
(a) T F T F (b) F F T F (c) T T F T (d) FTFT
33. An angle measuring 270° is an example of (2014)

(a) acute angle (d) obtuse angle (c) right angle (d) reflex angle
34. What combination of transformation is shown below?

3 2

(a) Reflection and then rotation (b) Rotation and then reflection
(c) Reflection and then reflection (d) Rotation followed by rotation
35. What kind of triangles will be obtained if X draws a line segment connecting points
A and C on the rectangle shown aside? (2010, Tricky)
A B

D C
(a) Isosceles (b) Right angled (c) Equilateral (d) Obtuse
36. What combination of transformation is shown below?

3
1

(a) Rotation followed by rotation (b) Rotation followed by reflection


(c) Reflection followed by rotation (d) Reflection followed by reflection
203
Geometry
37. Which of the following shapes has more than 1 line of symmetry? (2016)

(a) (b) (c) (d)

38. Which of the following statement(s) is true?


Statement A: A square has 4 right angles.
Statement B: A circle has 1 acute angle.
Statement C: A triangle has 3 obtuse angles.
Statement D: A rectangle has 2 right angles and 2 acute angles.
(a) F T F T (b) T F F F (c) F T T T (d) TFTF
39. Which one of the figures is a symmetrical figure? (2014, Tricky)

(i)   (ii) (iii) (iv) (v)


(a) Both (i) and (iv) (b) Only (v)
(c) Both (ii) and (iv) (d) Only (iii)
40. What fraction of a turn in this angle?

1 1
(a) turn (b) turn
2 4
3
(c) turn (d) One full turn
4
41. Complete the table below:

Shape Number Number Number


of acute of right of
angles angles obtuse
angles
EBD_7364
204 Olympiad Champs–Mathematics

(a) Shape Number Number Number (b) Shape Number Number Number
of acute of right of of acute of right of
angles angles obtuse angles angles obtuse
angles angles

3 0 0 2 0 1

2 1 0 0 2 0

0 4 0 2 1 1

2 0 2 1 2 1

0 0 5 2 1 2

(c) Shape Number Number Number (d) Shape Number Number Number
of acute of right of of acute of right of
angles angles obtuse angles angles obtuse
angles angles

1 0 2 2 1 0

2 0 1 1 1 1

1 1 2 1 2 1

2 1 1 1 2 1

1 3 1 5 0 0

42. Which of the angles in the figure is larger than 2 right angles? (2014, Tricky)

c
d

(a) ∠a (b) ∠b (c) ∠c (d)


∠d
205
Geometry
43. Which of the statements is true for the angle below?

B C

Statement A : ∠ABC is an acute angle.


Statement B : ∠ABC is a right angle.


Statement C : ∠ABC is greater than 90°.


Statement D : ∠ABC is an obtuse angle.

(a) F T F F (b) F F T T (c) T F F F (d) FTFT

44. Sudha is trying to fit a bed into a corner of a room. She knows that the walls and
the floor of the room are perfectly straight. What is the measure of the angle of
a corner of the room?

(a) 0° (b) 90° (c) 60° (d) 120°

45. Ria runs around a field of the shape given below. Which is true for the angles
formed by the corners of the field? (Critical Thinking)
E
F
A

B
C
D

(a) ∠A and ∠B are right angles (b) ∠A and ∠B are acute angles

∠E and ∠D are acute angles ∠E and ∠D are obtuse angles

∠F and ∠C are obtuse angles ∠F and ∠C are right angles

(c) ∠A and ∠B are acute angles (d) ∠A and ∠B are right angles

∠E and ∠D are right angles ∠E and ∠D are obtuse angles

∠F and ∠C are obtuse angles ∠F and ∠C are acute angles


EBD_7364
206 Olympiad Champs–Mathematics
46. Match the following objects with their respective nets. (Tricky)

List-I (Objects) List-II (Nets)


A. (1)

B. (2)

C. (3)

D. (4)


A B C D
(a) (2) (1) (4) (3)
(b) (3) (1) (4) (2)
(c) (2) (4) (1) (3)
(d) (3) (4) (1) (2)

47. Which of the following is INCORRECT? (2015, Tricky)


(a) A rectangle has 4 right angles
(b) A square has 2 sets of parallel lines
(c) A rectangle has only 1 set of parallel lines
(d) A square has 4 equal sides
48. What is the front view of the solid below?

(a) (b)  (c) (d)


207
Geometry
1
49. How many right angles are there in 2 complete turns? (2014)
2
(a) 8 (b) 10 (c) 12 (d) 14
50. What is the top view of the solid given below? (Critical Thinking)

(a) (b)

(c) (d)

51. Angle X is equal to _________ right angle(s). (2015)

(a) 1 (b) 2 (c) 3 (d) 4


52. What is the top view of the solid given below? (Critical Thinking)

(a) (b)

(c) (d)
EBD_7364
208 Olympiad Champs–Mathematics
53. Danish drew an angle as shown here. Sarah drew an angle that was twice the
measure of Danish’s angle. What was the measure of Sarah’s angle? (2017)

(a) 20º (b) 70º (c) 80º (d) 60º


54. Which shows the side view of the solid given below?

(a) (b)

(c) (d)

55. A triangle has sides that measure 5 cm, 12 cm and 13 cm. Identify the type of
triangle. (2015)
(a) Equilateral triangle (b) Isosceles triangle
(c) Obtuse angled triangle (d) Right angled triangle
56. Which of the following is true for a triangle?
(a) All the angles are always acute angle (b) One angle is always a right angle
(c) One angle is always on obtuse angle (d) One angle is always an acute angle
57. In ∆ABC, BC = CA its two equal angles are (2011, Tricky)
(a) ∠B = ∠C (b) ∠A = ∠B
(c) ∠A = ∠C (d) ∠A = ∠B = ∠C
58. The sum of all interior angles of a square is
(a) 180° (b) 360° (c) 270° (d) None of these
209
Geometry
59. The angles of a triangle are in the ratio 2 : 3: 4. Find the measures of all angles
(2015)
(a) 40o, 60o, 80o (b) 30o, 60o, 90o
(c) 40o, 50o, 90o (d) None of these
60. The angles of a triangle are in the ratio 1 : 2 : 3. Then, all the angles of the
triangle will be :
(a) 30°, 60°, 90° (b) 50°, 60°, 70°
(c) 40°, 80°, 60° (d) None of these
61. The measures of the angles of a triangle are in the ratio of 3 : 6 : 9. What is the
measure of third angle? (2009)
(a) 90º (b) 0º (c) 120º (d) 150º
62. An equilateral triangle ABC is inscribed in a circle of centre as follows :
A

B C

∠BOC will be equal to :


(a) 60° (b) 90° (c) 120° (d) None of above
63. The time shown on Manu’s watch was 45 minutes slow. He adjusted the watch to set
it to the actual time. How many right angles had he formed by moving the minute
hand? Did he make a clockwise or anti-clockwise turn? (2013, Critical Thinking)
(a) 2 right angles, anti-clockwise (b) 2 right-angles, clockwise
(c) 3 right angles, anti-clockwise (d) 3 right-angles, clockwise
64. The two acute angles of a right angled isosceles triangle are : (2017)
(a) 45°, 45° (b) 60°, 60° (c) 70°, 70° (d) 80°, 80°
65. If the sum of two angles in a triangle equals third angle then the triangle is :
(a) Equilateral (b) Right angled (c) Obtuse angled (d) None of these
66. In a ∆ABC, AB = 11 cm, BC = 60 cm and AC = 61 cm. What type of ∆ABC?
(2016, Tricky)
(a) Acute angled triangle (b) Right angled triangle
(c) Obtuse angled triangle (d) None of the above
67. The number of degrees in four and one-third right angles is
(a) 405 (b) 390 (c) 395 (d) 400
68. Number of degrees in two and half right angles is __________ . (2015)
(a) 245 (b) 225 (c) 200 (d) 180
EBD_7364
210 Olympiad Champs–Mathematics
69. Number of degrees in five and two-third of a right angle is
(a) 510 (b) 490 (c) 486 (d) 480
70. Four girls each made a statement about the given figure. (2016, Critical Thinking)

Sanchi : “I can see 6 pairs of perpendicular lines”.


Trishika : “There are 7 angles in the figure”.
Sidak : “There is only one pair of parallel lines”.
Mini : “6 of the angles are less than a right angle”.
Which girl made the CORRECT statement?
(a) Sanchi (b) Trishika (c) Sidak (d) Mini

RESPONSE GRID
1. a b c d 2. a b c d 3. a b c d 4. a b c d 5. a b c d
6. a b c d 7. a b c d 8. a b c d 9. a b c d 10. a b c d
11. a b c d 12. a b c d 13. a b c d 14. a b c d 15. a b c d
16. a b c d 17. a b c d 18. a b c d 19. a b c d 20. a b c d
21. a b c d 22. a b c d 23. a b c d 24. a b c d 25. a b c d
26. a b c d 27. a b c d 28. a b c d 29. a b c d 30. a b c d
31. a b c d 32. a b c d 33. a b c d 34. a b c d 35. a b c d
36. a b c d 37. a b c d 38. a b c d 39. a b c d 40. a b c d
41. a b c d 42. a b c d 43. a b c d 44. a b c d 45. a b c d
46. a b c d 47. a b c d 48. a b c d 49. a b c d 50. a b c d
51. a b c d 52. a b c d 53. a b c d 54. a b c d 55. a b c d
56. a b c d 57. a b c d 58. a b c d 59. a b c d 60. a b c d
61. a b c d 62. a b c d 63. a b c d 64. a b c d 65. a b c d
66. a b c d 67. a b c d 68. a b c d 69. a b c d 70. a b c d
211
Geometry

Solutions with Explanation


LEVEL 1

1. (b) Isosceles triangle is a kind of triangle and acute angle is a type of angle.

2. (a) Square is a type of polygon. Rotation symmetry is a kind of symmetry.

3. (b) The window grill has right angles. All other objects do not have right angles.

4. (a) Only (a) has right angle others do not.

5. (c) Option (c) shows reflection about vertical line.

6. (a) Option (a) shows rotation.


1
7. (d) When figure A is rotated counter clockwise after turn it looks like option (d).
4

8. (a) The figure has 1 line of symmetry.

9. (a) The figure has no line of symmetry?

10. (c) Only figure C has line of symmetry. The remaining objects has no line of
symmetry.

11. (c) 12 is closest to 10. Hence (c) is the correct answer.

12. (d) Since 104° is closest to 100 so (d) is correct option. The angle shown is an obtuse
angle.
EBD_7364
212 Olympiad Champs–Mathematics

13. (b) The measure of ∠ABC is greater than 90° and is an obtuse angle.

14. (c) The measure of the angle is 90° and hence it is a right angle.

15. (a) An obtuse angle is greater than 90°. A right angle is equal to 90°.

An acute angle is less than 90°. Hence only A is true.

16. (d) Option (a) has all right angles.

Option (b) has obtuse angle.

Option (c) has right angle.

Option (d) has acute angle i.e. less than 90°.

17. (b) The angle x is more than 90° in measure and is hence an obtuse angle.

18. (b) Option (b) is a net for the cone.

19. (c) Option (c) folds up to form a cube.

20. (a) Option (a) forms a cuboid.

21. (b) The net folds up to form a cylinder.

22. (d) See the can from top to get option (d).

23. (c) Has no line of symmetry.

24. (b) The option (b) shows right angle (= 90°) of figure A.

25. (c) Cone has one base and one vertex.

26. (c) Figure C has symmetry. Figure D also has symmetry.

27. (b) Net represent a cylinder.

28. (b) Only figure (c) does not have symmetry.

29. (b) 3 figures have right angles.

30. (a) It has 0 lines of symmetry.


213
Geometry
LEVEL 2
31. (d) When rectangles are formed, the corners form right angles.

32. (b) A: Acute angles are less than 90° - hence false.

B: Right angles are equal to 90°, hence false.

C: True

D: Obtuse angles are greater than 90° and right angles are equal to 90°, hence
false.

33. (d)

34. (b) Figure 1 is rotated to get Figure 2. Figure 2 is reflected about vertical axis to get
Figure 3.

35. (b) Right angled triangles will be obtained

36. (a) Figure 1 is rotated to get Figure 2. Figure 2 is rotated to get Figure 3.

37. (b) Hexagon has more than 1 line of symmetry

38. (b) A: A square has 4 right angles. Hence true.

B: A circle has no angles. Hence false.

C: A triangle cannot have 3 obtuse angles.


Hence false.

D: A rectangle has 4 right angles. Hence false.

39. (b) only (V) figure is symmetrical.


1 1
40. (b) One full turn represents 360°, so a right angle is of 360° i.e. turn.
4 4
41. (a) Table in option (a) gives the correct no. of angles included in each shape.
42. (d) ∠d is a reflex angle
43. (c) A: ∠ABC is an acute angle – True.
B: ∠
 ABC is a right angle – False.
Since it is an acute angle so its measure will be less than 90°
C: False - due to reason given in B above.
D: An obtuse angle is greater than 90°, hence ∠ABC is not an obtuse angle.
EBD_7364
214 Olympiad Champs–Mathematics
44. (b) Since the walls and floor are straight, they form a right angle at the corner.

45. (a) ∠A = ∠B = 90° (Right angles)

∠F & ∠C > 90° (Obtuse angles)

∠E & ∠D < 90° (Acute angles)

46. (d)

47. (c) As rectangle has 2 sets of parallel lines

48. (a) The front view is faces of 5 cubes stacked on one another.

1
49. (b) As 1 turn has 4 right angles so 2 turn have 10 right angles
2
50. (b)

51. (c) ∠x = 270° = 3 × 90°

52. (a) See the solid from top to get option (a).

53. (b) 35° × 2 = 70°

54. (c) See the solid from side to get option (c).

55. (d) According the Pythagorean theorem


2
13= 52 + 122
169 =25 + 144 ⇒ 169 =169
These sides represent a right angled triangle.

56. (d)
57. (b) A

B C

BC = CA
∴ ∠A = ∠B
[Opposite angle of equal sides are equal]
58. (b) As each of the angles of a square is equal to 90o.
Sum of all interior angle of a square is 360o.
Geometry 215
59. (a) 2x + 3x + 4x = 180°

9x = 180°
x = 180° = 20°
9
Then, 2x = 2 × 20° = 40°

3x = 3 × 20° = 60°

4x = 4 × 20° = 80°

Hence, angles of triangle are 40°, 60° and 80°.

60. (b) Let the angles are x, 2x and 3x, then x + 2x + 3x = 180°
180°
 6x = 180° ⇒ x = = 30°
6
Hence, the angles are 30, 60°, 90°.
61. (a) Let the angles are 3x, 6x and 9x
3x + 6x + 9x = 180°
18x = 180°
x = 10º
Hence, third angle = 9x = 9 × 10° = 90°
62. (c) ∆ABC is equilateral ∆.

∠A = ∠B = ∠C = 60o

So, ∠BOC = 2 ∠A = 2 × 60o = 120o



[as central angle (∠O) is double of the inscribed angle (∠A)]

63. (d) 3 right angles, clockwise

64. (a) In a right angled ∆, one angle is equal to 90o.

So rest of the angles are 45o each.

65. (b) Let angles are x, y and z.


x+y=z
In any ∆, sum of interior angles is equal to 180°
x + y + z = 180°
z + z = 180°
2z = 180°
z = 90°
Therefore, the triangle is right angled ∆.
EBD_7364
216 Olympiad Champs–Mathematics
66. (b) According to Pythagoras theorem
612 = 112 + 602
 3721 = 121 + 3600
 3721 = 3721
Then, triangle is right angled triangle
1
67. (b) Four right angle + rd right angle
1 3
= 4 × 90 + × 90
3
= 360 + 30 = 390
68. (b) Two right angles = 2 × 90° = 180°
1
Half right angles = × 90°= 45°
2
Total two right angles and one half right angle
180°
+ 45°
= 225°

69. (a) Five and two-third of a right angle


2
= 5 × 90° + × 90°
3
= 450° + 60° = 510°
70. (a) Sanchi is correct as there are 6 pairs of perpendicular lines in the given figure.
Data Handling 217

CHAPTER FOREWORD
Ramya was in the school library when her Maths teacher came and asked about a book from
the librarian. The librarian searched something in the computer and then told the teacher
where the book was exactly placed.

Ramya was amazed as according to her it was not so easy to identify the exact location of a
book amongst thousands of books. But due to proper maintenance of records it was possible
for the librarians to easily locate any book.

It is very obvious that nowadays data is considered as a valuable resource. Hence data
handling and record keeping have also become important issues.
Data Handling

Table
Bar Graphs Bar Charts

Data handling is also important because now organisations are using large amount of data for
various purposes such as report generation, deciding future targets, analysing competitor's
market value and strength, etc. Such large data should be handled very carefully for
accurate results.

After reading this chapter, you will learn various methods to keep good record of collected
data.

Data handling is important to analyse, interpret and draw can conclusions from the given

data. This is processing of data and representing it as a table , bar graph, pie chart, etc is

very helpful in drawing conclusions which is other wise different from a raw data It is a very

important tool to make reports or policies for the future.


EBD_7364
218 Olympiad Champs–Mathematics

12
Chapter Data Handling

LEARNING OBJECTIVES
This lesson will help you to:—
Amazing Facts
Historical Preview
v understand how to group data.
v Statistics is the collection, v draw frequency distribution table.
organisation, analysis and
interpretation of data. it can v learn how to make bar graph.
be assumed to have started
around 1749. There have been QUICK CONCEPT REVIEW
many changes since then. In
the beginning it was just about There are so many things and works in our lives. We
collecting the data about all manage and collect the details of the work we do.
states but later on it started Every office keeps the records. We need to collect the
to include all collections of information and there is even more need to arrange
information of all types. that information. We need to do grouping of the data
collected. We cannot keep the information ungrouped as
it will be very difficult to retrieve any information at the
time of need.
We shall learn about some of the ways to group or to
handle the data.
(1) Making the table: The first step to group any data
is to make a table category wise.
For example: Rajesh threw a dice twenty times.
The outcomes were:
Try It!
5,3,1,1,5,6,3,2,4,4,5,5,5,2,1,6,5,3,6,6
Example: What frequency is being If we want to see which number appeared how many times
represent by l l l l l l l l lll it becomes a bit difficult to count each number. So we can
group them as follows:
Solution: l l l l Stands for 5
\Frequency First of all we see how many numbers are there which are
appearing. As we know there are 6 faces of the dice so
= 5 + 5 + 3 = 13
there will be numbers 1,2,3,4,5,6 which will be appearing.
Each time the number appears we put a mark | in front of
that number. So | means 1, || means 2, ||| means 3, ||||
means 4 and we represent 5 as |||| so 6 will be |||| | and
so on. These symbols are called tally marks.
Data Handling 219
So the table will be as follows: (This table is called
frequency distribution table.)
Number on the dice Tally marks Frequency
1 ||| 3
2 || 2 Some Terms
3 ||| 3
v The data which is given in
4 || 2 the beginning (like data in
the example above) is called
5 |||| | 6
ungrouped or raw data.
6 |||| 4 v Each entry in the list is called
TOTAL 20 observation.

Now if you are asked how many times the number 5


appeared, you can easily see from the table and tell that
the number 5 appears 6 times.
Now you can easily make a frequency distribution table Primary Data and
for the marks obtained or the number of different types Secondary Data
of fruits that different people like.
Primary data is the data collected
(2) Bar Graphs: It is the diagramatic representation by the individual himself while
of the tabular information. the secondary data is collected
For example: If we have information about the by some one else word by another
monthly expenditure on different items in a house, person Normally, Primary data is
we can represent the information through a bar considered to be more reliable as
graph. the individual collects it himself
and so the probability of error is
Items Food Education Clothes Travelling Savings less.
Expenditure 1500 1000 1200 500 1000
The bar graph representing the above information will
be :

1500
1400
1300
1200
1100
Expenditure

1000
900
Food

800
Clothes

700
Education

Savings

600
500
Travelling

400
300
200
100
0
Items→
EBD_7364
220 Olympiad Champs–Mathematics
While preparing the bar graph these points should be
considered:
Ø The width of all the bars should be same.
Class Intervals Ø The space between the consecutive bars is always
the same.
A big data can be represented This is another way to represent the data.
by class intervals instead of
observation. If you have marks obtained by any of your classmate in
different subjects, then you can prepare a bar chart as
Class interval is written as a – b follows:
where a is lower class interval
and b is called upper Class Data: Mathematics → 90; Science → 80; English → 85;
interval. Hindi → 80; Computer → 60; S. St → 75
The difference of 'a' and 'b'
is called the class size or class
width class interval a – b contains
all those observation which are
equal to a and greater than a but
less than b. For example 0 – 5
class can have all the observation
less than 5 and greater than
equal to zero. 5 will be included
in next class 5 –10.
Other ways of representing
data.
Apart from Bar Graphs, data
can be represented by a pie
chat, histogram, frequency
polygon, ogives , etc but you will
Now by looking at the bar chart you can answer any
study this in higher classes.
question related to this.
Example 1. In which subject the marks are maximum?
(a) Science (b) Computer
(c) Mathematics (d) S.St
Solution (c) In Mathematics the marks are maximum.
(90).
Example 2. In which two subjects the marks are equal?
Range (a) Mathematics and Science
(b) Computer and S.St
The difference between the (c) English and Hindi
highest and lowest observation (d) Science and Hindi
is called range. Solution (d) In hindi and science marks are equal (80 in
each).
Example 3. What is the difference between the
highest and the lowest marks obtained?
(a) 30 (b) 45
(c) 23 (d) 60
Data Handling 221
Solution. (a) Highest marks: 90; lowest marks: 60
So the difference is 90 - 60 = 30.
Example 4. Growth chart: There is one more type of graphs.
Here is some information regarding the number of students taking admissions
in a college during a week?

10
9
No. of Students

8
7
6
5
4
3
2
1
0 5 10 15 20 25 30
Day

NO. OF NO. OF
DAY DAY
STUDENTS STUDENTS
0 0 20 7
5 2 25 8
10 3 30 10
15 6
Now looking at the growth graph you can answer the question:
During which days there were maximum numbers of admissions?
(a) 0-5 (b) 10-15 (c) 25-30 (d) 20-25

Solution (c) There was maximum numbers of admissions during days 25-30.
EBD_7364
222 Olympiad Champs–Mathematics

Multiple Choice Questions


LEVEL- 1
Directions (Qs. 1 to 5): Answer the following questions based on the following pictograph.
1. To make the pictograph 1000 children were asked about their hobbies. The
children who told about their hobby, has been shown in the above pictograph. But
some children did not reply. Which one of the following is the correct pictorial
representation for the children who did not reply? (Mental Mathematics)
Hobbies Number of Children

Dancing
               

Singing
           

Playing
             

Travelling
               

Adventuring
   

One represents 25 children

(a)      

(b)          

(c)        

(d)            
Data Handling 223
2. How many children are there whose hobby is adventuring?
(a) 200 children (b) 175 children
(c) 100 children (d) 75 children
3. Which one of the following hobbies is liked by least number of children?
(a) Dancing (b) Adventuring
(c) Singing (d) Playing
4. Find the number of children whose hobby is either travelling or adventuring?
(a) 300 (b) 200
(c) 100 (d) 50
5. Jack: Playing is the most favourite hobby among the children.
Codi: Not playing, it is travelling which is the most favourite hobby among the
childrens.
Who is correct?

(a) Jack (b) Codi


(c) Both are correct (d) Both are incorrect
6. The marks obtained in 4 subjects in an examination are 200. If the average of
marks obtained in 3 subjects is 48, then how many marks are obtained in fourth
subject? (2009, Critical Thinking)
(a) 50 (b) 144 (c) 152 (d) 56
7. From the following table tell the S.No. of the area from which the minimum numbers
of pedestrians passed :
S. No. Area Female Male
1 From Ghadi Chowk 132 318
to Jai Stambh
2 From Ghadi Chowk 14 286
to Railway station
3 From Ghadi Chowk 15 185
to Shankar Nagar
4 From Ghadi Chowk 22 128
to Kali Badi

(a) 3 (b) 1 (c) 2 (d) 4


8. In a village the number of members in 20 family are as follows : (2010)
6, 8, 6, 3, 2, 5, 7, 8, 6, 5, 5, 7, 7, 8, 6, 6, 7, 7, 6, 4.
The frequency of families having 6 members are :

(a)
7 (b)
5 (c)
6 (d)
4
EBD_7364
224 Olympiad Champs–Mathematics
LEVEL- 2
Directions (Qs. 9 to 15) : The number of dolls produced by a factory in different years has
been shown in the following pictograph. (Mental Mathematics)

2005
       

2006
   

2007
           

2008
     

2009
   

2010

2011
         

One represents 10000 dolls.


Read the above given pictograph and answer the following questions.

9. In which year did the factory produce maximum number of dolls?


(a) 2007 (b) 2011 (c) 2005 (d) 2009
10. How many dolls were produced in 2008?
(a)
50,000 (b)
40,000 (c)
30,000 (d)
10,000
11. In which years did the factory produce equal number of dolls?
(a) 2005 and 2006 (b) 2007 and 2011 (c) 2006 and 2009 (d) 2008 and 2009
Data Handling 225
12. Find the total number of dolls which the factory produced during 2005 to 2010.
(a)
2,60,000 (b)
2,80,000 (c)
2,40,000 (d)
2,00,000
13. How many more dolls were produced in 2007 than 2006?
(a)
20,000 (b)
30,000 (c)
40,000 (d)
10,000
14. By how much, production of the dolls increased in 2011 in comparison to previous
year.
(a)
20,000 (b)
30,000 (c)
10,000 (d)
40,000
15. In which year 60,000 dolls were produced?
(a) 2011 (b) 2007 (c) 2005 (d) 2008
Directions (Qs. 16 to 18) : Read the given bar graph and answer the following questions:

Marks obtained
90 by students
1 cm = 10 marks
80

70

60

50
Marks

40

30

20

10

0
Jenifer Katherine Helena Amelia Julia
Students

16. Name the student who got the highest mark.


(a) Katherine (b) Jenifer (c) Amelia (d) Julia
17. Find the marks obtained by Amelia.

(a)
60 (b)
80 (c)
70 (d)
50

18. What scale has been chosen in the bar graph.

(a) 1 cm = 10 marks (b) 2 cm = 10 marks

(c) 10 cm = 20 marks (d) 10 cm = 1 mark

19. If. ↑ represents 5 houses, then the number of houses represented by ↑ ↑ ↑ ↑ are:

(2014)

(a) 4 (b) 20 (c) 5 (d) 40


EBD_7364
226 Olympiad Champs–Mathematics
20. A symbol is used to represent 10 flowers. Number of symbols to be drawn to show
60 flowers is: (2017)

(a) 6 (b) 12 (c) 10 (d) 10

21. A graph drawn using pictures is called:

(a) pictograph (b) circle graph (c) column graph (d) line graph

22. A graph drawn using vertical bars is called:

(a) bar graph (b) line graph (c) pictograph (d) pie graph

23. A pictorial representation of data is called __________.

(a) attractive graph (b) pictograph (c) bar graph (d) pie diagram

24. The pictograph shows the distances of towns W, X and Y from town Z.
(Critical Thinking)

Town W

Town X

Town Y

Where represents 10 km

Which of the following road maps shows the correct positions of towns W, X, Y and Z?

Z
Z Y
km

50 km 30 km
(a) 30 km (b)
20

20 km
X
Y W X W
30 km 20 km

30 km W
20 km
30 km Z
20 km X
(c) X (d)
50 km 50 km W
Z

Y Y
Data Handling 227
25. Which of the following represents 18?

(a)       (b)      

(c)        (d)    

26. The pictograph shows the number of eggs sold by a trader in three days.
(2011, Tricky)

Monday
   
Tuesday
       
Wednesday
     


  represents 25 eggs.

If the trader still had 115 eggs left after the three days, calculate the number
of eggs he had at first.

(a) 185 (b) 300 (c) 215 (d) 415

27. The bar graph shows the grades obtained by a group of pupils in a test.

14

12

10
Number of Pupils

0
A B C D E
Grade

If grade C is the passing mark, how many pupils passed the test?

(a)
10 (b)
14 (c)
24 (d)
30
EBD_7364
228 Olympiad Champs–Mathematics
Directions (Qs. 28 to 32) : Read the following graph and answer the questions given below:
(2015, Critical Thinking)

28. At what subject is the student sharp ?


(a) English (b) Mathematics (c) Science (d) History
29. In which subject the student gets the least marks ?
(a) English (b) Mathematics (c) Science (d) History
30. What are the average marks obtained by the student ?

(a)
57 (b)
63 (c)
80 (d)
48
31. What is the percentage obtained by the student?

(a)
80% (b)
63% (c)
57% (d)
90%
32. What is the ratio of the highest marks to the lowest marks obtained by the
student?
(a) 2 : 11 (b) 9 : 2 (c) 2 : 9 (d) 11 : 2
33. The mode of a set of observations is the value which
(a) occurs most frequently
(b) is central
(c) is between maximum and minimum
(d) none of the foregoing
34. The difference between the maximum and the minimum observations in the data is
(a) class interval (b) frequency
(c) cumulative frequency (d) range
Data Handling 229
35. The number of times a particular item occurs in a class interval is called its
(a) mean (b) frequency
(c) cumulative frequency (d) none of these
36. The statistical data are of two types. These types are
(a) technical data and presentation data
(b) primary data and secondary data
(c) primary data and personal data
(d) none of the above
37. Following are the percentages of marks of 10 students of a class: (2016)
50, 60, 70, 72, 74, 75, 80, 88, 75, 100
Frequency of students in the class (60 – 68) will be:
(a) 5 (b) 1 (c) 6 (d) 3
38. The mode of the following data:

Size 1 3 5 7 9
Frequency 6 9 12 3 15
is

(a)
9 (b)
7 (c)
5 (d)
3
39. A photographer came to take the pictures of a student. There was a choice of five
different picture packages. The given chart shows how the number of wallet-sized
pictures increases from package to package. If the increase is the same from one
package to the next, how many wallet-sized pictures should be in package C?
(2017)

Package A 8 wallet-sized pictures


Package B 16 wallet-sized pictures
Package C ?
Package D 32 wallet-sized pictures
Package E 40 wallet-sized pictures
(a) 18 (b) 20 (c) 22 (d) 24
40. The given table shows that for Science class, Mr. Satish is ordering kits that
contain caterpillars. Based on the data in the table, what will be the total number
of caterpillars in 8 kits? (2010, Tricky)

Caterpillar kits
Number of kits 3 4 5 6
Total number of 18 24 30 36
caterpillars
(a) 48 (b) 38 (c) 40 (d) 42
EBD_7364
230 Olympiad Champs–Mathematics
41. Riya asked the students in her class if they have any pets. Her results are shown
below.
• 4 students have cats.
• 8 students have fish.
• 5 students have parrots.
• 12 students have dogs.
• No student has a frog. (2011)
(a) Cats are better pets than Fishes (b) Fishes are better pets than Dogs
(c) Frogs are the most popular pet (d) Frogs are the least popular pet
42. Megha has holidays on every second and fourth Saturday of a month. On which
dates she has holidays in January 2013? (2012)

(a) 1st and 15th (b) 8th and 29th


(c) 1st and 22nd (d) 12th and 26th
43. The given graph shows the number of children who visited an old age home in
5 days. Study the graph carefully and answer the given questions. (2013)

On which two consecutive days was there a difference of 80 in the number of children
who visited the old age home?
(a) Monday, Tuesday (b) Tuesday, Wednesday
(c) Wednesday, Thursday (d) Thursday, Friday
Data Handling 231
44. If one is added to the second digit of each number in given set of numbers, then
which number will be lowest? (2013)
489 441 454 473 483
(a) 441 (b) 489 (c) 454 (d) 473
45. The given bar graph shows the weight of old newpapers collected by 2 classes over
5 days. What is the difference between the total weight collected by class 5A and
class 5B? (2014)

(a) 20 kg (b) 16 kg (c) 24 kg (d) 9 kg


46. There is a connection between the numbers in the first row and those in the second
row. Find the missing number. (2016)
 First row : 15 21 34 12 19
 Second row : 105 147 238 84 ?

(a)
133 (b)
143 (c)
193 (d)
173
47.
Study the graph carefully. (2016)

If the salesman earned ` 150 from the sale of each massage chair, how much did he earn
in May?
(a) ` 6000 (b)
` 9000 (c)
` 12000 (d)
` 15000
EBD_7364
232 Olympiad Champs–Mathematics
48. The table below shows a baby’s weight on the first day of each month from April to
August. (2015)
Date Weight (in kg)
1st April 3.6
1st May 5.0
1st June 6.0
1st July 6.7
1st August 7.2
In which month did the baby’s weight increases the most?
(a) April (b) May (c) June (d) July
49. The given graph shows the number of toy cars some children have. (2016)
Number of toy cars

60
50
40
30
20
10

Samay Trishu Kiara Pratik


________ has thrice as many toy cars as the total number of toy cars Samay and Kiara
have.
(a) Trishu (b) Kiara (c) Pratik (d) Samay
50. The given bar graph shows the number of people like different types of fruits.
Study the graph and answer the question that follows (2015)

If number of poeple like grapes is double to the number of people like apple, then find the
total number of people in the survey.

(a)
1450 (b)
1500 (c)
1400 (d)
1350
Data Handling 233
RESPONSE GRID
1. a b c d 2. a b c d 3. a b c d 4. a b c d 5. a b c d
6. a b c d 7. a b c d 8. a b c d 9. a b c d 10. a b c d
11. a b c d 12. a b c d 13. a b c d 14. a b c d 15. a b c d
16. a b c d 17. a b c d 18. a b c d 19. a b c d 20. a b c d
21. a b c d 22. a b c d 23. a b c d 24. a b c d 25. a b c d
26. a b c d 27. a b c d 28. a b c d 29. a b c d 30. a b c d
31. a b c d 32. a b c d 33. a b c d 34. a b c d 35. a b c d
36. a b c d 37. a b c d 38. a b c d 39. a b c d 40. a b c d
41. a b c d 42. a b c d 43. a b c d 44. a b c d 45. a b c d
46. a b c d 47. a b c d 48. a b c d 49. a b c d 50. a b c d
EBD_7364
234 Olympiad Champs–Mathematics

Solutions with Explanation


LEVEL- 1

1. (d) One represents 25 children.

There are 33 in the pictograph. Which tells 33 × 25 = 825 children replied.

Thus, number of children who did not reply = 1000 – 825 = 175.

Thus correct pictorial representation for 175 children is

2. (d) 3 × 25 = 75. There are 75 children whose hobby is adventuring.


3. (b) There are only 75 children like adventuring. Thus adventuring is liked by least
number of children.
4. (a) Number of children whose hobby is travelling = 9 × 25 = 225
Number of children whose hobby is adventuring = 3 × 25 = 75
Total number of children whose hobby is either travelling or adventuring
= 225 + 75 = 300.
5. (b) Number of children whose hobby is playing = 7 × 25 = 175.
Number of children whose hobby is travelling = 9 × 25 = 225.
Thus travelling is the most favourite hobby among the children.
6. (d) Total marks obtained in 4 subjects = 200
Average of 3 subjects = 48
Total sum of 3 subjects = 48 × 3 = 144
Marks in fourth subject = Total marks obtained – Sum of marks in 3 subjects
= 200 – 144 = 56.
7. (d) Total numbers of pedestrians passed from the following areas :
1.  From Ghadichowk to Jai Stambh = 132 + 318 = 450
2. From Ghadichowk to Railway Station = 14 + 286 = 300
3. From Ghadichowk to Shankar Nagar = 15 + 185 = 200
4. From Ghadichowk to Kali badi = 22 + 128 = 150
From above data, it is cleared that minimum numbers of pedestrians passed from
Ghadichowk to Kali Badi.
8. (c)
Data Handling 235
LEVEL-2

9. (a) Pictograph shows maximum number of in 2007. Therefore, 2007 is the

year in which maximum number of dolls were produced.

10. (b) There are 4 in 2008. One represents 10000 dolls.

Thus 4 = 4 × 10,000 = 40,000.

11. (c) 2006 and 2009 have equal number of . It shows that the factory produced

equal number of dolls in 2006 and in 2009.


12. (c) Total number of dolls produced during 2005 to 2010 = 50,000 + 30,000 + 70,000
+ 40,000 + 30,000 + 20,000 = 2,40,000.
13. (c) The number of dolls produced in 2006 = 3 × 10,000 = 30,000.
The number of dolls produced in 2007 = 7 × 10,000 = 70,000.
Difference = 70,000 – 30,000 = 40,000.

14. (d) The number of dolls produced in 2010 = 2 × 10,000 = 20,000.


The number of dolls produced in 2011 = 6 × 10,000 = 60,000.
And 60,000 – 20,000 = 40,000.
15. (a)
16. (a) Katherine got the highest mark.
17. (c) Amelia obtained 70 marks.
18. (a) 1 cm = 10 marks.
19. (b) 5 × 4 = 20
20. (a) 6 × 10 = 60
21. (a) 22. (a) 23. (b) 24. (b)
25. (a)
26. (d) Total no. of eggs at first =
 (12 × 0) + 115
= (12 × 25) + 115
= 300 + 115 = 415
27. (d) 28. (b) 29. (d)
EBD_7364
236 Olympiad Champs–Mathematics
30. (a) Total marks = 55 + 90 + 40 + 80 + 20 = 285.
285
Average = = 57 .
5
285
31. (c) Percentage = ×100 = 57%
500
32. (b) Highest marks = 90
Lowest marks = 20
Ratio = 90 : 20 = 9 : 2
33. (a) By definition, mode is the value which occurs most frequently.
34. (d) 35. (b) 36. (b)
37. (b) Frequency of students in the class (60 – 68) is only 1.
38. (a) From the table, it is clear that the size corresponding to the maximum frequency
(15) is 9. Hence, the mode is 9.
39. (d) Package C has 24 wallet - sized pictures
40. (a) 8 kits will have 8 × 6 = 48 caterpillars
41. (d) No student has a frog. So, frogs are the least- popular pet
42 (d) On 12th and 26th January 2013, She has holiday
43. (d) Thursday, Friday as 520 - 440 = 80
44. (a) 441 as it becomes 451 which is lowest
45. (d) Total weight collected by class 5A = 28.5 kg
Total weight collected by class 5B = 19.5 kg
\  Difference = 9 kg
46. (a) 19 × 7 = 133
47. (c) 80 × 150 = `12000
48. (b) May
49. (c) Prank has thrice as many toy cars as the total number of toy cars samay and Kiara
have.
50. (c) New number of people like grapes = 2 × 150 = 300
\  Total number of people = 300 + 250 + 400 + 150 + 300 = 1400

You might also like